You are on page 1of 118

The Residents Guide to the LMCC II

5th Edition 2007-2008


The Licentiate of the Medical Council of Canada Exam, part II, also known as the MCCQE II, was the traditional means of qualifying for a
general license to practice medicine in Canada. Now that both the internship year and the general license are no longer available, many
residents view the exam as a stressful and expensive exercise in futility. While the process is stressful and expensive, it need not be futile.
Preparation for the exam can be an enlightening review. Scenarios tend to repeat over the years, the pass rate is greater than 95% on the first
attempt, and there is an option to rewrite, so dont panic.
The exam is an OSCE (Observed Scenario Clinical Exam) in which the candidate progresses through a series of stations. Your starting point
is determined alphabetically. At each station there is a physician examiner and either a real person posing as a patient or a telephone over
which you must speak to a patient or another physician requesting assistance.
The most recent sessions (since 1997) contain six short cases known as 5-minute couplets, in which the candidate is allotted 5 minutes to
assess a patient and 5 minutes to write short answers to questions related to the case. There was also a series of six longer cases in which the
candidates were presented with a more involved clinical problem, such as a resuscitation or psychosocial counseling session, lasting 10
minutes each. The physician examiner may ask one or two questions in the last minute of a 10-minute station. There is one minute between
stations during which you can look at a brief description of the patient and consider your approach. Occasionally pilot questions will be
included in the exam, which will not count towards the final mark but are used to test new questions. You will not know which questions are
pilot questions.
The content of the exam is general medicine. This means family practice & emergency medicine. The following topics appear consistently:
Pediatrics diarrhea, development, neonatal jaundice, asthma
Obs/Gyn amenorrhea, vaginal blood, abdominal pain, PIH, OCP, elective abortion counseling
Suturing choice of suture, tetanus vaccine
Chest Pain read CXR, ECG
Resuscitation fluid resuscitation after blood loss, ABCDs
Overdose ASA, TCA
Needle stick AIDS, hepatitis, vaccinations
Psychiatry depression, mania, schizophrenia
Neurosurgery back and neck radiculopathies, carpal tunnel
(Note that every history should include name, age, occupation, past medical history, family history, medications, drugs/alcohol, review of
systems)

At each PEP station, there is a sheet listing possible investigations e.g. electrolytes includes Na, Cl, K, etc. Liver enzymes include
AST, ALT, ALP, LDH, bilirubin etc. Renal tests include Urea and Creatinine. Coagulation studies include PTT, INR.
The scoring has the mean as 500, with a standard deviation of 100. On this exam you needed to score 374, and pass 9 stations in order
to pass the exam. And you have to demonstrate ethical behaviour and skills appropriate for a physician

LMCC General Review


History
Key Questions:
Adult:
1. Location; Severity; Timing; Aggrevating/Relieving factors; Associated symptoms
2. Why is that a worry to you?
3. What cant you do now that you can do before? How has it affected your ability to work?
4. Past Hx & Family Hx
5. Medications
6. Tobacco/EtOH/Drugs
7. Psychosocial eg. home situation
8. Review of Systems
Child:
1. Prenatal
2. Natal method; complications
3. Prenatal APGARS; onset of respirations; birthweight
4. Feeding Hx vit and iron supplements; solids
5. Developmental milestones eg. head up while prone; roll over, sit, walk, speech, dress alone
6. Sleep, toileting, sexuality if older (HEADSS)
7. Immunizations
CAGE Questionnaire
1. Felt the need to Cut down?
2. Annoyed by criticism?
3. Guilty feelings?
4. Eye-opener to steady nerves or get rid of hangover?
Abuse:
1. Many women tell me that someone at home is hurting or abusing them. How is it for you? Did somebody hit you?
2. Most parents get upset when their child cries or has been naughty? How do you feel? What do you do to discipline? Are you afraid
you may hurt your child?
Sex:
1. Any problems or concerns with your sexual function?
2. Maintained interest/appetite for sex? Setting?
3. Erection (eg. morning erections)? Ejaculation? Masturbation?
4. Changes in relationship or life situation?
5. General health (eg. smoking, peripheral vascular disease, meds, ETOH,
depression)

Symptoms & Approaches


A. GI
1. Abdo pain: assoc. w/ antacids, alcohol, eating, medications, defecation,
urination & menstruation
2. Bowel function: nocturnal diarrhea
3. Nausea/vomiting
4. Weight
5. Appetite
6. Jaundice: color of urine/stools; pruritus
: hepatitis risks (blood transfusions, IVDU, sexual contacts, etc.)
B. Urinary Tract
1. Voiding: urgency, frequency, dysuria, hesitancy, straining, weaker stream,
dribbling, nocturia, polyuria, incontinence; polydipsia
2. Hematuria: beets, medications
C. OBSGYNE:
1. Gravida Preterm Term Abortion Living
2. Parity

2. Menarche; Menopause
3. Periods: regularity, duration, onset, amount, premenstrual symptoms
(mood, weight, mastodynia, headaches) menstrual discomfort
5. Amenorrhea: r/o pregnancy (frequency, N/V, fatigue)
4. Abdominal or pelvic procedures
5. Infections; Vaginal discharge; Sores or Lumps; Pruritus
6. Partners: number; multiple; high-risk, same-sex
7. Contraceptive Hx
8. PAP smear
9. Menopausal symptoms: hot flushes, dyspareunia, incontinence, depression
D. Musculoskeletal:
1. Joints: pain, swelling, redness, heat, stiffness, location, symmetrical,
migration, limitation of motion
2. Activities: ADLs; climbing stairs; sitting; standing up from chair; pinch;
writing
3.Generalized symptoms: fever, anorexia, weight loss
4. Rashes, Nails, Conjunctivitis, GI, Urethritis, Preceding sore throat
E. Nervous
1. Weakness: onset, progression, location, distal vs. proximal (eg. tripping for
distal leg weakness)
2. Seizure disorders; Head injuries
Differential Low Back Pain/Leg Pain
1. Common Low Back Pain
-pain relieved by rest, aggravated by moving, lifting or twisting motions
-lumbosacral area to posterior thighs but not below knee
2. Sciatica
-shooting pain to below knee in dermatomal distribution (L5, S1)
-paresthesia and possible local weakness
-pain on straight leg raising, decreased reflexes esp. ankle jerks
3. Lumbar stenosis
-worse with walking and improves with flexion of spine
4. Nocturnal Back Pain - r/o malignancy
5. Referred Pain - eg. pancreas, aortic aneurysm, peptic ulcer
6. Vascular
-CHECK DISTAL PULSES
-claudication: improves with rest not position, skin trophic changes
-in arteriosclerosis obliterans, relief with legs dependant
-in venous insufficiency, may have pigmentation and ulceration; relief with leg elevation
Seizures
A. Partial - unilateral or focal
1) Simple - Motor (Jacksonian); Sensory; Autonomic (eg. epigastric discomfort, pallor or
flushing); Psychic (flashback or hallucinations)
2. Complex - consciousness impaired; automatism may develop, aura
B. General - bilateral, consciousness impaired, no aura

PHYSICAL EXAM: Inspection - Palpation - Percussion - Auscultation


A. Eye
a) Pupils - anisocoria (pupil inequality <0.5 mm)
b) Narrow-angle glaucoma - crescentic shadow cast
c) Inspect reflection in corneas for symmetry - strabismus (esotropia vs exotropia)
d) Afferent pupillary defect (Marcus Gunn) - swinging flashlight test
e) Argyll Robertson - do not react to light but reacts to near effort = CNS syphilis
f) Diabetic retinopathy - microaneurysms, neovascularization, dot/blot hemorrhages
g) Hypertension - cotton wool, hard exudates, copper wire arteries
B. Ears
a) Normal range 300 to 3000 Hz
b) Weber test (lateralization)
-in conductive hearing loss, sound is heard in impaired ear.
-in sensorineural hearing loss, sound is heard in good ear
c) Rinne test - AC>BC in sensorineural hearing loss

C. Respiratory
a) lower border: anterior (6th) and posterior (10th) at inspiration
b) check for cyanosis and clubbing
c) tactile fremitus - decreased with pleural effusion & pneumothorax; increased with
pneumonia
d) normal diaphragmatic excursion = 5-6 cm
e) bronchial breath sounds - loud and long expiratory phase with high pitch
f) egophony, whispered pectoriliquy = consolidation
D. Cardiovascular
a) JVP - axvy (atrial contraction-relaxation-filling-emptying); 4 cm ASA
-a waves increase with tricuspid stenosis and disappear w/ a.fib.
b) S3 (ventricular gallop) - ventricular overloading (heart failure); S4 (atrial gallop) increased ventricular stiffness (eg. hypertensive cardiomyopathy)
c) Splitting physiological increases with inspiration; paradoxical split in LBB and
widened split in pulmonic stenosis
d) BP -in both arms (differential should not be >10 mm); check for postural
hypotension (systolic drop > 20)
-cuff: length equal to 80% and width equal to 40% of limb circumference
-pulsus paradoxus - drop of >10 in systolic indicates tamponade, constrictive
pericarditis or obstructive airway disease
e) Apical impulse - increase duration w/ hypertension; displacement with enlargement
f) Murmurs:
i) Sit up and lean forward to auscultate aortic murmurs; innocent murmurs
usually disappear on sitting or leaning
ii) Grade 5 - steth partly off chest; 6 - entirely off chest
iii) Pansystolic murmurs - regurg. across AV valves
iv) Early diastolic - regurg. across semiluminar or aortic
v) Midsystolic - aortic stenosis & innocent murmurs
vi) Presystolic & middiastolic - AV valve stenosis
vii) Continous - PDA
viii) Increased stroke volume (eg. squatting, no Vasalva) increases intensity of
aortic stenosis but decreases hypertrophic cardiomyopathy
g) Pulsus alternans - LVF; Large bounding pulse - aortic regurg.
E. Abdomen
a) Check inguinal and femoral areas
b) Listen for bruits (aorta, iliac, renal, femoral)
c) Check upper and lower borders of liver (normal span - 6-7 cm)
d) Spleen in Traube's space (should remain tympanic even on inspiration)
e) Mention DRE
f) Kidneys, Aorta (normal width no more than 3 cm)
g) Psoas sign - raise thigh against hand; Obturator sign - internal rotation of hip
h) Intrabdominal mass obscured by contraction vs mass in wall
F. Genital
a) retroverted vs retroflexed uterus
b) thelarche: 8-13 yrs menarche: 10-16 yrs
G. Pregnancy
a) fundus at: pubic symphysis (12 w), umbilicus (20 w)
b) softening of isthmus (Hegar's), engorged bluish cervix (Chadwick)
c) breast tenderness, nausea/vomiting, urinary frequency, no menses
d) Naegele = minus 3 months and add 7 days
e) Lie, Position & Presentation
H. Peripheral Vascular
a) size, symmetry & swelling
b) venous pattern & engorgement
c) pigmentations, rashes, ulcers; gangrene - medial malleolus for venous insufficiency
d) colour, distribution of hair; trophic changes
e) check pulses & lymph nodes; temperature; cap refill; pitting edema
f) r/o DVT for leg swelling or pai
g) Trendelenburg - pt supine; elevate leg & occlude saphenous vein; ask pt to stand

-normally, vein fills from below (35 sec)


-release compression (normally, no additional filling)
h) Allen's test
i) Marked pallor on elevation suggests arterial insufficiency with unusual rubor on
depenency; cyanosis on dependency suggest venous insuffiency
I. Muskuloskeletal
General approach:
Inspection - swelling, redness of jts; deformities; surrrounding tissues (eg.
atrophy)
Palpation - bony landmarks; heat; tenderness; crepitus; CHECK PULSES
Range of Motion
Strength
a) Hands & Wrist
i) make a fist
ii) flex & extend, ulnar & radial deviation
iii) Heberden's nodule - OA of DIP; snuff box for scaphoid fracture
b) Elbows
i) carrying angle
ii) flex/extension; pronation/supination
iii) lateral epiconylitis (tennis elbow) with pain on extension
c) Shoulders
i) landmarks: acromion, coracoid, clavicle, greater tubercle of humerus,
scapula
ii) reach behind back to test abduction/adduction & internal & external
rotation
iii) Apprehension test; Impingement (supraspinatus weakness [abduction at
50 deg; external rotation weakness; impingment in external & internal rotation)
iv) Drop arm test for rotator cuff tear - unable to lower arms smoothly
v) Yergason - for bicipital tendinitis
d) Ankle & Feet
i) dorsi/planter flexion (tibiotalar jt); eversion/inversion at two levels
(talocalcaneal and transverse talar jts)
ii) drawer sign & Thompson sign
iii) hallux & tallus valgus/varus; pes planus
e) Knees
i) bulge sign; balloon sign; ballotment test - effusion
ii) Patellar apprehension test; Patella-Femoral grinding test (chondromalacia);
McMurray test (external rotation & valgus stress) & Appley stress test;
Anterior Drawer test & Lachmann & Sag test; Collateral ligaments; Jt line tenderness
f) Hip
i) leg length - ACIS to medial malleolus; flex knees
ii) Trendelenburg - test gluteus medius
iii) Thomas test - flexion deformity
J. Nervous System
a) Corticospinal tracts cross over at medulla; spinothalamic (pain/temp) + crude touch
cross over at level of cord; posterior columns (fine touch + vibration) cross over at
medulla
b) C3 - back of neck; T4 - nipple; T10 - umbilicus; L1- inguinal; L3- knee; L4- lateral
cutaneous; L5- first dorsal interspace (superficial peroneal); S5 - peroneal
c) Trigeminal nerve - facial sensation; jaw clench
d) Body position; involuntary movements; muscle bulk
e) Tone, Strength (1- flicker; 2 -gravity eliminated)
f) Opposition of thumb (median); finger abduction (ulnar)
g) Tinel & Phalens for carpal tunnel
h) Coordination - alternate movements (tap hand w/ ball of foot), point-to-point
i) Gait - heel-to-toe (tandem), walk on toes & heels; shallow knee bend, rise from sitting
j) Romberg (close eyes for 20 sec) with feet together
Pronator drift - pronation or downward drift suggests contralateral lesion of
corticospinal; upward or sideways suggest loss of proprioreception
-tap arms downwards: weak arms easily displaced and remains so; may not
correct if loss of proprioreption
k) Start distal when testing sensation
l) Testing sensory cortex - stereognosis; graphesthesia, two-point discrimination

m) Babinski's response (abnormal) - dorsiflexion of big toe


n) Clonus- sharply dorsiflex foot and maintain; asterixis (hepatic encephalopathy)
o) Brudzinki's - flex neck causes flexion of hips & knees; Kernig's - bilateral pain on
knee extension
p) LMN - ipsilateral upper/lower face paralysis; UMN - contralateral lower face
q) Unconcious Patient
-ABC and stabilize C-spine
-pupils (light rxn often intact in metabolic)
-ocular movements (gaze preference towards structural lesions and away from
irritative lesions)
-oculocephalic (loss suggests brainstem damage)
-oculovestibular (towards cold stimulus if intact brainstem)
-check for unusual odors, look for needle marks or head trauma, skin color

October 2009
5+5 minute stations
1) 42 yo female with low mood. Take a hx.
Q. Diagnosis, investigations, 3 things you would do to manage.
A. Depression. ?TSH, exercise, cbt, ssri
2) 55 yo male with 50-100 RBCs on urinalysis. Take hx.
Q. 3 most likely diagnoses, inv, what features on hx most important?
Hx: Patient complained of flank pain, weight loss, + + smoking hx. No signs of infection.
A: RCC vs polycystic kidney vs pyelo. Abdo renal U/S. Flank pain, weight loss.
3) 3 yo male. with hx of fever x 2 wks. Take hx
Q. 3 most likely dx, next inv, what phys exam clues are you looking for
Hx: 2 weeks daily fevers, recent red pinpoint rash on chest, easy bruising, pallor, weak, pain in lower extremeties
A. leukemia vs ITP vs ? Kawasaki (not sure but maybe HSP?)
4) 35 yo m. with neck and R arm pain. Do Phys exam
Q. Most likely diagnosis. Look at c-spine x-rays any abnormalities? Management?
Phys ex: weak triceps, reduced triceps reflexes, decreased sensation middle finger
A. C7 radiculopathy, C7 subluxation?, conservative with rehab and NSAIDs
5) 24 yo female with 24 hrs abdo pain and nausea. Do physical exam.
Q. 3 Most likely dx. next 2 investigations. 2 most common long term complications.
Phys ex: percussion tenderness max in LLQ, no HSM, when ask about DRE/Pelvic examiner tells you there is pain on L with rectal and L
adnexal tenderness, no cervical motion tenderness.
A. R/O ectopic; HCG and U/S, complications: fertility and ?bowel obstruction?
6)? Can't remember which of the next stations was 5&5
10 minute stations
1) 55 yo f. Recent colon ca dx now refusing surg. Counsel
2) 35 yo f. with poor sleep and low interest. Take hx. Q. What's the dx: dysthymia
3) 55 yo f. With chest pain in er. Manage. No cath lab available!
Initial ECG doesnt show much but then pain worsens. Must order repeat ECG! Then show tombstone STEMI
4) 18 mo male. with pallor. Take hx. Respond to mother's questions. Q. What is the most likely dx? Fe-def anemia secondary to too
much milk consumption and poor diet.
5) 60 yo male. with 2 yr hx of DM managed by diet only. Now having decreased sensation in feet bilat. Do phys exam for PVD &
neuro.
6) 35 yo male. With bipolar on Li for 10 yrs now wants to go off. Take hx and counsel.
7) 17 yo male. with recent head injury. Take hx and px, respond to pt's questions.
8) 45 yo male. Refused insurance for increased LFTs. Take hx and repond to pt's questions.
9) 20 yo female. 35 wks preg with bp 140/90 & 2+ proteinuria. Counsel pt. Giving dx, rx and risks.

October 2008
10 minute stations:
1) Mom of a 2 year old comes in because of concerns of childs diet. Wondering if the child is getting too much sugar. Grwoth chart given
and child is at 50% Ht and Wt.
Find that child is having temper tantrums
Take a history and address Moms concerns.
2) Son of a 76 year old mother who is in a nursing home with dementia. Mother has DM II and takes 30/70 15 u ac breakfast and 5 u q hs.
She was accidentally given 30 u R at hs and blood sugars OK now. The mistake was just found this morning. The patient has increased
confusion this am with marked irritation and mumbling. Prior to this she was able to converse. The son is very upset.
Address his concerns and questions regarding this incident.he wants to know how this happened, what recourse there is and is this mistake
why his Mom is now more confused and agitated.
3) 65 year old male with 2 hour history of chest pain..Manage
Once you ask for an ECG you are given an ECG to read which shows a STEMI. There is a nurse in the room to give orders to.
4) 50 year old male with low back pain for 1 week
Take a history and do a focused physical exam
You find he has L5 distribution ridiculopathy.
You are asked to make a diagnosis.
5) 42 year old woman in to talk about her Mothers meds. She has a 1 week history of increased confusion with the last 2 days worse as she is
not eating. She is now in the ER irritated, combative, disoriented.
The daughter has a list of her mothers meds and you are to give advice regarding her meds.
You find she is on:
trazadone
Ativan
HCTZ
Detrolwhich is a new med over the last week.
6) 60 year old male 6 days post op THR. He developed sudden L sided chest pain nad SOB.
Vitals 140/90 HR 120, RR 26 28
Do a focused physical exam (pt. is very distraught)
You are asked for a diagnosis and asked what 1 diagnostic test you would send him for.
Finding are consistent with a L leg DVT and subsequent PE
7) 17 year old male seen in ER 3 X over the past month with chest pain, SOB, suffocating feeling, anxiety, and feeling as though he is losing
his mind. All exams have been and are normal. You have known him for 18years as his family GP and he has always been healthy, active and
into sports..
Take a history and counsel.
You are asked for a diagnosis.
8) A husband and wife come into ER, because they got into an argument and he pushed hershe fell and lacerated her forehead. She is now
being sutured up by another physician and you are sent in to talk to the husband. He is very distraught and anxious and tearful. You find he is
remorseful.
Take a history for potential things that may cause family violence and counsel the man.
You find that they have financial problem, behavior problems with their teen age son, the husband is drinking more and the wife is upset over
that.
5 + 5 minute stations:
1) 26 year old female with a 7 day history of vaginal discharge. Take a history
Written: 1) top 3 diagnosis 2) What are her risk factors 3) What advise would you give her regarding sexual activity.

2) 30 year old female with pain to posterior neck radiating down arm. Do a focused physical.
Written: 10 interpret a lateral c-spine x ray 2) diagnosis 3) recommendations of treatment.
3) 50 year old male who drinks 4-6 beer / day, has vomited blood 1x 6 months ago and again last week. DO a focused physical exam. (you
find all the stigmata of liver disease)
Written: 1) What are his risk factors for his problem 2) how will you treat him
4) 3 year old had a first time seizure and is now in ER. You are talking to Mom.take a focused history ( you find child has had a febrile
illness).
Written 1) what is the chance this will happen again in the future 2) what tests would you order on the this child 3) what is the likely
diagnosis
5) 56 year old male had 10 mins of chest pain 1 week ago. He was seen in ER at the time and had a Normal ECG neg trops, CXR normal. He
is now in your office for FU. Take a focused history.
Written: 1) what FU test would you do 2) what are his risk factors 3) what life style changes should he make 4) what meds, if any, would you
start him on
6) 67 year old female has found blood in the underwear. Take a focused history (she is post menopausal and you must remember to ask all the
screening questions for uterine/endometerial cancer)
written: 1) what are her risk factors for endometrial cancer 2) what FU test would you order 3) top 2-3 differential diagnosis

October 2007
1. A 30 year old man with Type 1 diabetes for 20 years presents with abdominal pain, nausea and vomiting. Take a history. (He has diabetic
ketoacidosis)
PEP station on management plan.
2. A mother has questions about her 18 month old boy who is pale. Take a history. (He has iron deficiency anemia.)
3. A 71 year old woman wonders if she has dementia. Take a history and perform an initial assessment of cognitive functioning. (She has not
been taking her B12 shots or her thyroid medication.)
PEP station asking your diagnosis, your initial investigations, and your advice to her about driving.
4 A 30 year old man is brought into the emergency department after losing consciousness. He was revived by two electrical shocks. He now
feels fine and wants to leave. Manage. (The initial strip shows vtach. The current strip is normal. He admits to using cocaine.)
5. A 56 year old male with diabetes for 6 years wants to discuss the state of his diabetes. Take a history. (He is diet-controlled, does not
monitor his sugars, and has not seen a doctor regarding his diabetes for several years. At a recent eye exam, he was encouraged to see his
family doctor to discuss his diabetes.)
6. A 16 year old girl was admitted yesterday for an aspirin overdose. She is now medically cleared. Take a history to assess her fitness for
discharge.
7. A 40 year old man is complaining of worsening low back pain. Take a history and perform a focused physical exam. (He has sciatica in an
L5 distribution.)
8. A young man fell on a beer bottle an sustained a large laceration to his wrist. Perform a focused physical exam. (He cannot feel or move his
thumb.)
PEP station asking which structures have been damanged, and your initial management plan.
9. A 10 year old boy has been brought in by his mother with concerns about frequent illnesses. Take a history, perform a focused physical
exam, and counsel. (He has had 6 viral URTIs in the last 10 months.)
10. You take a phone call from a nurse from a small hospital that would like to transfer a 23 year old woman who has had a seizure. She is
febrile, tachycardic and her BP is 90/60. No orders have yet been given because the doctor is current involved in a cardiac arrest. Manage.
(She has had two brief grand mal seizures. Her neck is stiff, and she has peripheral bruising. She likely has meningococcemia.)
PEP station asking interpretation of the patients ABG.
11. A 45 year old patient has come in to discuss the results of her Pap smear, which showed CINIII; a colposcopy has been recommended.
Counsel and assess for risk factors.
12. 50 yo woman c/o severe left lower quadrant pain. Perform a focused physical exam. (LLQ tenderness with peritoneal signs.)
PEP station asking for interpretation of an abdominal x-ray, differential diagnosis and initial management.
13. A 30 year old primip at 31 weeks has been sent in by a walk in clinic for elevated blood pressure (140/90). She has 2+ proteinuria.
Discuss the diagnosis, its risks and suggest initial management.
14. A 16 year old boy is complaining of diarrhea. Take a history. (For 10 days he has had frequent diarrhea that has become bloody.)
PEP station asking for your diagnosis, a major risk of the diagnosis, and your reporting duties if any.
10 minute stations:
1. New diagnosis of a child with T1DM. Get a history from the parent and & counsel.
2. ACS young male with a suspicious history for MI. Do full ACLS assessment. Dont forget to ask about cocaine usage!
3. back pain - hx & exam

4. OCP counsel a young woman on birth control options. Get hx to r/o contraindications.
5. breast feeding - hx & counsel
5+5 minute stations:
1. T2DM - hx
2. kid w/ knee pain - exam
3. nephrotic syndrome - hx
4. abdo pain - exam, X-rays that you need to interpret.
5. phone advice re: trauma there will be a nurse on the phone asking you for step by step management of a patient that has

May 2007
1.
2.
3.
4.
5.
6.
7.
8.
9.
10.
11.
12.
13.
14.

male loss of interest in pleasurable activities low mood low energy decr conc for 4 yrs but still functioningdysthymia vs
depression treatment
panic episodes counseling RX patient phoned after a week and cancelled next appointment what to do? Want a copy of his chart do
u give?
10 month child with foster parent s having non specific diarrhea . Biological mother phone want to ask about his health what
would you tell her?
3 yr child with intermittent fever 38.5 for 2 weeks and reddish rash on stomach and chest for 1 day diffential Dx 3 tests to do
21 female for PAP counseling early sexual activity and smoker
41 female infertility menorrhagia dysmenorrhea vaginal bleeding counseling
ER station chest pain 2 hours responded partially to nitro first ECG no changesunstable angina second ECG inf MI
Crohns disease repeated vx abdominal pains X-ray multiple fluid levels Mx
CHF F/U exam
neck pain with radicular pain to the back of arm and forearm and weakness of elbow flexion and extension C6 tenderness limited
extension of neck X-rays odontoid, flexion-extension AP Lat Mx
65 male dysphagia to solids progressive hist and PE general, CNS , neck, abdo what is the single diagnostic test.
65 female shoulder and pelvic pain early morning stiffness rx bllod tests
young female bloody diarrhea for 2 days 3 most common causes inform public health.
smoking cessation counseling

October 2006
10 minute stations (8)
1. A 16 y.o. female is asked by her mother to come and see you because of a six month history of vomiting.
a. Take a relevant history.
i. (She hands you piece of paper at beginning of station: 57, 135lbs)
i. Thinks she is fat, normal BMI, self-induced vomiting
ii. Academic difficulties
iii. Frequent arguments between parents
iv. Exercises 1.5 hours/day, abuses laxatives
b. 9 min mark - What is your diagnosis?
2.

A 25 y.o. female with a history of DM is brought in by her employer unwell and confused. Has had a previous episode 1 year ago.
a. Assess and manage concurrently.
i. ABCs what IV solution do you want?
ii. Finger-prick glucose = 2.0 what intervention do you want?
iii. Symptoms resolve post intervention, repeat BG = 7.0
iv. On further history, she stayed up most of the night drinking, she took insulin R and N in the am, and she has not
had breakfast.
v. What IV solution do you want now?
vi. Turns out her previous episode was just like the current one.

3.

A 60 y.o. male POD 3 for colon cancer. Presents with sudden onset left sided sharp chest pain. HR and RR elevated. BP stable.
a. Perform a physical exam. Explain your examination and describe your positive/negative findings.
i. ABCs
ii. Patient is in obvious respiratory distress, clutching L side
iii. Left calf markedly tender, positive Homans sign.
b. 9 min mark What are your two differential diagnoses?

4.

A 25 y.o. male is stabbed in the abdomen. Hypotensive and tachycardic.


a. Physical exam, manage, and investigate concurrently. Explain your examination and describe your positive/negative
findings.
i. Primary survey. ABCs and aggressive resuscitation
ii. Pain and nausea management
iii. Order trauma labs, investigation(s), consultation(s)
iv. Secondary survey.
b. ABCs, improves with O2 and IV fluid
c. Shake tenderness and general surgery consult
d. ATLS

5.

A 68 y.o. female POD 4 right hip replacement. Patient is confused and depressed. She is refusing heparin. Confusion onset on
POD 3, progressive decline since.
a. Take complete history, do focused mental status exam and assess for capacity to consent for treatment.
i. Confused and disorientated Thinks she is in a hotel
ii. Unaware of recent O.R.
iii. Visual hallucinations
iv. No medication or alcohol history
b. MMSE 20, seeing spiders on ceiling, clearly poor insight.
c. Examiner says to forego a delirium Hx and Px. You have to address the heparin issue.
d. She ends up refusing no matter what you do.

6.

A 14 y.o. male presents with 6 months history of headache. Parent is present in the room.
a. Take a complete history
i. Bilateral headache, with phono/photophobia, nausea, and vomiting
ii. No infections symptoms
iii. No auras, no constitutional symptoms, not stress related
iv. Family history of brain tumors

7.

A 10 y.o. male brought in by his father, with a several day history of rhinorrhea, low-grade fever, and cough.
a. Take a complete history and perform a full physical exam. Explain your examination and describe your
positive/negative findings.
i. No G.I. Symptoms
ii. Sore throat, lymphadenopathy, non-productive cough
iii. Numerous sick contacts with the same symptoms
b. Fathers concerns

i. What is your diagnosis? Is it serious?


ii. How should the child be managed? (not meningitis and can kid go to friend's b-day party?)
8.

A 30 y.o. woman has questions of childhood immunizations for her six-week old baby. She emigrated from Eastern Europe 1 year
ago. She has heard that shots can make children sick (from Chatelaine magazine)
a. Address her concerns
i. Discuss risks/benefits of vaccinations
ii. Discuss the types of vaccinations
iii. Discuss schedule of vaccinations
b. Questions she asks:
i. Who administers vaccinations?
ii. Is she allowed to refuse immunizations for her child?
iii. How much do they cost?
iv. Her husband and her have not had immunizations, which immunizations should they get?

9. 40F chronic constant A/P, seen many doctors without Dx. Hx.
A On Hx she was Dx with IBS, has intermittent constipation/diarrhea.
10. 70 y.o. female unable to cope at home, wondering if should go into nursing home. Take a history and counsel.
Dx: Elder abuse (Ques: in this situation, when do you have to report to authority?)
11. 36F would like to quit smoking. Take a history and counsel on smoking cessation aides.
12. 16 y.o. female, ASA overdose. Medically stable. Assess for possible discharge. (Everyone else discharged her with a contract for safety,
but I admitted her and asked for a psych consult because this was her second overdose. She didnt seem certifiable, though, so probably
discharge was the right answer.) Be sure to address confidentiality.
13. 80 y.o. female collapses in shopping mall. Manage (history, investigations, physical.) Found to be in 3 rd degree heart block. Ask about
advance directive. Get medication history (on digoxin,) give atropine, and ask for pacing pads.
Kinda tricky cause you have to talk to the nurse and order tests, get a history from granddaughter and to physical exam/assess pt.
14. 30ish male with 6 weeks of back pain, worse in AM, improves with activity. Do a focused physical exam.
At the end examiner asks about most likely diagnosis ankylosing spondylitis.
15. 50ish y.o. female with chronic complaints wants refill of her T-3s and ketorolac. She seems like a difficult patient. gave her the
prescription and didnt. At the end you are asked about a differential diagnosis. Possible answers were: depression, fibromyalgia,
rheumatoid arthritis, somatitization disorder
16. 40ish y.o. female with chronic aches and pains. Less demanding than previous patient. Most screened for depression. Seemed like a good
idea to come up with a management plan to review her records and have her back for a full physical, as she is new to your practice. Lots
of FIFE
17. 30ish man with acute abdominal pain. Manage. He looks bad (Vitals 90/60, HR 120, RR 20, 37.8C), and you initially manage as a
possible GI bleed/perforated viscous. You find out he has had pancreatitis before and is a heavy drinker. At the end of the station his
girlfriend calls and wants to know how he is. He doesnt want her to know anything. The nurse asks what you want to say. I told her that
he was in stable condition and we were taking good care of him, and that she can call back later to talk to her boyfriend herself.
18. 12 y.o. girl with IDDM. New to your practice. Be sure to ask about ER visits, because they didnt offer this freely, but she had been to ER
recently with what sounded to be DKA. Girl is wanting to get more involved in her own management, and both girl and Mom interested
in more diabetes education.
5 + 5 minute stations (6)
1. A 35 y.o. male who is HIV positive and on HAART therapy presents with new onset diplopia. CD count is decreased (55), viral load
is elevated (30M).
a. Perform a cranial nerve examination. Explain your examination and describe your positive/negative findings.
i. Note: they have coffee grounds in the room for your use
b. Multiple-choice questions:
i. Describe the lesions
(multiple choice, pick 1)
1. You are given a photograph of a CT scan. (Multiple ring enhancing lesions)
1.
What is your differential diagnosis for the lesions?
(multiple choice, pick 3) PML, CNS lymphoma, or TB lots of choices.
ii. How would you manage this patient?
(multiple choice, pick 1)

1.

7.

Options included referrals, admission to various possible services, admission with consultations, outpatient follow-up.

2.

A 14 y.o. boy athletic male presents with anterior right knee pain that is reproducible during physical activities.
a. Perform a focused physical examination. . Explain your examination and describe your positive/negative findings.
R tibial plateau pain on exam.
b. Written Questions:
i. What is your diagnosis?
(written)
ii. List investigations that you would order, state none if none needed
(written)
iii. How do you manage this patient?
(written)
iv. The father asks you, what is the patients prognosis?
(written)

3.

A 35 y.o. female presents with a 7 month history of amenorrhea


a. Take a focused history. Hirsutism
b. Multiple-choice questions:
i. What is your differential diagnosis?
(multiple choice, pick 3-4)
ii. What investigations would you order?
(multiple choice, pick 3-4)
iii. What immediate intervention would you perform? Folic acid??
(multiple choice, pick 1)
iv. or Main relevant findings from history

4.

A 24 y.o. female is several hours post-partum. Her baby had a normal birth weight and was initially doing well. The baby is now
becoming lethargic, feeding poorly, and is becoming mildly jaundiced (bilirubin = 210, N < 200)
a. Take a focused history. Prolonged ROM with fever.
b. Written questions:
i. What is your diagnosis?
(written)
ii. Name 5 investigations you would perform to confirm your diagnosis.
(written)
iii. What intervention would you perform for the baby?
(written)

5.

A 45 y.o. female is several hours post-partum with a low birth weight baby.
a. Take a focused history. Concerned re: LBW. Smoker.
b. Despite denying substance abuse, it turns out the mother was using cocaine during pregnancy. Written questions:
i. Aside from cocaine use, what are three contributing factors in the history for the birth weight?
(written) ? wrong dates
ii. What intervention would you perform for the baby at this time?
(written) (it has great APGAR scores, and good primitive reflexes, in no respiratorydistress)
iii. What intervention would you perform for the mother at this time?
(written)
iv. Woman is depressed that the thinks she hurt her baby
1. How to manage mom
2. How to manage baby

6.

A 45 y.o. female who has never had children before, presents with a several month history of menorrhagia (volume and duration)
and spotting.
a. Take a focused history. Took TriCyclen 28 in distant past. No Pap x years
b. Multiple-choice questions:
i. What is your differential diagnosis?
(multiple choice, pick 4)
ii. What investigations would you perform?
(multiple choice, pick 4)
iii. Preventive measures

2. 16 y.o girl with heavy menstrual bleeding and cramping, missing school. Sexually active. Otherwise healthy. Take a Hx. No paps
PEP; Order 3 lab tests (lots of answers seemed right, but CBC, TSH, swabs and pap were popular responses)
What is the single most likely diagnosis (primary dysmenorrhea)
What 1 treatment would you give? (most of us gave the pill)

8. 3 month old infant with vomiting. Non-bilious, non-projectile. Sounds like GERD. You find out that she changed from breast milk to
Enfelac with iron 1 month ago. Mom a bit stressed because husband lost his job recently. Take a history.
PEP:
Interpret the growth chart given (shows normal length and HC, but decreased wt)
What is single most likely diagnosis (gerd)
List 2 non-pharmacological managements (upright after feeds, thicken feeds)
9. 30ish y.o. man with renal colic symptoms. Perform a focused exam.
PEP:
What 2 investigations would you do? (CBC, urinalysis, urine cultures, creatinine) CT shows 4mm L ureteric stone
He comes back febrile list 2 more investigations (blood cultures, CBC etc). c. Patient received Septra but is allergic to sulpha.
Upon stopping Septra, rash resolves. What three things do you need to do now?
List 3 management steps (hydration, analgesia, antibiotics)
10. 20ish man with a suspected head injury. Perform a focused cranial nerve and neuro exam. This was a tough station. He was pretty much
unresponsive. Most of us forgot fundoscopy, and I forgot babinski reflex. Otherwise, say youd do corneal and gag reflexes, check
peripheral reflexes, do a good head and neck inspection. We were all confused
You find he has a GCS ~7-8 and C-spine collar on with big laceration to his R forehead.
PEP:
List two investigations (most of us did CBC, crossmatch)
List two managements for transfer (most did oxygen, foley)
List three important contact before transfer (paramedics, ER doc, family)
What would you need to do before you transfer him? i.e. get approval from the accepting trauma physician, send chart.(MCQ)
11. 20ish woman in her 3rd trimester presents with bleeding. Has had cramping as well. Take a history.
PEP:
List 2 features on history that lead to your diagnosis (bleeding + pain)
List most likely diagnosis (abruption)
List one management step (U/S)
12. 30F with third trimester bleeding. Take a history. You find out that there is no pain. No problems during pregnancy.
a.
FHR is OK and Hgb stable. What two things on hx would help you confirm your dx?
b.
What is the most likely dx?
c.
What would you do next?
13. 3 month old infant with vomiting. Non-bilious, non-projectile. Sounds like GERD. Mom a bit stressed because husband lost his job
recently. Take a history.
PEP:
Interpret the growth chart given (shows normal length and HC, but decreased wt)
What is single most likely diagnosis (gerd)
List 2 non-pharmacological managements (upright after feeds, thicken feeds)
14. 27 yo male with nonpainful bilateral leg swelling - likely nephrotic syndrome
15. Counsel young women re: abnormal pap smear (LSIL) (and indications/risk factors)

Spring 2006
10 minute stations
1. Middle aged male comes in with wife after having 2 seizures lasting 2min each. Both were tonic clonic, generalized. Assess and
manage. Wife there, no nurse.
a. Temp elevated. Rest of vitals normal. GCS ~10
b. Important hx: alcohol abuse, but none in the last 2 days. Used to take ativan prn.
c. Assess and manage (including tests and treatments) for alcohol withdrawal seizures and decreased LOC.
d. At the end, you have to tell the wife 2 tests or treatments you would do initially for the patient.
2.

VBAC: 29yo female had a previous C section (low transverse incision) emergently for a true knot in the cord. Meconium staining
observed. Otherwise, pregnancy was normal. Now 10wks pregnant.
a. Now wanting you to ensure that she will have a vaginal birth.
b. Wants to know why she didnt have a vag birth last time (FIFE)
c. Discuss reasons for not having a VBAC and reasons for a C section and explain that she may or may not be able to have a
vag birth this time.
d. Wants a copy of her discharge summary from last time
e. Asks why her friend said that once a C/S, always a C/S
f. Asks if its safe for her to have her delivery at home

3.

Infertility: 30yo female comes in with concerns about infertility. Hx and Counsel.
a. Trying to get pregnant x 1 yrs. Never pregnant previously. One Chlamydial infection in the past. No PID.
b. Periods normal. Now measuring temperature for ovulation and has intercourse 3x/wk.
c. Poor timing during her menstrual cycle. Partner had kids from previous marriage.
d. Examiner will ask 2 most likely causes on DDx.

4.

28yo male coming in asking for refill on Percocet for tension headaches. Sees another colleague of yours normally. Very jittery and
aggressive
a. Prescription filled out only 4d ago. 50tabs. All taken
b. Denies abuse
c. Is not willing to try any other med

5.

Approximately 60yo male comes into emerg with lightheadedness and SOB. Vitals stable but HR is >150 and irregularly irregular.
Assess. Do not offer investigations or treatment options.
Find out he has some CVS risk factors. Hx of palpitations in the past. Drinks quite a bit of alcohol. HCTZ for HTN, no other meds.
No indications this is vaso-vagal or neuro (eg. Menieres disease)
a. Do CV and chest exam

6.

Mother concerned that daughter not speaking (3 yo)


a. Normal pregnancy and birth, reached all milestones
b. Speaking some phrases, but only about 5-6 words
c. Multiple AOM/yr (3-4x/yr), not always responding to verbal stimuli
d. Doesnt give a very detailed history on hearing
e. Examiner will ask likely Dx: ?hearing loss due to AOM

7.

16yo male already assessed for accident involving ankle. Only has a sprain with no ligamentous injury. Advise on rehab and the use
of crutches and tensor bandage.
a. RICE
b. Crutches and tensor bandage???
c. PT for ankle

8.

Middle aged woman comes in with fatigue and insomnia. Recently seen your colleague and has normal CBC, TSH, etc. Assess.
a. Lost her job and husband just lost his.
b. Assess sleeping habits
c. R/O depression, anxiety, psychosis, SI
d. Did mental status exam minus MMSE
e. R/O thyroid and fatigue syndrome anyways.
f. Examiner asks what you would advise: SSRI, adjust sleeping habits, social support for financial situation.

5+5 minute stations


1. 45yo male with hemoptysis. New patient to you. Focused history.
a. Smoker
b. B Sx all positive.
c. Questions:
i. Interpret CXR: cavitary lesion in RUL?

ii. Likely Dx (name 2): TB. ?Lung CA


iii. Initial management:
1. sputum cytology
2. Acid fast
3. CT?
2.

Vaginal discharge x 1wk in young female


a. One episode of PID (not very willing to give info. Only comes up when you ask about surgeries.)
b. Fishy, foul smelling. No abdo pain/other symptoms of PID.
c. Sexually active (has had 20 partners). Only on OCP. Partner has no symptoms. Pap 2yr ago normal. Smoking.
d. Questions:
i. Microbial causes: name 3
1. Gardnerella vaginalis
2. Trichomoniasis?
3. N. gonorrhea?
ii. Investigations
1. C+S of discharge
2. KOH (Whiff) test
iii. 2 causes that would warrant informing public health
1. Gonorrhea
2. Chlamydia
3. (Note: they ask for MICROBIAL causes)

3.

Mother coming in concerned about 2yo son who has been having cough x 1mo and no relief with Amoxil from other dr.
a. History of eczema. Family history of eczema and atopy
b. Father smokes in house.
c. Nocturnal cough
d. No other symptoms (including fever, wheeze, etc)
e. Questions:
i. Likely cause: reactive airways
ii. Factors in history supporting this diagnosis:
1. Nocturnal cough
2. Hx of eczema
3. FHx of atopy
iii. Management
1. inhaled beta agonists and steroids
2. counsul on smoking cessation

4.

Young female with increasing epigastric pain x 1hr. Nauseated. Vitals stable. No temp. Has history of indigestion and takes
TUMS. PEx.
a. +peritoneal signs. DRE and vag exam normal
b. Questions:
i. During station with patient: what investigation would you do (Xrays only): AXR 3 views with CXR
ii. Interpret CXR: free air under diaphragm
iii. Likely cause: perforated peptic ulcer
iv. 2 most important intial aspects of management (in next 60 min?):
1. Gen surg consult
2. iv PPI (pantoloc)?

5.

Middle aged male comes in angry because he has been denied insurance due to elevated LFT (ALT>AST, increased Alk Phos,
normal bili). Hx.
a. IVDU cocaine. Shared needles but did wash them with water inbetween. Only in teens.
b. Safe sex practices.
c. Drinks 12 beers/weekend and 2 Rye and cokes/night.
i. CAGE: denies alcoholism
d. Forgot to ask about transfusions
e. Works as a housekeeper in a hospital
f. No other systemic symptoms (fever, abdo pain, etc.)
g. LFT repeated no difference. Questions:
i. 2 most likely causes: hep C and alcoholic hepatitis (alcohol, acute hepatitis, chronic hepatitis, fatty liver on the list)
ii. without any further investigations back yet, what is your advice to patient:
1. safe sex practice, quit alcohol, test partner, stop driving on the list
2. cant remember the other 2 ?dont share toothbrushes, (supposed to be 3 in total)
3. responsibility to public health at this time (no confirmed diagnosis of hep C only LFTs)
a. I said no need to inform public health. (Multiple choice)

6.

Middle aged female coming in because of easy bruising and epistaxis. PEx.
a. Lights were off. Turn them on!!!
b. Make-up on: petechiae and bruising and epistaxis
c. PEx: skin, lymph nodes, liver, spleen, percuss chest
d. Questions:
i. Likely cause: ITP
ii. Points on history that lead to Dx? hx of flu, fever, wt loss on the list.
iii. Initial treatment has failed. What do you do now?
1. bone marrow
2. ?coag profile (was done before)/blood smear

7.

62 y o woman with acute onset thoracic back pain. Take Hx. No constitutional symptoms. Find out she had a hx of PMR and was
probably on steroids at some point. Also, family hx of early hip # in mother. PEP: list 4 risk factors for osteoporosis in this woman,
acute pain management, management of osteoporosis (3 steps with doses of meds).

October 2005
1)
2)
3)
4)

6)
7)

10 minute stations
61yo female C/O Radiation leaking to her house x10 yrs (keeping a metal ring to protect her). Focused Hx and Mental state exam
Q at 9 minutes: How would yo assess if she is competent to start on antipsychotics?
51 yo woman with back pain. Focused history and focused physical.
Pain is worse with extension and going downhill, better with flexion, normal neuro exam
Q at 9 minutes: What is your provisional diagnosis? (spinal stenosis)
30 yo male with 48hr Hx severe R hip pain, wose with ambulation, fever. Focused Hx and focused PEx
Q at 9 minutes: provisional Dx? (septic arthritis) What ONE investigation would you do? (aspirate)
80 yo woman collapsed in shopping mall. Brought by EMS to emerg where you see with one RN. Vitals 80/40, hr 40. ACLS
Given an EKG: 3rd degree heart block, unable to get code cart for 15 minutes
At 8 min nurse found DNR note in her bag. Advance directive found in wallet if you ask no intubation, no CPR, no defibrillation)
Question: what would you do if she went into Vfib and arrest?
29 yo female 10 weeks pregnant comes in to discuss current pregnancy and possibility of VBAC You are given a copy of the
discharge summary from the last delivery sectioned for decals and meconium/fetal distress. True not in cord. Take a focused history and
counsel. She also wants to do a home birth.
Foster mother brings in her 10 mo foster child to discuss 8 wk Hx of loose stools. Nurse is weighing pt. Focused Hx and advise.
Q at 9 minutes: will you disclose the information to the biological mother if she calls to see how child is doing?
41 yo male with 5yr Hxof asthma comes in to discuss 3 visits to ER in last 2 weeks for SOB, chest pain. Focused Hx,and manage.
NOTE: he was started on Metoprolol 2 weeks ago by a physician in Vancouver.
18 yo previously health patient known to you practice with history of recent visits to ER complaining of flushing, palpitations, fear,
SOB, chest pain. CXR, EKG, echocardiogram and TSH normal. Take a focused history.
Q at 9 minutes: Provisional diagnosis?
9)
Young woman, recent immigrant, poor spoken English and cant read English wants Rx refill. (none required)
10)
20 something pregnant woman at 37 weeks, first pregnancy. Has had swelling in fingers and ankles for two weeks, and has 2+
protein in her urine. Manage and counsel.
Uncomplicated pregnancy so far, one U/S at 20 weeks which was normal. Past two weeks, has gained 7.5 pounds.
Denies headaches, visual changes or RUQ pain. Negative family history. First pregnancy. No medical issues. On materna.
I said I would examine for hyperrflexia, do some blood work (CBC, LFTs, Urate), do a NST and an US for BPP.
If all is good, I would manage with labetolol and have pt induced next week, and recommended no work and bed rest at home.
Told her to come back if has decreased fetal movement, visual changes, abdo pain or seizures immediately to hospital.
If on the other hand the physical and the tests were off, I would admit her and observe her closely and induce her next week.
I explained what PIH was, briefly talked about why it happens (placenta producing some toxic material) and talked about risks to
mom (seizures, preterm labour, forgot to mention HELLP), and risks to fetus (IUGR, death)
11)
A mother finds marijuana in her sons sock drawer. Hasnt confronted him. Wants advise.
Mom wondering if using drugs periodically is ok. Kid has some new nasty friends (shop lifters), marks have been declining for 1
year. Stable home environment. I said come back with son and husband and we can talk to him about all this. Also offered resources
like drug rehab programs etc.
13)
Father and son age 6, poor attention in school, disrespectful to female authority fiqures, marks falling. On further questioning find
out mother died of leukemia in past 12 months after fighting it for 2yrs. No social support at home. No FHx ADHD. Advise.
14)
40 yo male, collapsed in a concert, cardiac arrest on route, defibrillated. Now ER, BP 160/90, HR 120, RR 20, 96% RA. Manage.
Nurse in room. Now stable, talking. No SOB now, mild chest pain. Hx shows he did coke twice in the concert. Denies doing any
other drugs. Does coke regularly. Had typical chest pain.
You have to read the strip from EMS: v.tach. Also must read the strip after cardioversion: sinus tachy. EKG in ER: no ST elevation,
ST depression V1-V4. As you are about to finish, the nurse tells you that the guy was found to have a bag of coke on him. He gets
agitated and asks if you are going to call the police.
Most people found this to be a difficult station. The guy was OK already with his ABCs. I didnt give him any meds, but I did put
him on O2, started an IV with NS at 150 cc/hr, and ran blood work including troponins (although I realized that they will be high as
the guy was defibrillated), and tox screen. I also said I would have call cardiology, have him admitted to CCU for observation, and
call social worker. Some people gave him nitro and morohine, and beta blockers to slow the heart rate as he was tachycardic. I guess
you could also consider calcium channel blockers to relieve the vasoconstriction. I said the guy arrested due to vasoconstriction
secondary to cocaine.
15)
20 something girl, comes in with fatigue and depressed mood. Make the diagnosis and come up with management plan.
Positive for depression. No mania features, no psychotic features. No stress in life or in work, supportive husband. I said we should
rule out medical conditions like thyroid, send her to counselors or start medications. Make follow up plans.
16)
40-50 something year old woman with diabetes, comes in with pain and tingling in her feet.
In 9 minutes, do a focused peripheral vascular exam and a neuro exam. In the last 1 minute, examiner will ask questions.

17)

18)

Gait is funny, as if walking on glass. Normal motor exam. Sensory exam shows loss of soft touch and pin prick bilaterally in both
legs, in stocking distribution. Also no vibration and proproception in feet. Loss of dorsal deep tendon reflexes, normal knee jerks.
Q at 9 minutes: what is the diagnosis? What is your management plan?
78 yo female on many medications. Recently seen for ? pneumonia and given Erythromycin. Vomiting and anorexic. For recheck
with CXR. Daughter states more confused lately. Presents list of medications. 2-3 benzos, Abx, heart medications. Manage meds.
Describe CXR (no convincing pneumonia)
(Polypharmacy erythromycin, ibuprofen for OA, cimetidine, digoxin, theophyllin for COPD, furosemide, ativan for anxiety after
husband died 8 months ago, triazolam for insonia)
I said we should give mother fluids, stabilize her in ER, do some blood wotk including theophilline levels, and switch her onto
inhaled steroids instead, wean off the benzos.
58 year old guy with erectile dysfunction. Counsel and manage.
Hx DM x5yr, on Metformin, ASA. Ophtho normal. HTN, on Altace. Hx CP 1yr ago, stress test negative. Not on nitro.
Married to wife x 30 years, no relationship problems. Non-smoker. ETOH 2-3 beers/week.
I said we need to control DM, get blood work (A1C, FBS, ACR), cut down ETOH, start Viagra, be aware if goes to hospital with
chest pain, let them know about Viagra. I should have also asked him to come back with his wife to talk about this more, but forgot!

5+5 minute stations


24 yo female with LLQ pain and fever. Do PE (Ectopic) (speculum exam IUD thread +, told by examiner when asked), green
discharge from cervix, blood at os, L adnexal tender, DRE tender L side
PEP
1) 3 possible DDx
2) 3 investigations
3) 5points in Hx to make the diagnosis
4?) What 2 complications can arise from her condition in the future
61 yo female, increasing SOB for 2wks.PMHx Cardiomyopathy, on ramipril, lasix, and aspirin. Focused Hx (gained 7lb, started
eating salted pickles that husband is making) (no symptoms of pneumonia/DVT. + orthopnea, + ankle edema, no PND)
PEP
1) interpretation of CXR (cardiomegaly, no pleural effusion , no Kerley B, no fluid in fissures)
2) diagnosis (CHF)
3) management, 3 points (diet counselling, stop eating pickles, increase po Lasix)
3)
18mo child took mothers ASA tablets at home, mother refusing to speak to anyone except ER doc (you)
PEP
1) Management in ER in 5 lines
2) Another sibling had same incidence in 3 yrs ago, what do you do (CAS)
3) What do you tell the mother?
3b)
Telephone in the room. Distraught mother wants to talk to ER doc. 3 year old has ingested some drain fluid.
Mother is at home with son. Grand mother present. 10 minutes before, kid found to have ingested some drain fluid from the closet,
unknown amount. Original bottle not present. They tried to induce vomiting by giving milk and mustard powder to kid, but no luck!
Kid restless and crying now.
PEP
1) FOUR things on PEx when the kid gets to ER (Vitals, orophrayngeal exam, respiratory exam and mental status)
2) Principles of Mx in ER (I said ABCs, B/W including ABG, osmolality, checking bottle/agent ingested, poison control)
4)
30yo male with MVA and femur/tib # fixed 36 hrs ago. Now c/o SOB, fever. Do PE (had rashes on the chest) (fat embolism)
PEP
1) CXR given Diagnosis
2) What 3 investigations would you order?
5)
65 yo female with bloody nipple discharge. Focused Hx (on Hx pt had mother who died at age 45 of breast Ca)
PEP: Mammo normal.
1) Given discharge is from a single duct, what is your next investigation to check for breast cancer?
2) What 3 factors will affect her prognosis?
3) What would you advise her 37 and 39 yo daughters about mammograms?
6)
55 yo male with arterial claudication symptoms. Do a focused Hx (also do screening neuro to r/o neurogenic claudication)
PEP
1) What 2 investiation would you do to confirm PVD
2) 5 risk factors for PVD, on Hx
3) Interpret ECG (normal)
7)
40 yo male, manual labourer c/o R shoulder pain after repetitive heavy lifting. Focused PEx
Cant forward flex or abduct R arm, decreased strength, unable to touch shoulder blade
PEP
1) Likely diagnosis (rotator cuff tendinopathy)
2) Next 3 steps in management (rest, anti-inflammatory, physio?)
8)
69 yo male with 3 wk Hx of progressive dysphagia. Take a history.
Smoker, 2ppd x50yrs, 6 beers/day, 15lb weight loss in 3wks. Dysphagia for solid and soft solid. FHx lung and stomach Ca
PEP
1) Presented with abnormal ba swallow. Described findings (applecore lesion mid-lower esophagus)
2) Likely diagnosis (esophageal Ca)
3) Next 3 investigations (endoscopy, )
9)
60 yo male with gross hematuria x1 week. Focused Hx
No UTI symptoms, no cancer symptoms, no prostatic symptoms. On warfarin for A fib, INR 2.6.
PEP
1) Likely diagnosis?
2) 3 initial investigations?

May 2005
5+5 minute stations
1) Male 48 yrs Leg cramps Hx & PE (PVD)
PEP Q1) Diagnosis
Q2) Investigation
Q3) Management in 3 steps
2) Male 60 yrs Hematuria Hx (on Warfarin)
PEP Q1) Describe the IVU findings (ureteric stone)
Q2) Management
3) 48 hrs old newborn with jaundice Hx (from mother)
PEP Q1) Two DD
Q2) Investigation
Q3) what are the two index youll look in the S.Bilirubin report
4) 19 yrs Female with acute lower abd pain Hx and PE (Ectopic)
PEP Q1) 3 DD
Q2) Investigation
Q3) what is the next step in her management
5) Male 60yrs acute urinary retention Hx (ca prostate)
PEP Q1) what is the Dx
Q2) Three Investigations
6) Male 27 yrs Chest pain, dyspnea,fever and cough Hx and PE (Pneumonia)
PEP Q1) Describe the CXR (Middle lobe pneumonia)
Q2) what is the most like etiological agent?
Q3) Write an out patient prescription for him
10 minute stations
1) 6 yrs old child admitted after an anaphylaxis (peanut butter), counsel the mother
2) Emergency station: Male 40 yrs Hemet emesis Managemant
3) Female 19 yrs want an abortion counsel her
4) 45 yrs old female Bipolar disorder on Lithium for 10yrs,she want to stop Lithiumsome polyuria)

Counseling (she had features of hypothyroidism and

5) 35 yrs male with acute mania Hx


At 9 min examiner asked what is the diagnosis and nest step in Mx
6) Emergency Station: Male 55 yrs old brought by paramedics after Cardiac arrest now having chest pain- management (inferior ST elevation
MI)
7) Male 35 yrs acute low back pin Hx and PE
At 9 min examiner asked Dx and Ix
8) 19 Yrs old boy with fever and sore throat and Lumps in the neck Hx and PE (Infectious mononucleosis) At 9 min Examiner asked Dx

October 2004
10 minute stations:
1) 48 yo with fever for the past 3 weeks. Focused Hx.
Night sweats, some weight loss. At an Ontario farm 3 wks ago. Smoked x15 yrs but stopped 10 yrs ago, mother died of lung cancer
2) 20 yo fell off a roof and hit his head. Manage. He is responsive but is a bit drowsy, closing eyes. Has a headache, neck pain.
Q at 9 minutes: lateral C-spine XR (if you ask for it), interpret.
3) 12 yo diabetic, just moved to town. Take a history of diabetes control, counsel
4) 2 wk old infant, breastfed. BW 3.6kg, now 3.9kg. Mother worried not breastfeeding right and not gaining enough weight. Hx, counsel.
Or Young F who missed her 6 wk postpartum appt. is here 4 months after delivery. Her baby was fine at delivery with a wt. of 3000g.
5) 80 year old woman collapsed in a shopping mall, comes in with BP 80/60, pulse 40. Manage.
ECG shown with complete heart block, get ready to intubate because poorly responsive and unstable.
Q at 9 minutes: find advanced directive in her purse saying no CPR, etc. What would you do if she went into V-fib?
6) 52 yo woman comes in to review pap smear which shows severe dysplasia suggestive of HPV/CIN III and need to go for colposcopy
Explain the findings, take a history for risk factors for cervical cancer, arrange follow-up
Q at 9 minutes: she just applied for life insurance and asks if you have to give them the results of this test.
7) 27 yo male comes in with 3 episodes of going to emerg for chest pain. SOB, palpitations. Diagnosis of panic disorder made
Explain panic disorder, answer his questions about if he is going crazy, tell him what kinds of things are available to help him
Q at 9 minutes: He wants a new MD, are you responsible for him until he finds new one? If he requests copy of his chart, do you give it?
8) Man hits wife and she hits her head on edge of table causing laceration. He says this is the first time he has ever done this and wants help
Take a history examining risk factors for family abuse, counsel
Q at 9 minutes: Are you obliged to provide police info regarding the case, and do you have obligation to inform police?
5+5 minute stations
1) 52 year old woman with LLQ pain. Focused physical
PEP:
1) Interpret AXR
2) Most likely DDx
3) Choose 5 investigations you would do (choose from a list)
2) 7 month old with diarrhea for 1 week. Focused Hx.
Mom is giving double concentrated formula for last week because she is concerned about how much weight he is gaining, although says
that weight checks fine at WBVs. Child also drinking 2 bottles apples juice/day for several months. Child is afebrile and 5% dehydrated
PEP:
1) Diagnosis (inappropriate diet vs. osmotic diarrhea)
2) Investigations
3) Management (oral rehydration, dietary advice)
3) 48 yo with shoulder pain after lifting something heavy at work. Do a PEx
PEP:
1) Most likely diagnosis (Rotator cuff tear)
2) Management (NSAIDS, physio)
3) What directions do you give him if he wanted to apply for WSIB? Will you give WSIB info about pt if they ask for it?
4) 32 yo with 3 measurements of high blood pressure. BP 160/100 both arms. Do a PEx (dont need to do BP)
PEP:
1) Most likely causes of the hypertension in him (pick 3)
2) What investigations you would do (pick 5)
3) What do you advise him (pick 3)
5) 6 week old constantly crying, mother in to see you. Focused Hx. Make sure to screen for possibility of abuse
PEP:
1) Diagnosis (most likely colic)
2) Risk factors for abuse in this family
3) Investigations, if any, you would do ( ultrasound abdo?)
6) Secondary amennorhea in a 27 yo who wants to have children. Focused Hx (has always had a bit more hair than other women)
PEP:
1) Most likely diagnoses (3)
2) Investigations (5)

May 2004
10 minute stations
1. 30 yo male presents with fall. Has bruise over L side of abdomen. In C-spine collar. Vitals stable. Manage for 1o or 2o survey. DPL
shows free fluid in abdomen. Likely splenic rupture. Acute management (type and screen, X-match, conult Gen Sx). SURGERY
2.

40 yo male inferior STEMI. Presents with CP initially, then has CP in ER with inferior ST elevations. Treat medically, consider TNK.
Question: if man wants to leave AMA (after treatment, before seen by cardiology), what do you do? Acute management. MEDICINE

3.

65 yo female with rectal CA does not want OR. Counsel her. Turns out her friend died with the same OR. Offer ostomy nurse to come
in, talk about fears etc. Hx only (counselling /CLEO) SURGERY

4.

35 yo female 8 wks pregnant worried about risk of genetic defects. Counsel. She asks if her husband didnt want abortion but she did, if
the tests were abnormal, who whould have the legal rights? Hx only (counselling / CLEO) OBS/GYN

5.

56 yo male with hemoptysis. On coumadin for A fib. Chronic hemoptysis, now worse. Ex-smoker. No CP, no DVT risks. Hx and Px.
Question: what would you do if he is dissatisfied with care and wants to take his chart to go to another MD. Hx and Px. MEDICINE

6.

Mother is worried her 3 yo daughter, still at home, not using complex sentences, just single words. Recurrent OM. Other milestones
okay, no social determinants/abuse identified on history. Brother who is 5 is okay. Mother concerned she is developmentally delayed.
Question : what is your Dx (hearing loss). History only. PAEDS

7.

49 yo female depressed x 3 wks. No SI/abuse/substance abuse. Insomnia. Soon to be laid off, husband fired. Older children at home.
Question: what would you do treatment-wise (SSRI, refer to Psych). History only. PSYCH

5+5 minute stations


1. 40s man presents with abdominal pain and vomiting. One BM before this started, now not passing gas. Pain is colicky, in waves. No
one ate with him at last meal. Prior appy. No IBD. Uncomfortable during history. History only. SURGERY
PEP:
1. Shows AXR with dilated SB loops and air-fluid levels. Interpret AXR
2. What is Dx (SBO)
3. How would you manage (NG, IV fluids)
4. If worse pain, acute abdomen, tachycardic, what would you do (3 things): (IV fluids, antibiotics, gen surgery consult)
2. 60 male with claudication. PEx for PVD (U/E and L/E). PEx only (hair distribution, cap refill, bruits). SURGERY
PEP:
1. ECG is normal. Ask you to interpret it.
2. What 2 investigations would you order for PVD (dopplers with ABI, angiography)
3. 5 risk factors for PVD (Hypertension, diabetes, smoking, hypercholesterolemia, CAD, CVD, ESRD)
3. 30 yo male schizophrenic, acute dystonic reaction. Haldol x 5 days, prior other agent. Not psychotic/suicidal. History only. PSYCH
PEP:
1. What is the diagnosis (haldol induced dystonic reaction)
2. How would you treat it (benztropine 1-2 mg IM)
3. Mother calls you back after he leaves and says that he is trying to jump off the balcony. What do you do (tell her to call
911, issue a Form 1 which you can do as you had assessed him previously)
4. 40s man presents with elevated AS T (200) and ALT (200) (ALP 110, bili 26) on screening at time of insurance. Wants test repeated. 12
drinks on weekend. CAGE equivocal. Never drank more. IV heroin as teens. Janitor at a hospital. Had perinatal jaundice but nothing
since. Previously healthy. Never transfused. No FMHx. History only (explain why Qs about sex, etc. Say must do PEx). MEDICINE
PEP:
1. What is his diagnosis (acute hepatitis)
2. What are the three likely etiologies (HBV, HCV, alcohol)
3. What is your reporting responsibility at this time (none, but if HBV or HCV, must report to public health)
5. 27 F with 1 day Hx of LLQ abdominal pain. Physical exam only. Positive rebound, shake, guarding, cough tenderness. Rectal exam
reveals L sided tenderness. On pelvic has IUD, L sided tendeness, cervical motion tenderness. No CVAT. OBS/GYN
PEP:
1. What are your 3 diagnoses (ectopic pregnancy, PID, ruptured ovarian cyst)
2. What 2 investigations would you do (abd/pelvic u/s, B HCG)
6. 2 son has persistent cough after having URTI 3 weeks ago treated with amoxil and antitussives. Child has Hx of atopy. Wheezing.
Cough worse at night. Wife smokes. Immunizations intact. No one else sick in family. Hx only.
PAEDS
PEP:
1. What is your diagnosis (reactive airways)
2. What supports it on history (wheezing, atopy, worse at night)
3. 3 management options (bronchodilators, inhaled steroids, parents stop smoking).
7. 40 yo male cant weight bear on R hip, pain x1 day, fever. No steroids/trauma/IVDU/STD RFs. ? septic hip. Hx and Px. MEDICINE
PEP:
1.
What is the diagnosis? Single investigation?
2.
How do you manage this?
150. 71 year old female who has not had a family doctor for three years. Her daughter who is your patient and has asked that you see her
because her mother is worried about problems with her memory and is worried about developing dementia.
Findings: Previously got a shot q month and used to take thyroid pills but has not kept up with these since her family doctor moved.
PEP:
1. Most likely diagnosis.
2. What are two likely causes of this diagnosis.

3. What two tests would you order to confirm your suspicions.

October 2003
10 minute stations
1. 60 yo female four days post-op uncomplicated hysterectomy for fibroids. Now confused, with visual and auditory hallucinations. Wants
to leave hospital. Receiving Tylenol #3 for pain control until 36 hours prior; Ativan 1 mg qhs for insomnia/anxiety.
Complete Hx, focused MSE. Assess for capacity to consent for treatment / investigation / discharge.
- Visual hallucinations only strangers dancing with lights; no suicidal / homicidal thoughts
- No fever, chills. No allergy to T3 or Ativan
- Previous history of hepatitis. History of heavy EtOH use; consumes 10+ drinks per day, prior to OR
- MMSE difficult; pt keeps refusing; need to be persistent; MMSE score < 24
Q at 9 minutes: Is this patient competent? If she chose to go home right now, what would you do?
History: onset of hallucinations, duration, description. Associated fever, agitation, sweating, tremor, decreased level of consciousness,
seizure? Misperceptions and illusions, impaired attention span, disorientation, impaired level of consciousness, delusional thinking, affective
symptoms (mood sad). Fluctuating course? Any problems with surgical recovery, wound healing, mobilization? Amount of alcohol
consumed at home. History of alcoholism, leg swelling, SOB, chest pain. Current state. Post-op medications (morphine, Demerol)
previous bad reactions to these or to antibiotics? Previous episode like this one? Past medical history, medications, drug and alcohol use,
smoking, allergy, family history, review of systems.
Most likely diagnosis: delirium.
Differential Diagnosis: I WATCH DEATH: I = infectious (encephalitis, meningitis, UTI, pneumonia), W withdrawal (alcohol, barbiturates,
benzodiazepines), A acute metabolic disorder (lytes, hepatic/renal failure), T trauma (head injury, postop), C CNS pathology (stroke,
hemorrhage, tumor, seizure, Parkinsons), H hypoxia (anemia, cardiac failure, pulmonary embolus), D deficiencies (vit. B12, folic acid,
thiamine), E endocrinopathies (thyroid, glucose, parathyroid, adrenal), A acute vascular (shock, vasculitis, hypertensive encephalopathy),
T - toxins, substance abuse, medication (alcohol, anesthetics, anticholinergics, narcotics), H Heavy metals (arsenic, lead, mercury).
Investigations: CBC, lytes, calcium, phosphate, magnesium, glucose, ESR, liver/renal tests, urinalysis, ECG. As indicated by history: TSH,
CT head, toxicology screen, VRDL, LP, LE preparation, B12 and folic acid levels, EEG.
Management: Treat underlying cause (GMC etc.). Stop all non-essential medications. Maintain nutrition, hydration, electrolyte
balance and monitor vitals. Psychosocial quiet/well lit environment, room close to observation, family member for
reassurance. Pharmacological haloperidol 2-5 mg IM, lorazepam 1 mg SL, physical restraints if patient violent.
2.

23 yo woman. Friend concerned re: weight loss, difficulties concentrating at work. Worried about losing her job. Father died 15 months
ago, and family now relies on her salary. Focused history.
- Works as receptionist, not want to change jobs; made errors at work, late for work
- Broke up with her boyfriend 4 months prior. Anniversary of her fathers death four weeks prior
- Positive depression screening (3 mo. hx), no suicidal / homicidal ideation
- Negative for mania screening, negative anxiety screen
- No report of abuse from home (lives with mom, a few siblings). No substance use, smoking, EtOH

3. 30 yoman with 5 hrs of acute epigastric pain, nausea, vomiting. Worsening. Vitals provided: BP 90/60, P120, RR28, 37.8C. Manage.
- History of EtOH abuse drank more the previous night watching the game. 1 prior episode of pancreatitis requiring admission
- Hx of intussception requiring surgery. No NSAID/ASA use
- Examination: diffuse peritonitis w/ guarding, no bowel sounds
Q at 9 minutes: Girlfriend calls for information. Patient does not want you to tell the girlfriend anything, saying shell nag him about the
drinking. Nurse asks What should I do?
4.

80 yo female collapsed at shopping mall. Loss of consciousness. Brought to ER by ambulance. Daughter present. BP 80/40, HR 40,
T37.2C. Manage patient with nurse. * Treat as ACLS station *
History:
- Patient complained of shortness of breath, heart pounding, dizziness just before fainting
- History of hypertension and diet-controlled type 2 diabetes; no other cardiac history. No prior history of similar presentation.
- Medications: digoxin and unknown diuretic (Spironolactone?). NKDA. Last meal 1.5 hours prior to event
Physical Examination:GCS: no response to voice, withdraws from sternal rub. Cardiac/resp exam normal. Fluid status normal.
EKG: third degree heart block, no ST changes, no QRS widening.
Investigations:Blood sugar = 6. Order bloodwork incuding digoxin level. Order CXR
Management:fluid bolus. Atropine. Transcutaneous pacer. Digibind. Hyperkalemia management
BP and HR never changed; pt never regained consciousness
Q at 9 minutes: what would you do if found DNR directive in purse and patient goes into V fib? You never get a serum dig level back so
didn't give digibind, but treated like presumed dig toxicity exacerbated by hypokalemia. The document is from a lawyer so I presume
they want us to do nothing.

5.

6 yo boy managed in the ER for an anaphylactic reaction to a peanut butter sandwich. Treated with epinephrine and anti-histamines.
Now stable and ready for discharge. Parent wants to talk to you about anaphylaxis and the peanut allergy. Advise and counsel.
- Otherwise healthy kid. Normal development. Normal diet. Okay at school.

6.

Hives with penicillin; Otherwise no Hx or FHx of such allergy / reaction. No family history of atopy
Younger sister; no reported Hx of peanut allergy
Mother wants to know symptoms to look out for, what to do next time, etc. Advise regarding use of epi-pen, medications, etc.

72yo woman with vulvar itch x2mths. Normal CPx 5 mth ago. Take Hx. You will be given info about a physical exam. Advise.
- Vulvar itchiness, dyspareunia and post-coital bleeding, no discharge otherwise. No constitutional symptoms.
Q at 9 minutes: Picture provided. (? White plaques ?) You are told that on exam she has an atrophic vagina. DDx lichen sclerosis,
vulvar Ca, atrophic vaginitis. Recommend biopsy and follow-up.

7.

42 yo woman wants to quit smoking following an episode of acute bronchitis one month prior. Take a relevant history and advise.
- 18 pack-year smoking history. Previous attempt to quit (cold turkey) failed when her father had a stroke (stressor)
- Negative screen for substance dependence (craving, withdrawal, etc.). No substance use, EtOH
- PMHx: Episode of bronchitis one month prior, no asthma history, no hospitalizations. FHx: Father has had two strokes
- Husband smokes but encourages quitting. Teenaged kids are interested in smoking. Tried quitting cold turkey 2x before, failed.
Advise re: behavioural techniques, meds (bupropion ask about seizures), patch, gum, support groups. Offer to speak to husband and
kids too. Patient wants to quit cold turkey instead of transitioning; does not want to use meds due to expense
History: Name, age, occupation. Smoking habits: amount, duration, frequency, time of day. Gain from smoking (e.g. weight loss, decreased
anxiety, social relationships). Personal concerns about smoking and quitting, foreseen benefits from quitting, interest in quitting (a person will
only quit if they are willing). Previous attempts and results, medical situation: SOB, cough, asthma, COPD, HTN. Social situation: other
smokers in family/social network. Nicotine dependence: preoccupation or compulsion to use, impairment or loss of control over use,
continued use despite negative consequences, minimization or denial of problems associated with use. Past medical history, allergies,
medications, alcohol/drug use, family history, review of systems.
Counseling: 2 important components that need to be addressed: 1) physical/chemical addiction: symptoms of withdrawal
(tremors/irritability) and 2) habitual/environmental factors: psychological, social, and spiritual components. Advise patient of health risks:
Smoking is the single most preventable cause of death responsible for 80% of lung cancers, COPD, cardiovascular disease, also a factor in
PUD, low birth weight babies, premature aging, upper GI/respiratory cancers, respiratory infections, SIDS. Ages 25-34 have highest
prevalence of smoking, 15% of smokers smoke > 25 cigarettes/day. After assessing smoking habits advise every smoker to quit at every
visit assess stage of change (see table below). Motivate smoker to attempt to quit: benefits include decreased respiratory infections,
increased exercise tolerance/energy, increased taste/smell, ask for a commitment to quit (set a date), assist the smoker to quit (physician
counseling).
Begin with self-help materials: remove ashtrays/lighters, increase high fiber snacks/gum, increase aerobic exercise, self-reward, may also use
nicotine patch/gum or attend smoking withdrawal programs. Reward goals that are met: plan for new social relationships and activities to
make it easier to make a serious attempt to change behavior. Follow-up: set firm dates. Anticipate problems: weight gain, withdrawal
symptoms. Continue to monitor/support. Do not give up if failed. Most relapses occur in first year; most people try > 5 times before
quitting.
Stages of Change Model
Barriers
Processes
Motivational drift
Maintenance
Reinforcement management
Low social support
Helping relationship

Lack of perceived self-efficacy


Action
Reinforcement
Attitudes and emotions

Lack of knowledge
Contemplation
Self evaluation

Denial/trivialization
Precontemplation Consciousness raising
Perceived invulnerability
Faulty conceptions
Treatment:
Nicotine patch continuous self-regulated amount of nicotine, decreases craving and/or withdrawal, will not replace immediate effects of
smoking, habit or pleasure. Indications: nicotine dependent, high motivation to quit smoking. Contraindications: smoking while on patch,
allergy, MI, CVA. Relative contraindication: pregnancy. Duration of treatment: 4-12 weeks usually adequate.
Zyban (bupropion) approved in Canada in August 1998 acts on dopaminergic (reward) and noradrenergic (withdrawal) pathways.
Contraindications: seizure disorder, alcoholism, eating disorder, recent MAOI use, current pregnancy; caution if using SSRI (reduction of
seizure threshold). Dose varies with amount the patient smokes. Patient continues to smoke for first week of treatment and then completely
stops (therapeutic levels reached in one week). Recommend abstinence from alcohol due to risk of toxic levels with liver dysfunction. Side
effects: headache, insomnia, dry mouth, weight gain.
8.

30 yo man with a 6 mth Hx worsening diffuse back pain. Morning stiffness; stiffness gets better with exercise. Focused PEx.

- No tenderness over CTL spine. Limited ROM of the back (flexion, extension, lateral flexion, rotation)
- Strength, sensation, reflexes all normal. Negative straight leg raise, no signs of sciatica, no signs of cauda equine, normal anal tone
- Negative Trendelenberg, hip exam normal. No flexion contracture, normal leg length. + Schobers test.
Q at 9 minutes:What is the most likely diagnosis? What other 3 associated conditions? (IBD, uveitis, arthritis, etc.)
9.

40 yo patient presents to your office for incision and drainage of a sebaceous cyst on his back. Perform the procedure. (? Pilot station)
In the room was a mock piece of skin with a sebaceous cyst in it. You had to perform the procedure under sterile technique. You had to
cut deep enough so that fake pus came out of the wound, pack it with string-gauze and dress it appropriately.

10. 36 yo female with bipolar disorder. On Lithium x 10 yrs. Wants to stop. Counsel.
- Stable course of BAD x 10 yrs. No change on Li dose; never tried other meds; previous Li level 1 month ago - normal
- Negative screen for depression, mania
- No other meds, NKDA; no smoking, EtOH
- Previous thyroid test low, no liver problems, no heart problems, +ve polyuria (frequency)
- Thought Lithium interferes with her creativity (works as a publicist, designer)
5+5 minute stations
1. 30 yo male hit by a car, sustains a closed head injury. To ER hemodynamically stable. Focused neuro exam. Do not have to do GCS.
C-spine precautions, in a collar, no airway protection, Rt face laceration. No hemotympanum, etc. GCS 8 (1E, 2V, 5M)
Any attempt to address ABCs was discouraged by the examiner. Has blood all over him.
CNs normal pupils, corneal reflex, gag reflex. Peripheral reflexes normal, normal tone, -ve Barbinski, normal rectal tone
- CXR, pelvic x-ray and lateral c-spine xray are all normal
PEP:
1. Name 2 investigations you would do prior to transport
2. 4 management steps
3. You are in a community hospital. What are the 3 most important medical actions/interventions before transport
2.

35 yo male presents with colicky left sided abdo pain. Focsed PEx.
Normal BS, no peritoneal findings, normal light palpation, normal percussion. no organomegaly
Focal tenderness on LLQ w/ deep palpation. Tenderness over the left CVA
-ve psoas, obturator sign; -ve DRE; -ve abdo bruits / pulsatile mass. -ve genital exam
PEP:
1. Interpret IVP (shows filling defect, left hydronephrosis, KUB shows stone)
2. 2 further investigations at this point
3. Patient back 2d later with worse pain & chills; T=39.2, BP90/60, HR110, RR20. 3 Investigations? 4 x Management?
4. Patient receives septra despite having an allergy to sulpha documented on the chart. He breaks out into hives, receives
benadryl, and is given another antibiotic instead. What are your responsibilities now (3)?

3.

15 yo female presents with painful periods, causing missed school. Focused Hx.
Menarche at age 12. Heavy painful periods for six months, flow getting heavier over the last six months
LMP 6d ago, regular, 4 pads needed. No BCP use
Now using mothers Tylenol #3s for pain, prior was using about 10 ibuprofen a day
Sexually active w/ 1 partner, condom use, +ve dyspareunia; STD Hx unknown. No unusual vaginal discharge, no dysuria
No substance use, smoking, smoking. Doing ok at school but worries about lagging behind and grades
No Hx / FHx of fibroids, endometriosis
Asks if you will tell her mother about the conversation
PEP:
1. 1 investigation (from a list including no investigation)
2. 1 Dx (from a list)
3. 1 treatment (from a list including BCP)
4. Who should consent for Rx (2 answers from a list)

4.

42 yo male with cough, temperature, & pleuritic pain on right side of chest for a few days. Focused PEx.
VS stable, except febrile. Inspection normal, patient coughing throughout
Palpation normal (diaphragmatic excursion, focal tenderness, tactile fremitus); no tenderness. Percussion normal
Auscultation and egophony normal; decreased breath sounds on right
PEP:
1. Interpret CXR (RML consolidation
2. Most likely diagnosis and etiology (CAP, S. pneumoniae)
3. Write a prescription

5.

Mother comes with three month old boy who has been vomiting and crying. Nurse is weighing and measuring the baby. Focused Hx.
Symptoms since 1 wk old; worsening. Cries before vomiting; non-projectile; non-bloody.
Occurs 3-4X /day associated w/ feeds (1/2 hr post feed). Child able to sleep, no vomiting at night
Normal BMs, no diarrhea. Growing, but weight was down when checked at two-months of age
Breast fed until 2 wks ago, switched to Enfalac w/ iron; no change in symptoms
Term pregnancy, vaginal delivery, no antenatal complications, normal course post birth; no congenital problems; otherwise
healthy
Developmental milestones ok, able to support head and roll body, alert & responsive to mom

PEP:

6.

One older sister (6 y/o), no Hx of such problem, no dietary problems


Lives w/ husband; husband lost job recently but family is coping (no signs of child abuse)
1. Interpret the growth curve (Ht 50th %tile, Wt previously 75th now < 25th %tile)
2. Most likely diagnosis (GERD)
3. 2-3 management options other than medications
4. 3 RFs for child abuse? What is your responsibility if you suspect child abuse? (who to report to)

28 yo woman, third trimester, presents with vaginal bleeding. Focused Hx.


G1P0, currently 30 weeks gestation. Normal pregnancy course, no htn, normal lab tests, normal 18 week U/S
Bleeding started 24 hours prior, heavy. Now associated cramping over the past 6 hours prior
No trauma, no ROM (membrane rupture), no unusual D/C, no dysuria. Feels baby moving ok
No FHx of such problem. Good relationship w/ husband
Pt asks you whatll happen to her is she in labour?
PEP:
1. 2 points on history essential to diagnosis
2. Most likely diagnosis
3. Would you do a digital vaginal exam?
4. 4 things to do for treatment/management

October 2002
10 minute stations
1. 16 yo female with presumed ASA overdose. 1 previous attempt, no meds. Assess for safety/suicide risk and ability to discharge from
emergency. Took 15 ASA, didnt intend to kill herself. Her mother is high-powered lawyer who works all the time. Seen with maternal
grandmother. Was in Cuba x 2wks, staying with father, step-mom, and their new baby. Doesnt get along well with father.
Q at 9 minutes: do you have a duty to tell her parents.
2.

25 yo pregnant female. G2T0P0A1L0 at 30 weeks gestation. Requesting elective c-section. Uneventful pregnancy. No prenatal care.
Previous therapeutic abortion at 16yo with D&C for hemorrhage, many complications. Many social stressors and has been ill-advised by
friends. No indication for elective c-section. Demands second opinion if you say you cannot do it. Counsel

3.

60 yo man with acute chest pain. Manage. Initial EKG normal, then develops ST elevation. Acute Mx, discussion re: thrombolysis.
Q at 9 minutes: man wants to leave AMA. You have explained risks if he does so and deem him competent. What do you do?

4.

48 yo male requires regular transfusions for myelodysplastic anemia. Called to assess for fever during an elective blood transfusion.
Nurse present. Blood still running, wrong blood but correct type. Manage with nurse. No Hx transfusion reaction. 1u earlier was ok.
Advise patient regarding error. Patient receives news well.
Q at 9 minutes: what do you have to report regarding an error. Make sure you say you would call the blood bank right away and tell them
about mistake! Obviously this patient's blood is going to another person and could cause a transfusion reaction

2.

40 yo man with lower back pain, radicular signs down left leg to toe. Hx and PEx.
O/E has unilateral pain on SLR, decreased light touch sensation.
Q at 9 minutes: Diagnosis? (sciatica) What level?
5.

9 yo boy with Type 1 DM. Focused Hx. Only checks BG BID, mother does all BS and insulin. Single mother, father not involved.
Multiple hospitalizations for hypoglycemic and DKA, no meal plan, unsure of last HbA1c, glucose levels run high all the time.
Issues are primarily social recent move, no endocrinologist in city, not compliant with diet, BS checks, dietician. Counselling.
At 9 minutes, child asks to go home

6.

52 yo woman with hot flashes and night sweats. Assess and counsel this patient regarding her suitability for HRT.
Has hot flashes, vaginal dryness. Smokes. Heard about HRT from friend at work. Has no contraindications to HRT

7.

Man with ankle sprain. Counsel re: rehab, care, follow-up

8.

Parent of child with language delay. Focused Hx. Q at 9 minutes: diagnosis?

9.

Man with impotence. Focused Hx and counsel.

5+5 minute stations


1. 30 yo male presents with cough, sputum, pleuritic chest pain. Focused PEx. Right sided pleuritic chest pain.
PEP:
(1) Returns to ER in 3d, still febrile, leukocytosis. Interpret CXR (RML infiltrate)
(2) What is the diagnosis (CAP)
(3) What is the most likely pathogen (S. pneumoniae)
(4) List 3 management steps
2.

40-ish yo female with shooting leg pains when she flexes her neck. Also fleeting visual symptoms and urinary retention. (Alternative
Hx: sudden left hemiparesis). FHx of MS. Similar episode 4 mths ago, visual symptoms and dysphagia, well between episodes. Hx only.
PEP:
(1) Diagnosis (MS)
(2) What if she worsens, needs mechanical ventilation, living will DNR, sister pressuring you to intubate/ventilate her?

3.

Mother brings in 3 month old child c/o regurgitation after every meal. Vomiting 4-8 x per day. Non-bilious. Denies projectile vomiting.
Feeds 8 oz formula q3h. Child irritable, normal bowel/bladder, no other symptoms. Mother depressed and overwhelmed. Varies on
abuse RF. Growth curve: <5 %ile, fallen 3 curves. Hx and counsel.
PEP:
(1) What is your diagnosis (r/o pyloric stenosis)
(2) What is your management plan (referral to surgeon for further investigations)
(3) What are 4 risk factors for abuse in this child? (only child, first child, young mother, possible mood disorder in mother
. . .) What do you have to do if you suspect abuse?

4.

60s man presents to the ER with acute urinary retention. Foley: 1200cc clear urine. Hx only. Poor stream, weight loss, low back pain.
PEP:
(1) Most likely diagnosis?
(2) 4 other possible DDx?
(3) 3 investigations?
(4) Physical examination shows an enlarged prostate. Next step in management?

(5) Daughter does not want him told if he has cancer. How do you deal with that?
5.

Middle-aged man presents with acute colicky flank pain x 4 hours. Focused PEx. Physical exam shows CVAT, no peritoneal.
PEP:
(1) Interpret IVP (shows unilateral hydroureter, nephrolithiasis with stone at PU jxn)
(2) DDx
(3) 3 investigations
(4) 4 management steps, if he presents again to emerg with fever, chills

6.

60 yo man with hematemesis. Known alcoholic, consumes 6-8 beers/day. Focused PEx. (Look for signs of liver failure, abdo exam)
PEP:
(1) Most likely diagnosis
(2) Differential diagnoses
(3) Investigations
(3) Management
(4) What do you do if you find out he is the town school bus driver?
(5) If he presents with encephalopathy, what factors (list 4) help determine prognosis?

7.

20 yo male with acute onset neck pain. Hx only. Schizophrenia. Recent changed from risperidone to haldol. Likely acute dystonia.
PEP:
(1) Primary diagnosis
(2) Acute management
(3) Mother calls back in 2d, son was on roof trying to fly. What are your obligations now?

8.

Mother of child with chronic cough, unresponsive to ABx. Focused Hx


PEP:
(1) DDx
(2) Key elements from Hx

(3) Investigations

Young man with LLQ pain. PEx


PEP:
(1) DDx

(2) Key elements from Hx

(3) Investigations

10. Woman with menorrhagia. Focused Hx


PEP:
(1) DDx
(2) Key elements from Hx

(3) Investigations

11. Man with calf claudication. PEx


PEP:
(1) Interpret EKG

(3) Investigations

9.

(2) Risk factors

12. Man with SOB and sputum. Focused Hx


PEP:
(1) ER management
(2) Investigations

(3) Advice re: prevention

October 2001
5+5 minute stations
1) 40 yo male, periumbilical abdo pain x24 hrs, febrile. Focused Hx. LBM 24hrs ago, Hx appendectomy
PEP:
1) 3 views of abdo xrays - list 3 negative or positive findings on exam
2) Lab results given - give diagnosis (SBO?)
3) 3 treatments you would do in E.R.
4) When, if ever, would you consider surgical intervention
2) 29 yo female with 2-3d Hx lower abdo pain. Focused Hx and PEx.
Pt febrile, tachycardic. Abdo guarding, rigid, peritoneal, severe pain LLQ, wont straighten legs. DRE - ve.
Gyne exam - ask specifically about speculum exam, appearance of cervix and os and any discharge.
Ask about bimanual palpation- cervical tenderness, adnexal tenderness, any masses.
PEP:
1) What 5 things on history would help determine diagnosis.
2) What are the 3 most likely diagnosis.
3) Told chlamydia grown on culture -What 2 responsibilities do you have to public health concerning this issue (3)
3) 81yo male for follow up of bloodwork after seen by colleague c/o dizzy, light-headed, ataxia. Pancytopenia, MCV 120. Focused Hx.
Peripheral neuropathy, probs walking at night. Tingling. Some mental symptoms. No consitutional illnesses. No ETOH
PEP:
1) What is most likely diagnosis.
2) What other potential Dx for neurologic Sx (peripheral neuropathy 2o lymphoma, gastric, causing b12 deficiency, or DM)
3) What 2 investigations would help confirm diagnosis
4) Pt is a school bus driver, forgot to pick up 2 stops the other day, wants to keep driving. What advice would you give.
4) 81yo female with complaint of severe headache. Focused Hx. jaw s/s and R visual changes. No afib/other Hx. No other findings on hx
PEP:
1) Most likely diagnosis- temporal arteritis
2) 2 things on history that confirm the diagnosis
3) Most common complication of this condition
4) What investigations would you order to confirm diagnosis (3)
5) How would you manage this patient
5) 50 yo male with hx of PVD. Focused PEx.
PEP:
1) What 5 things on history would be important to know
2) What 2 investigations would you order for this diagnosis
6) Young mother brings in 4 mos old infant for crying and iritability x weeks. Nurse is weighing infant currently. Focused Hx (r/o abuse)
Baby crying 3hrs/day, >3x/week = colic. Hasn't tried usual stuff to comfort colicky baby. Father not home much. Gassy after eating.
PEP:
1) What is the most likely diagnosis.
2) What 1 investigation (if any) would you order.
3) 4 things on FHx that make you suspect child abuse. (This came out of nowhere for all of us; I said female child, young
mother, first baby, mother overwhelmed, or something like that.) You also get the baby's growth charts, and she is in the
50th percentile.
7) Hysterical mother on phone, child took prescription medication. Mother will only talk to doctor on call. Focused Hx.
Get her # and address right away. Kid playing in grandmother's room, took ?? BP meds, mother threw bottle out, quanitity unclear.
Kid crying. Keeps asking what to do, frantic. Moms scripts filled at a far pharmacy. Calm her down, find out where she is, send EMS
PEP:
1) 5 things you would do when the child arrived in ER
2) 3 things you would monitor specifically as they pertain to poisoning? (or something like that).
3) Find out similar incident prior with kid's sibling. Who do you call? What is your responsibility? What to tell mom?
10 minute stations
1) 70 yo woman fell down at shopping mall. Pt in E.R., vitals given. Nurse in room with you. Assess and manage patient
Do GCS (8), meds (Digoxin, Spironolactone), advanced directives
ECG: 3rd degree heart block. BP 70 systolic, HR 40. HR 70 after 0.5m Atropine. No problems with fluid bolus
Q at 9 minutes: Given advanced directive of DNR, how would you further manage this pt? If she goes into VF, what would you do?
Make sure you tell them that you would give supportive measures i.e. pain control and that you would write a DNR order.
2) 35 yo female comes to your office with complaint of lethargy and decreased interest in activities. Focused Hx and counsel.
Young children at home, no PMHx, no Rx, no SI. Hx low mood and energy x4yrs. Likely dysthymic.
Q at 9 minutes: is there something that can be done about her condition? Wants reassurance
3) 60 yo female c/o radiation leaking into house, Russians are after her. Focused Hx and PEx, assess competence for consent to Tx.
Not suicidal or homicidal. Fulfills some criteria for schizophrenia. No time or paper in room to do a Folstein.
Says she will consent to a treatment if it can make the tingling in her arms stop (which she says is from radiation).

4) 30 yo male at home with wife, onset severe nausea and vomiting, called 911. RN in room. Manage, giver orders to RN.
++EtOH, Hx pancreatitis. No cardiac features, EKG normal. Epigastric pain, maybe slightly hypotensive.
I gave him some demereol, kept him NPO, etc
Q at 9 minutes: pt's wife phones requesting info, patient says "She'll nag me about my drinking, tell her I have the flu". Respond to this.
5) 60 yo m, lawyer, on coumadin for afib, presents w hemoptysis of 1 tbsp, very worried. Focused Hx and PEx.
o/e: no findings. Smoker. No other bleeding. INR last checked 1mth ago, getting Coumadin 5-7mg/d from FMD. No constitutional Sx
Q at 9 minutes: what if the next day, patient calls saying he is unhappy with the care you have given him and wants a copy of his chart.
6) Foster mother of 10 month old child she has cared for for 4 months. c/o baby has had 8 weeks diarrhea. Focused Hx.
No PMHx available for baby. They have been giving baby all manner of food, including adult. Diarrhea with milk and juice.
Q at 9 minutes: 2 potential Dx? (could be toddler's diarrhea and lactose intolerance, but maybe better to say diet is too advanced)
What if biological mother called that afternoon requesting info on babys condition?
7) 30 yo woman with Hx C/S for true knot in cord. 10 wks pregnant, wants to talk to you about delivery options.
Given her delivery report out in the hall. No probs with the birth, C/S unanticipated. She is irritated and upset because she couldn't have
a vag delivery, wants guarantee of vag delivery this time. Also wants to try to have home birth, seeking go ahead from you.
VBAC counselling, warning her about risks associated with home deliveries for VBAC (?), etc.
Q at 9 minutes: what if patient asks for a copy of the chart? What do you do if she refuses to give it back/wants to take it with her?

October 2000
10 minute stations
1.
32 year old mother presents to your office because her 4 year old son has had an allergic reaction to peanuts. He was brought to the
emergency departement and treated yesterday. He is now well. Counsel. Acute MI: manage with help of nurse
2.

Man in 20's fell off ladder 14 feet. Bruised over left anterior ribs. On table in c-collar. CVS, resp normal. Asking for pain relief.
Manage.

3.

Woman with nerves in stomach and normal investigations from gastroenterologist and internist: history

4.

30 year old G1P0 with pre-eclampsia. Counsel.

5.

40 year old male just passing through wants Tylenol #3 to hold him until next week. Manage.

6.

Back pain, radiating down leg: history and physical

5 + 5 minute stations
1.
A 2 day old infant with jaundice and a serum bilirubin of 220 mol/L (ref. Max 200 mol/L). Take a history from the mother.
Q: What are the possible causes for this abnormality? Give investigations and counsel.HIV patient with SOB: examine and CXR
2.

30 yo female with RLQ pain x1 wk. Perform a physical exam. Findings include distended abdomen and tenderness at McBurneys.
Q: The patient has a history of Crohns and presents with the following abdominal x-ray (shows small bowel obstruction), what is
the diagnosis?Mother wants to go on a camping trip with her 9yo son who suffers from primary enuresis. Hx, Counsel

3.

66 year old male with symptoms of claudication. Do a focused physical exam.


Q: What 2 investigations would be most appropriate. What 5 risk factors on history would point to the diagnosis.

4.

Young female with secondary amenorrhea for 6 months. Take a history.


Q: What investigations would you order. Give a differential diagnosis. What is the most likely diagnosis, what results would
confirm this diagnosis? Counsel with regards to OCP.

5.

Telephone in the room. Nurse in a peripheral center calls wishing to transfer a patient who has had a febrile seizure. Manage over
the phone. Counsel to get patient ready for transfer.

6.

35 yo male outpatient with SOB, cough, sputum. HIV+. Take a history and perform a physical exam. X-ray findings: PCP.
Q: Recommend further investigations and treatment.

October 1999
10 minute stations
1. Man in 20's fell off ladder 14 feet. Bruised over left anterior ribs. On table in c-collar. CVS, resp normal. Asking for pain relief. Manage.
2. Woman in 60's discharged 4 days ago from hospital with PE. Was on 5 mg of coumadin OD, discharged with INR of 2.0. INR today 1.0.
Counsel and manage.
3. 10 year old child with frequent URIs, in today with one. Physical exam and address parents concerns.
4. Female 63 years old 4 days post GI surgery with 15 minutes of SOB. Physical exam.
5. 16 year old female in office because her mother is concerned about her vomiting. History.
6. Immigrant woman who has 6 week old baby. Her friends have told her that he needs shots. Her mother has told her that the shots harm
the baby. Counsel.
7. 48 yo woman with multiple complaints. Negative investigations by several other doctors. Hx and mental status exam
Q: Without looking at the patient again, describe her appearance. What is your diagnosis?
5 minute stations (A patient encounter; B written station)
1. A. 26 year old male refused life insurance because of elevated LFTs. Focussed history.
B. Ddx, other tests would order.
2. Forty-two year old executive/lawyer? with heavy bleeding per vaginum. Focussed history.
B. Ddx, testing
3. Woman thirties with infertility, couple trying to get pregnant 18 months. Husband couldnt be there because of sudden business trip.
B. Ddx, testing
4. Young male, thrown 20 feet from bike by car, in ER. Neuro exam.
B: interpret and comment on c-spine film
5. HIV patient with increasing exercise intolerance and dyspnea, now dyspneic at rest. Physical exam.
B: interpret CXRay, Ddx, Rx
6. Visit mother who has given birth 6 hours earlier to a full term baby weighing less than 5 lbs. Focussed history.
B. Ddx. ?, ?
7. Construction worker with shoulder pain. Physical exam R shoulder.
B. interpret xray, Ddx,Rx

1998
5+5 minute stations
1. Take hx. From man with acute urinary retention.
Pep: most likely dx, ddx, investigations
2. Take hx. From mom of child with febrile seizure
pep: dx, tell mom risk of recurrence, tell mom how to manage a future seizure
3. Take hx. From man with dysphagia
pep: shown barium swallow, describe xray finding, give dx, give workup
4. Examine young man with acute hip pain
pep: ddx, key hx points, investigation
5. Examine young woman with purpura and epistaxis
pep: hx, dx, investigations
6. Examine 20 y.o female with hypertension
pep: ddx, investigations, what to do if your investigations are normal
7. Take history from mother of child with chronic cough unresponsive to abx
pep:ddx, key hx. Elements, investigations
8. Examine young woman with llq pain
pep: ddx, key hx. Elements, investigations
9. Take hx. From woman with menorrhagia
pep: ddx, key hx elements, investigations
10. Examine man with calf claudication
pep: interpret ekg, risk factors, investigations
11. Take history from man with sob and sputum
pep: er rx, investigations, advice re: prevention
10 minute stations
1. Counsel young woman with unwanted pregnancy (mostly re: abortion)
2. Head to toe neuro exam for a confused lady already seen by another md
3. Counsel young woman re: oral contraceptive pill
4. Manage an epigastric stabbing in er with a nurse
5. Woman with "abdominal blockage" seen and cleared by another md
q1-describe pt's appearance without looking,
q2-dx
6. Take hx from woman with sleep disturbance
q1- dx,

q2- management

1. Man with ankle sprain. Counsel re: rehab, care, followup


2. Manage acute mi in er with nurse
3. Take hx. From parent of child with language delay
q1-dx
4. Examine man with low back pain
q1- dx (disk prolapse)
5. Take history and counsel a man with impotence.

q2- what level

1997
1. First year university student, 9 weeks pregnant, considering abortion. Take a history and counsel. Findings: tearful, guilty, sleep
disturbance, has not engaged social supports.
History: combine a pregnancy history with a social history and a screen for depression.
Pregnancy: Patient ID (name, age, occupation). GTPAL (number of gestations, term pregnancies, premature births, abortions, live children),
history of problems, if any, with previous pregnancies. Current pregnancy, establish gestational age (GA) by last menstrual period (LMP) if
regular periods and sure dates (if unsure a dating ultrasound would be needed). The GA is the number of weeks from the first day of the
LMP. The EDC is first day of LMP + 7 days 3 months. Ask about use of alcohol, smoking, drugs, domestic violence (50% begins in
pregnancy), maternal illnesses during the pregnancy (particularly diabetes, rubella, toxoplasmosis, herpes, CMV, thyroid dysfunction, HTN,
hypercoagulation). Use of birth control, if any. Past medical history, family history of pregnancy related problems, medications.
Social: Status of any relationships at present including relationship with the childs father. Social supports (family, friends, boyfriend), do
they know? Are they helping? Employment/financial/educational status of the patient, does the patient feel prepared to raise a child?
Psychiatric: How does the patient feel about this decision? How is she coping? Cover mnemonic for major depression. MSIGECAPS:
mood (depressed), sleep (increased or decreasedif decreased, often early morning awakening), interest (decreased), guilt/worthlessness,
energy (decreased or fatigued), concentration/difficulty making decisions, appetite and/or weight increase or decrease, psychomotor activity
(increased or decreased), suicidal ideation positive diagnosis of major depression requires five of these over a 2 week period, one of the five
must be loss of interest or depressed mood. Symptoms do not meet criteria for mixed episode, significant social/occupational impairment,
exclude substance or GMC, not bereavement.
Counseling: Make empathetic statements, e.g. This must be very hard for you.
Health while pregnant: recommend abstinence from harmful agents (alcohol, smoking, drugs) while pregnant and use of medications only
after consulting with a physician, treatment for pregnancy-related illnesses as above, and healthy eating habits.
Social supports: Discuss the importance of engaging social supports, and consider a visit with both the patient and her partner or other
supporting person.
Abortion: Provide information on local abortion services. Make the patient aware that the gestational age limit after which many
practitioners will not perform an elective abortion in Canada is 20 weeks, but that this is a late limit and her decision should be made sooner,
before 16 weeks would be best. Inform the patient that further advice is available from private gynecologists who perform abortions and
counselors at elective abortion centers. Offer to refer the patient if she wishes.
Depression management: Normalize the patients depressed mood in view of her circumstances. If there is evidence of major clinical
depression, arrange close follow up to monitor for suicidal ideation, refer to psychiatry. Do not prescribe medications at this time (because of
the pregnancy).
2. 20 year old female wants an oral contraceptive. Take a history and counsel.
History: Name, age occupation/school level. Why does patient want an OCP? Has she been on it before or other forms of contraception? If
so, why was it stopped? How long has the patient been sexually active? How many partners? Current contraception used. Is there a
possibility that the patient could be pregnant? Obtain the date of last menstrual period.
Pregnancy history: GTPAL (number of gestations, term pregnancies, premature births, abortions, live children), history of problems, if any,
with previous pregnancies.
Gynecological history: Ask about sexually transmitted disease (STDs), PID, migraine, fibroids, diabetes, thromboembolic disease, heart
problems, cancer, liver disease. Date of last Pap smear, history of abnormal Pap smear and follow-up/treatment? When did the patient start
menstruating? Menstrual history: regularity and length of cycle and duration of periods, heaviness of flow (number of pads required),
cramping, associated discomfort/pain, bloating, mood swings (PMS). Medications, drugs, alcohol, smoking, past medical history (especially
breast cancer), family history, review of systems.

Counseling:
Contraindications to OCP: current pregnancy, undiagnosed vaginal bleeding, active cardiovascular/thromboembolic diseases (includes
coronary and carotid disease, symptomatic mitral valve prolapse, cerebrovascular disease, moderate-severe HTN, active DVT), proliferative
retinopathy, history of breast cancer or other estrogen dependent tumors (liver, breast, uterus), impaired liver function (obstructive jaundice in
pregnancy), congenital hyperlipidemia, age > 35 and smoking, Wilsons disease. Relative contraindications to OCP: smoker > 35 years old,
diabetes, migraines, fibroids.

Mechanism of action of OCP: standard preparations contains estrogen and progesterone or just progesterone, prevents ovulation by
interfering with feedback of hormone signaling, atrophic endometrium, change in cervical mucous (mucous plugthought to be due to
progesterone component).
Available preparations: 21 day vs. 28 day tablets (7 placebo days). Other preparations: Depo injections q3m (Depo-Provera
medroxyprogesterone, restoration of fertility may take up to 1-2 years, irregular menstrual bleeding), implants q5y (Norplant
levonorgestrel, six capsules inserted subdermally in arm, irregular menstrual bleeding). Longer term preparations offer lower cost over the
duration of action (but greater one-time cost) and greater convenience.
Benefits of all the hormonal contraceptives: ABCDEs: Anemia reduced, often clears Acne; Benign breast disease and cysts decreased;
Cancer (ovarian decreased), Cycles regulated, Increased Cervical mucous which reduces STDs; Dysmenorrhea decreased, decreases Ectopic
pregnancy rates and of course: virtually no chance of pregnancy when taken as directed (98-99.5%).
Risks of hormonal contraceptives: slight weight gain is usual (5 lbs), increases risk of DVT especially in combination with smoking, may
stimulate estrogen-receptor positive breast cancers, but does not appear to cause them, may have to try two or three different preparations to
arrive at the one for the patient. Also note that hormonal contraceptives do not provide as much protection against sexually transmitted
diseases, compared to barrier methods.
Directions: Start OCP on the first day of the next menstrual period. Place package in an obvious location to help you to remember. Take at
the same approximate time each day. Use additional contraception for the first two months, as OCP contraception is not reliable until then. If
you miss a day, take two pills the next. If you miss two days, take two pills for the next two days and use an alternative method until the next
period. Give prescription for OCP of choiceany family members (sisters/mother) on OCP? What works for them? Arrange follow up.
3. 16 year old boy with epilepsy documented by neurologist, comes to you because he does not want to see his parents family doctor.
Wants a drivers license. Take a history and counsel.
History of seizure disorder: Patient ID. Age of onset (primary generalized rarely begin < 3 or > 20 years old). Precipitants: Sleep
deprivation, drugs, EtOH, TV screen, strobe, emotional upset. Describe seizures (Jacksonian march? Salivation, cyanosis, tongue biting,
incontinence, automatisms, motor vs. visual/gustatory/olfactory), frequency, duration, what body parts affected and in what order (motor
frontal lobe, visual/olfactory/gustatory hallucinations = temporal lobe), promontory signs (presence of aura: implies focal attack), post-ictal
state (decrease in level of consciousness, headache, sensory phenomena, tongue soreness, limb pains, Todds paralysis - hemiplegia), degree
of control achieved with medications, at what dose and for how long, corroboration from family if possible. Was a CT scan done when
seizures were first diagnosed? Number and description of recent seizures, are they different from previous seizures? Is the patient having any
new symptoms such as headache, morning vomiting, new neurological deficits. If the drug worked in the past why does the patient believe it
isnt working now? Side effects of antiepileptics: drowsiness, poor concentration, poor performance in school, ataxias, peripheral
neuropathy, acne, nystagmus, dysarthria, hypertrichosis (excessive hairiness), gingival hypertrophy (phenytoin). Medications, drugs and
alcohol, smoking, allergies, past medical history, family history, review of systems.
Compliance: Is the patient taking meds? Why not? Problems at school or home? Ask about relationship problems. Depression screen as in
#19 above. Social supports.
Physical exam: neurologic exam including mini mental, cranial nerves, bulk, tone, power, sensation, cerebellar exam, deep tendon reflexes.
Treatment: Discuss importance of compliance with medication and avoiding dangerous activities such as driving until good control is
achieved. Ministry of Transportation regulations require 1 year seizure free before they will grant a drivers license in Canada. Inform the
MOT of the patients seizure disorder if you have not already done so and inform the patient that this is required by law.
If alcohol is an issue, inform the patient that chronic alcohol intake may decrease blood levels of antiepileptics (via increased liver
metabolism), and excess alcohol intake can precipitate seizures by lowering the seizure threshold thereby precipitating a seizure. It is
generally recommended that the patient not drink at all. Fatigue and concomitant illness can also lower seizure threshold. The patient should
consult a physician before taking other medications, as they may also lower the seizure threshold. The same is also true of sedatives, cocaine,
amphetamines and insulin. Fatigue and other illnesses can also lower seizure threshold, in addition to various other medications.
If patient is having stress management, anxiety issues, he may require further counseling. Outline a treatment plan consisting of: EEG, CT
head, metabolic screen, medications (if not done already), and follow up appointments. Get the parents involved if possible.
Send blood for serum Dilantin (phenytoin) levels if patient is on this already. If Dilantin levels are therapeutic, but the patient is having severe
side effects or poor seizure control, a second drug may be added (usually carbamazepine or valproic acid).

Discuss what to do in the event of seizure, counsel parents if possible. Bystanders are not to insert objects into the patients mouth. Turn
patient on his side while seizing. Call ambulance or take to Emergency if seizure doesnt stop in 5 minutes.
Arrange regular follow up to monitor progress and serum Dilantin levels.

Indications and important side-effects of major antiepileptic drugs


Drug
Carbamazepine
(Tegretol)

Indication
Partial or generalized
tonic-clonic seizures

Dose-related
Diplopia, dizziness, headache, nausea,
drowsiness, neutropenia, hyponatremia

Phenytoin
(Dilantin)

Partial or generalized
tonic-clonic seizures,
status epilepticus

Valproate
(Epival,
Dapakene)
Ethosuximide
(Zarontin)

All generalized
seizures or partial
seizures
Absence seizures

Nystagmus, ataxia, nausea, vomiting,


gingival hyperplasia, depression,
drowsiness, paradoxical increase in
seizures, megaloblastic anemia
Tremor, weight gain, dyspepsia,
nausea, vomiting, alopecia, peripheral
edema
Nausea, anorexia, vomiting, agitation,
drowsiness, headache, lethargy

Idiosyncratic
Morbilliform rash, agranulocytosis,
aplastic anemia, hepatotoxic effects,
Stevens-Johnson, teratogenicity
Acne, coarse facies, hirsutism, blood
dyscrasias, Lupus-like syndrome,
rash, Stevens-Johnson, Dupuytrens,
hepatotoxic effects, teratogenicity
Acute pancreatitis, hepatotoxic
effects, thrombocytopenia,
encephalopathy, teratogenicity
Rash, erythema multiforme, StevensJohnson, Lupus-like syndrome,
agranulocytosis, aplastic anemia

4. 48 year old woman complains of diseased stomach. Has had negative investigations by several other doctors. Take a history and
perform a mental status examination. Q: Without looking at the patient again, describe her appearance. What is your diagnosis?
History for depression: ID (name, age, employment status, marital status, living arrangements), chief complaint, HPI: MSIGECAPS mood
(depressed), anxiety, diurnal variation in mood and activity with nadir in the early morning, irritability, change in sleep pattern (increased or
decreased, if decreased usually morning awakening), interest (decreased), anhedonia (inability to enjoy previously enjoyable activities), guilt,
hopelessness, worthlessness, energy (fatigue), concentration, memory, difficulty making decisions, appetite and weight gain or loss,
psychomotor retardation/agitation, suicidal ideation. Also inquire about past psychiatric/medical/surgical history, allergies, meds, family
history, personal history, review of systems.
Suicidal ideation: does patient intend to harm self, reason for suicidal thoughts, current plan, lethality of plan, access to lethal means, has
patient given away prize possessions or written final notes to loved ones, previous attempts.
Diagnosis of major depression: Mnemonic for major depression MSIGECAPS: mood, sleep, interest, guilt, energy, concentration,
appetite, psychomotor, suicide, positive diagnosis of major depression requires five of these over a 2 week period. One of the five must be
either loss of interest or depressed mood. Cannot be a mixed episode and symptoms must cause social/occupational impairment. Cannot be
GMC or substance abuse or bereavement. (A diagnosis of depression cannot be made in the face of bereavement within the past two months
or drug or alcohol abuse.)
Dysthymia: A diagnosis of dysthymia requires depressed mood for most of the day, more days than not, for at least two years. Presence
while depressed of at least 2 of: poor appetite or overeating, insomnia or hypersomnia, low energy/fatigue, low self esteem, poor
concentration or difficulty making decisions, hopelessness. Never without depressed mood for more than 2 months at a time, no MDE,
manic, mixed or hypomanic episodes in past, not GMC or substance abuse.
Manic episode: Expansive, elevated or irritable mood x 1 week with 3 of following: GSTPAID grandiosity (or inflated self esteem), sleep
(less need for), talkative, pleasurable activities (with painful consequences), activity increased (goal directed or psychomotor), ideas (flight
of), distractibility. Not mixed episode. Severe enough to cause psychotic features/impaired social/occupational functioning. Not substance
abuse or GMC.
Differential for depression: Check for bipolar mood disorder (manic-depressive), schizophrenia, psychotic depression and obsessivecompulsive disorder. Ask about manic episodes, paranoia, hallucinations (esp. voices), obsessive thoughts, previous psychiatric problems,
family history of psychiatric disorders, substance abuse, relationship problems, problems at work (basically a mental status exam).
Medical causes of depression: ask about hypothyroidism, adrenal dysfunction, hypercalcemia, mononucleosis. Consider chronic fatigue
syndrome. Drug use, smoking, allergies, past medical history including psychiatric history and history of abuse. Family history, review of
systems.
Mental status: appearance, behavior (dress, grooming, posture, gait, apparent age, physical health, body habitus, expressions, attitude cooperative?, psychomotor activity, attention, eye contact), speech (rate, rhythm/fluency, volume, tone, quantity, spontaneity, articulation),
mood (subjective emotional state in patients own words), affect (Quality euthymic, depressed, elevated, anxious; Range full, restricted;
Stability fixed, labile; Appropriateness; Intensity - flat, blunted), suicidal ideation (low, intermediate, high poor correlation between

clinical impression of suicide risk and probability of attempt), thought process (coherent, flight of ideas, tangentiality, circumstantiality,
thought blocking, neologisms, clanging, perseveration, word salad, echolalia), thought content (delusions bizarre vs. non-bizarre,
obsessions, preoccupations, phobias, recurrent themes), perceptual disturbances (illusions, hallucinations, depersonalization, derealization),
insight, cognition, judgment.
Multiaxial Summary:
Axis I clinical disorders DSM IV; differential diagnosis
Axis II personality disorders DSM IV; mental retardation
Axis III GMC (as they pertain to Axis I and other Axes)
Axis IV psychosocial and environmental problems
Axis V global assessment of functioning (GAF) 0-100
Mini-Mental Status: orientation to time/place (5 pts, year, season, month, day, day of week; 5 pts, country, province, city, hospital, floor),
memory (3 pts, honesty, tulip, black; 3 pts, delayed recall), attention/concentration (5 pts, serial 7s, WORLD backwards), language tests:
comprehension (3 pts, three point command), reading (1 pt, close your eyes), writing (1 pt, complete sentence), repetition (1 pt, no ifs,
ands or buts), naming (2 pts, watch, pen), spatial ability (1 pt, intersecting pentagons)
Patients appearance: slovenly
Diagnosis: major depression
Treatment of major depression: pharmacotherapy, psychotherapy, family therapy. Start fluoxetine (prozac) 20 mg qAM, may increase to 40
mg qAM after 1 week. Takes 2-4 weeks to work. Explain side effects of sleep disturbance, anorgasmia, nausea (use SSRI summary below).
Admit: when patient is actively suicidal, i.e. plans are specific, patient has given away possessions, written final notes to loved ones. For
suicidal ideation without current intent, discharge only with written contract with the patient to inform you immediately if they feel likely to
make an attempt. Give number to call and arrange regular follow up before patient leaves the office.
SSRI antidepressant summary
Fluoxetine (Prozac) generally energizing
Fluvoxamine (Luvox) more sedating
Sertroline (Zoloft) used in the elderly
Paroxetine (Paxil) used in mixed anxiety/depress

Start at: (mg OD)


20
50
50
10-20

Therapeutic: (mg OD)


20-80
150-300
50-150
20-60

5. 60 year old woman with acute confusion. Perform a focused physical exam excluding mental status.
Neurologic exam: what follows is a practical, regionally organized neurologic exam which can be completed in less than eight minutes (with
practice). It begins with the patient sitting, then standing, then lying down. Note that every physical exam should include vitals, although in
this case the examiner will ask you to move on.
Patient sitting (shoes and socks removed):
GCS/MMSE only if patient poorly responsive. The examiner will remind you to omit the mini mental status exam.

Glasgow Coma Scale


Eye Opening
Spontaneous
To speech
To Pain
never

(E)

4
3
2
1

Verbal
Response
Oriented and converses
Confused conversation
Inappropriate words
Incomprehensible sounds
None

(V)

5
4
3
2
1

Best Motor
Response
Obeys commands
Localizes pain
Withdrawal to pain
Abnormal flexion (decorticate)
Abnormal extension (decerebrate)
Nil

(M)

6
5
4
3
2
1

Mini-Mental Status: orientation to time/place (5 pts, year, season, month, day, day of week; 5 pts, country, province, city, hospital, floor),
memory (3 pts, honesty, tulip, black; 3 pts, delayed recall), attention/concentration (5 pts, serial 7s, WORLD backwards), language tests:
comprehension (3 pts, three point command), reading (1 pt, close your eyes), writing (1 pt, complete sentence), repetition (1 pt, no ifs,
ands or buts), naming (2 pts, watch, pen), spatial ability (1 pt, intersecting pentagons)

Cranial nerves CN III, IV, VI: Extra-ocular movements (patient follows your finger or the handle of a reflex hammer in an H-pattern,
check for diplopia in the center and at the extremes of the visual fields. CN II: Visual fields by confrontation (one eye at a time) patient
holds own hand over one eye and counts fingers flashed in left and right fields simultaneously (upper and lower) or identifies the wiggling
finger as it enters each quadrant on confrontation if too confused to count. By acuity: Snellen card. Fundi check for papilledema using
ophthalmoscope. CN II, III, V1 & VII: Pupillary light reflex and accommodation, corneal reflex. CN V: Facial sensation to light touch in the
ophthalmic, maxillary and mandibular divisions of the trigeminal nerve. Trigeminal motor: clench teeth, lateral jaw movement. CN VII, XII:
Facial muscle power raise eyebrows, squint eyes closed, show teeth, protrude tongue, observe palatal movement on saying Ah, puff
cheeks, platysma. CN VII, IX, X: Gag reflex, observe symmetric movement of palate, swallowing, taste. CN VIII: Gross hearing rub
thumb and index lightly while approaching the patients ear, note when they begin to hear the sound. CN XI: Sternocleidomastoid power and
trapezius power.
Body Pronator drift arm straight out in front of patient with palms up, eyes closed. Look for curling of fingers from the ulnar side,
pronation and downward movement of the affected side. Hoffmans reflex with the patients relaxed hand in a palm-down position, squeeze
and flick the nail of the index or long fingers between your thumb and long finger. Thumb flexion indicates a positive test and denotes upper
motor neuron lesion (similar to Babinskis reflex). Cerebellar tests finger-nose rapid alternating movements (dysdiadocokinesis), heal-shin.
Body power deltoids, biceps, triceps, wrist extension and flexion, finger abduction and adduction, psoas (hip flexion lift each knee off the
bed against resistance), quadriceps, hamstrings, ankle dorsiflexion (test plantar flexion while standing). Body sensation light touch, pin
prick, and cold temperature (use a meta K-basin or turning fork) are tested on the distal upper limbs (forearms and hands) and lower limbs
(foreleg and feet). Vibration sensation is tested using a C-120 Hz tuning fork on the joint capsules of the most distal joints at which the
vibration can be felt. Body reflexes biceps jerk, brachioradialis, knee, ankle, Babinski.

Patient standing
Gait observe for wide-base, Parkinsonian festination/shuttle, consistent lateralizing falls. Balance tight-rope walk, stand with arms
crossed and feet in line, Rhomberg (feet together, eyes closed). Plantar flexion power hold patients hands for balance, ask patient to stand
on one foot, then raise themselves up on the toes.

Patient lying supine


Tone passive rapid alternating forearm rotation, passive rapid elbow flexion/extension with one thumb on the biceps tendon to feel for cogwheeling. Rapid lifting of the relaxed leg from behind the knee heel remains on the bed in normal tone.
6. A young man is brought to the Emergency department with an epigastric stab wound sustained in a bar fight. There is a nurse in
the room. Manage.
Unstable means either an acutely changing condition, vital signs dangerously beyond normal ranges (e.g. low BP), or any life-threatening
condition which is inadequately controlled. This patient is unstable due to stab wound with unknown consequences which is an acutely
changing and inadequately controlled problem and therefore requires stabilization or resuscitation.
The ACLS and ATLS format is useful: i.e. primary survey, orders, secondary survey, orders. Note that the simulated patient may have an
impressive simulated injury. Do not allow this to distract you from the primary survey.
Primary survey: (mnemonic ABCD)
Airway: check for patent airway (look in mouth), is airway threatened by blood? High chance of aspiration due to poor level of
consciousness, neck or face swelling. If airway is compromised, immediately place an oral airway or intubate.
Breathing: is patient breathing, check O2 sat, may require immediate manual bag-valve mask followed by intubation with positive pressure
ventilation.
Circulation: BP, HR, rhythm on monitor, active high volume bleeding. Patient may require immediate chest compressions, defibrillation or
cardiac drugs if there is an unstable rhythm (Ventricular fibrillation, unstable ventricular tachycardia, PEA, asystole, PST, or bradycardia).
Debilitation: refers to the Glasgow Coma Scale (GCS):

Glasgow Coma Scale


Eye Opening
Spontaneous
To speech
To Pain
never

(E)

4
3
2
1

Verbal
Response
Oriented and converses
Confused conversation
Inappropriate words
Incomprehensible sounds
None

(V)

5
4
3
2
1

Best Motor
Response
Obeys commands
Localizes pain
Withdrawal to pain
Abnormal flexion (decorticate)
Abnormal extension (decerebrate)
Nil

(M)

6
5
4
3
2
1

Note: Standard painful stimulus is rubbing the knuckle on the sternum. For withdrawal, apply pressure on the base of the nail bed with a pen.
Decorticate posture is arm flexion with leg extension on the same side of the body, may be unilateral or bilateral. Indicates a lesion above the
brainstem. Decerebrate posture is arm and ipsilateral leg extension, may be unilateral or bilateral, indicates brainstem involvement. A GCS
of 8 or less is considered an indication for intubation because of the risk of poor protection of the airway from aspiration.
Primary orders: oxygen, monitoring (ECG, O2 sat, automatic BP cuff or arterial line), IV access: need two large-bore (16 gauge, 14 if
possible femoral vein cortice with 2 lumens), run wide open with normal saline for acutely low BP, may need to be more restrained if
pulmonary edema is a problem. Coma cocktail if diagnosis not known already: thiamine 100 mg IV, narcan 1 mg IV, flumazenil 0.1 mg IV (1
amp D50W is no longer included in this cocktail because of deleterious effects of high serum glucose on the injured brain). Initial
investigations: CBC, lytes, urea, creatinine, ABG, glucose, ionized Ca, CK-MB, troponin, INR/PTT, ECG, portable CXR, cross-table lateral
C-spine and hard collar if there is head injury or any significant trauma.
Secondary survey: Head to toe physical exam. Vitals. Head and neck: inspect for lacerations and contusions, pupillary response, dolls eyes
(careful of neck, may not be able to turn head enough to do this), corneal reflexes, palpate facial bones for stability, look in nose and ears for
blood or CSF leaks, hemotympanum. Check oral cavity, gag reflex, palpate dorsal cervical spines for pain and alignment, is the trachea
midline? Chest: breath sounds, heart sounds, radial pulses bilaterally. Abdomen: rigidity is an indicator for immediate general surgery,
auscultate for bowel sounds, palpate liver and spleen. Log roll patient onto back, inspect. Rectal. Palpate for pelvic stability and intactness
of long bones.
Secondary orders: foley, NG tube if patient may go to surgery or require charcoal. Specific interventions based on findings. Further X-rays,
CT head if cause of decreased level of consciousness unclear or if there may have been a seizure.
Clearing C-spines: The principle of clearing C-spines is to rule out both bony fractures and ligamentous injury, either of which can make the
spine dangerously unstable. Most emergency physicians will clear the cervical spine in the case of an alert patient who has no pain on
palpation of the dorsal spinous processes and a normal cross-table lateral C-spine X-ray. If the patient has neck pain, flexion/extension plane
films are done. These involve gently flexing, then extending the neck and taking views at each extreme. The patient must be alert enough to
warn the examiner of paresthesias in the hands or increased neck pain on movement during this procedure, which may indicate compromise
of the neural elements. Flexion/extension views may be done under fluoroscopy if the patient is not alert.
History: if available: How did the patient obtain the wound, was there mechanism of action for other injury, i.e. collapse/fall, preceding and
subsequent events, did patient lose consciousness, duration of unconsciousness, did the patient fall, injuries during fall, medications and
drugs, smoking, allergies, past medical history, family history, review of systems.
Management: For unstable blood pressure, blood loss is the most likely cause (CBC may be normal with a large acute blood loss), type and
cross for 4-6 units depending on estimated severity and hang blood as soon as possible. Is the patients abdominal wound the first priority?
Examine for head injury, other injuries. Explore wound with a gloved finger on secondary survey, if wound is more than superficial, consult
general surgery and prepare patient for immediate surgical exploration in the OR. Obtain details of the history from family or witnesses if
possible.
7. 30 year old woman with six weeks of epistaxis, petechiae and easy bruising. Perform a focused physical exam. Findings: petechiae,
bruises. Q: The patient has a normal CBC except for platelets 20 (normal 130-400). What is the most likely diagnosis? What four
areas on history would help to confirm this diagnosis? What four investigations would you order?

Hematologic exam: patient disrobed to underwear, draped below the waist.

Patient sitting
Inspect the patient generally for petechiae, abnormal skin tone, hair falling out. Inspect the finger and toe nails for dystrophy, flame
hemorrhages, leukonychia, inspected the palm for erythema and Dupuytrens contracture. Look in the nose and mouth for bleeding,
petechiae, masses. Palpate the anterior and posterior triangles of the neck, the supra and infra clavicular areas, and the axillae for lymph
nodes. Palpate the thyroid while standing behind the patient, ask her to swallow. Chest from behind the patient, inspect the skin. Percuss
the lung fields for effusions and consolidations, auscultate the lung fields. Percuss and auscultate the anterior lung fields. Listen over the
aortic (right upper sternal border 2nd ICS systolic = stenosis, diastolic = regurgitation, continuous = BT shunt, right or left), pulmonary (left
upper sternal border 2nd ICS systolic = ASD, pulmonary flow, continuous = ductus, LBT shunt), tricuspid (left lower sternal border 5 th ICS
systolic = regurgitation ... Stills murmur, diastolic = stenosis) and mitral (apex 5 th ICS and midclavicular line systolic = prolapse,
regurgitation; diastolic = stenosis) areas.

Patient lying supine


Compress the sternum and ribcage for pain (seen in multiple myeloma). Inspect the abdomen. Auscultate for bowel sounds. Palpate for
enlargement of the spleen and liver. Percuss the liver. Palpate the groin for lymph nodes. Note: avoid rectal exam as this trauma may cause
bleeding.
Most likely diagnosis: idiopathic thrombocytopenic purpura (ITP), also called immunologic thrombocytopenic purpura or Wedhofs disease.
Four findings on history which would help to confirm the diagnosis: 1. Remitting-relapsing course, 2. Mild fevers, 3. Splenic discomfort
due to mild enlargement, 4. Bleeding after low doses of NSAID.
Four investigations: Blood smear, INR/PTT (for hemophilia), serum urea/creatinine (for hemolytic-uremic syndrome), serum plateletassociated IgG (for ITP).
8. 62 year old man presents to the Emergency Department with 12 hours suprapubic discomfort and inability to urinate.
Catheterization yields 1200 cc urine. Take a history. Q: What is the most likely cause of this mans problem? Give three other
possible diagnoses. What four investigations would you order?
History: name, age, occupation. History of suprapubic pain and inability to urinate. History of pain on urination, frank blood in the urine,
color of urine, difficulty initiating or maintaining urinary stream, fever, renal pain, groin pain. Previous renal colic/ diagnosed prostate
hypertrophy, prostate cancer, prostatism, nephrolithiasis, UTIs? Malignant symptoms: night sweats, weight loss, fatigue. Medications,
drugs/alcohol, smoking, past medical history, past surgical history, history of pelvic radiation, TURP, family history, review of systems.
Most likely diagnosis: benign prostatic hyperplasia.
Other possible diagnoses: UTI, prostatitis, prostate cancer.
Four investigations: urea/creatinine, urinalysis, prostate specific antigen (PSA), renal ultrasound.
Treatment: watchful waiting (50% resolve spontaneously), medical (alpha-adrenergic antagonists Terazosin, doxazosin, tamsulosin; 5alpha-reductase inhibitors finasteride), surgery (TURP vs. open prostatectomy), minimally invasive (stents, microwave therapy, laser
ablation, cryotherapy, HIFU, TUNA).
9. 6 month old child who just had a seizure. Take a history from the mother in the Emergency Department. Findings: short seizure
with T 39.5 C. Never had seizures in the past. Developmentally normal. Q: What is the most likely diagnosis? What would you tell
the mother about any possible recurrence? What advice do you give if the child has another seizure?
History: Name, age. Describe seizure duration, what body parts affected and in what order, premonitory signs, post-ictal state (decrease in
level of consciousness, headache, sensory phenomena). Previous seizure? Ask about preceding trauma or illness or medications taken, the
childs temperature at the time of the seizure. Meningitis signs, neurological. History of problems during the pregnancy and birth.
Developmental history. Childs medical history, surgical history, medications, allergies.

Selected Developmental Milestones


Speech
Gross motor

Fine motor
Social

6 months
12 months
24 months
2-3 years
6 months
9 months
12 months
15 months
12 months
24 months
6 months
9 months
2 years
5 years

initiates sounds, eye contact


2 words beyond mama and dada
2-3 word phrases
short sentences
roll over
stand
cruise
walk
pincer grasp
turns pages in a book
stranger anxiety
separation anxiety
says no
prints name

Most likely diagnosis: febrile seizure (febrile seizures usually 6 months to 6 years, associated with initial rapid rise in temperature, no
neurologic abnormalities/evidence of CNS infection/inflammation before or after, no history of non-febrile seizures, most common
generalized tonic-clonic, < 15 minutes duration, no recurrence in 24 hours, atypical may show focal origin/> 15 minutes/> 1/24
hours/transient neurologic defect).
Prognosis: after a single febrile seizure 65% will never have another seizure. 30% will have further febrile seizures, 3% will go on to have
seizures without fever and 2% will develop lifelong epilepsy.
Management: find source of fever, Tylenol (antipyretics), LP to rule out meningitis if signs of meningitis, counseling and reassurance to
patient and parents if febrile seizures.
Treatment of recurrence: control fever with antipyretics (Tylenol), tepid bath, fluids for comfort only and use Ativan (lorazepam) 1 mg
SL/PO (or diazepam 5-10 mg PR) if a seizure occurs at home. Turn patient onto his/her side, do not force objects or fingers into mouth.
Bring to ER if seizure does not stop within fifteen minutes. Seizures do not cause mental impairment unless they are prolonged (> 30 min),
although seizures can be a symptom of brain damage. Patient should be investigated with CT head and EEG. Prophylactic anticonvulsant
therapy is a consideration with repeated seizures.
10. 68 year old man with difficulty swallowing. Take a history. Findings: throws up after eating. Can swallow liquids only. Weight
loss and fatigue. Smoker. Q: X-ray of barium swallow showing narrowing of contrast at T5-6: describe the abnormality. What is
the likely diagnosis? What investigation would confirm the diagnosis? What further investigations would you order?
History: (Dysphagia = difficulty swallowing) onset, chronology, description of problem, aggravating and relieving factors. Is the difficulty
transferring food from mouth to esophagus suggesting oropharyngeal dysphagia with food getting stuck immediately after swallowing often
with nasal regurgitation? With esophageal dysphagia food seems to be stuck further down. Is the problem worse with solids (suggests
mechanical obstruction), or liquids (suggests neuromuscular dysfunction, often cant swallow either solids or liquids). Is there a sensation of
a lump in the throat? (globus hystericus = transitory sensation of a lump in the throat related to anxiety). Progression from solid swallowing
difficulty to difficulty swallowing both solids and liquids suggests progressive obstruction such as from a worsening stricture or growing
tumor. The combination of intermittent obstruction and chest pain suggests esophageal spasm. Ask about peptic ulcer, reflux, hiatus hernia,
weight loss, night sweats, fatigue, hematemesis, black stools, pain. Medications, drugs/alcohol, allergies, smoking, past medical history,
family history, review of systems.
Oropharyngeal: Neurological cortical pseudobulbar palsy (UMN lesion) due to bilateral stroke; bulbar ischemia (stroke);
syringobulbia; tumor (LMN); peripheral polio; ALS. Muscular MD; cricopharyngeal incoordination (failure of UES to relax with
swallowing), sometimes seen with GERD; Zenkers diverticulum.
Esophageal: solid food only mechanical obstruction intermittent = lower esophageal ring/web; progressive heartburn = peptic ulcer;
age > 50 = carcinoma. Solid or liquid food neuromuscular disorder intermittent = diffuse esophageal spasm; progressive reflux =
scleroderma; respiratory symptoms = achalasia.
Differential diagnosis: Mechanical obstruction: tumor, stricture (secondary to GERD, trauma etc.). Neuromuscular obstruction: achalasia
(cardiac sphincter does not relax), cranial nerve palsy, MS, supranuclear palsy, stroke, motor neuron disease, myasthenia gravis, muscular
dystrophy.

Description of Barium swallow findings: string sign, graded narrowing of intra-esophageal diameter extending from T5 to T8 level.
Most likely diagnosis: esophageal cancer.
Investigation to confirm diagnosis: endoscopy with biopsy.
Further investigations: CT chest (for mediastinal and lymph node involvement), chest X-ray, liver function tests, abdominal ultrasound (for
mets).
11. 23 year old with BP 160/100 in both arms. Perform a focused physical exam. Q: Give four possible diagnoses. What four
investigations would you order? If these investigations were negative, give 5 steps in your initial management plan.
Physical exam for hypertension: combines exams for atherosclerosis, coarctation, hyperthyroidism and Cushings. Patient should be
disrobed to underwear and draped below the waist.

Patient Sitting
Take vitals (need BP in all four limbs legs to be done when patient is lying down). Inspect for cyanosis, arcus senilis in the eyes (sign of
high cholesterol), bulging veins in the upper chest (SVC syndrome), supraclavicular fat pad, buffalo hump, moon face, truncal obesity, striae,
nicotine stains on fingers, clubbing, flame hemorrhages on nails, obesity, high work of breathing, intercostal indrawing, symmetric chest
movement, visible apex beat.
Fundoscopy for retinopathy of hypertension: (in order of increasing severity of damage) constriction and sclerosis of retinal arterioles,
hemorrhages, exudates, papilledema.
Thyroid exam: Inspect patient for proptosis, thyroid stare (upper lids do not overlap the irises). Have patient follow your finger up and
down to check for lid lag and globe lag. Is skin thin dry and flaky or diaphoretic. Palpate thyroid standing behind patient, ask patient to
swallow. Inspect nails for leukonychia and hands for tremor (can place a piece of paper on the hand held horizontal to detect fine tremor) and
clubbing/thickening of distal phalanges. Check biceps reflexes with thumb held over the tendon, feel for slow return phase reflex of
hypothyroidism.
Palpate the apex, note whether it is laterally displaced (lateral to the mid-clavicular line) and feel for thrill or heave, feel radial pulses in the
arms simultaneously, note any delay.
Percuss the lung fields anteriorly and posteriorly.
Auscultate the lung fields anteriorly and posteriorly, listen over the aortic (right upper sternal border 2 nd interspace systolic = stenosis,
diastolic = regurgitation, continuous = BT shunt, right or left), pulmonary (left upper sternal border 2 nd interspace systolic = ASD,
pulmonary flow, continuous = ductus, LBT shunt), tricuspid (left lower sternal border 5th interspace systolic = regurgitation ... Stills
murmur, diastolic = stenosis) and mitral (apex at 5th interspace and midclavicular line systolic = prolapse, regurgitation; diastolic =
stenosis) areas, as well as over the right clavicle, and both carotids. Listen for rub. To bring out an aortic murmur (typically aortic
regurgitation), and coarctation bruits, ask patient to lean forward, exhale and stop breathing while you listen over the aortic and pulmonic
areas.

Patient Lying Supine


Auscultate for bruits over the renals on the abdomen. Observe for pulsations due to abdominal aortic aneurysm, palpate abdomen for
hepatomegaly. Palpate femoral pulses, and auscultate for femoral bruits, palpate the popliteal pulses, inspect the legs and feet for venous
stasis or arterial insufficiency ulcers, palpate the dorsalis pedis and tibialis posterior pedal pulses. Feel the ankles for pitting edema.
Tibial BP: BP cuff placed around calf, auscultate the tibialis posterior pulse posterior to medial malleolus on right and left.
JVP: Raise the head of the bed 30 degrees and inspect the neck. A jugular venous pulsation higher than 4-5 cm above the sternal notch
(approximately the level of the right atrium) is abnormal. If no JVP is visible then you may have to raise or lower the bed for high or low
JVPs respectively. Check the hepatojugular reflux (supine, mouth open, breathing normally compress the liver, the jugular venous pulse
should either not rise or remain elevated only transiently, a sustained elevation > 1 cm in the JVP is pathological). This is used to assess high
jugular venous pressure and RV function.

Classification of BP
DBP: < 90 (normal), 90-104 (mild HTN), 105-114 (moderate HTN), > 115 (severe HTN)
SBP: < 140 (normal), 140-159 (borderline isolated systolic HTN), > 160 (isolated systolic HTN)
Four possible diagnoses: essential hypertension, renal HTN (renal artery insufficiency, renal parenchymal disease), endocrine (thyroid
hormone, OCP, primary hyperaldosteronism, hyperparathyroidism, pheochromocytoma, Cushings) coarctation of the aorta, others
(enzymatic defects, neurological disorders, drug-induced (prolonged corticosteroid use), hypercalcemia, watch for labile white-coat HTN).
Investigations: Repeat BP 3x over next 6 months, urinalysis, CBC, Cr, lytes, fasting serum glucose, cholesterol panel, 12 lead ECG, TSH,
renal U/S.
Initial Management: 1. Smoking cessation. 2. Alcohol restriction to low risk drinking guidelines. 3. Salt restriction (max. 90-130 mmol 37 g per day). 4. Saturated fat intake reduction. 5. Weight reduction if BMI > 25 (at least 4.5 kg). 6. Regular aerobic exercise (50-60 min, 3-4x
per week).
12. 21 year old female with bloody diarrhea. Take a history. Findings: Abdominal cramping. Six watery stools in the past four hours
containing maroon colored blood. Feels dizzy and weak. No previous history of diarrhea previously well. Q: What two findings on
history indicate the seriousness of the problem? Give three possible diagnoses. Give four investigations appropriate to this situation.
History: name, age, occupation. Onset, duration, frequency, of diarrhea. Appearance of stools: how well formed, is blood on (anal/rectal
laceration) or admixed with stools, is blood bright red (lower tract bleed) or dark brown-black (upper tract bleed, e.g. stomach). Pain with
bowel movements, abdominal pain or cramps with location, radiation, precipitating factors and alleviating factors, quality, severity, timing
with respect to defecation, gas bloating. Heart burn, peptic ulcer, reflux, hiatus hernia. Extra-intestinal manifestations of inflammatory bowel
disease: ask about iritis, arthritis, mouth ulcers, anal ulcers, skin lesions, kidney stones. Infectious diarrhea: inquire about fever, nausea,
vomiting, weight loss, fatigue. Recent travel, consumption of unusual foods or foods which may have been contaminated. Recent exposure
to antibiotics. Family members sick at home. Pelvic pain, vaginal discharge, vaginal bleeding. Past medical history, medications (especially
NSAIDs, laxatives, antibiotics), family history of Crohns, ulcerative colitis, familial polyposis, review of systems.
Two findings which indicate the seriousness of the problem: patient feels dizzy and weak.
Three possible diagnoses: gastroenteritis, bleeding peptic ulcer, inflammatory bowel disease.
Four investigations: CBC with differential, stool for ova & parasites with culture & sensitivities, Clostridium difficile toxin. Endoscopy
(but above first). Type and cross for 4 units PRBCs.

1996
13. Middle aged woman with systolic ejection murmur radiating into the carotids. Perform physical exam.
The physical exam for a patient with a heart murmur is a cardiopulmonary exam.
Patient in sitting position: take vitals
Inspect for surgical scars, trauma, bony abnormalities, cyanosis, arcus senilis in the eyes (sign of high cholesterol), bulging veins in the upper
chest (SVC syndrome), nicotine stains on fingers, clubbing, flame hemorrhage on nails, obesity, work of breathing, intercostal indrawing,
symmetric chest movement, visible apex beat.
Palpate the apex, note whether it is laterally displaced (lateral to the mid-clavicular line) and feel for thrill or heave, feel radial pluses
bilaterally.
Percuss the lung fields anteriorly and posteriorly.
Auscultate the lung fields anteriorly and posteriorly, always evaluate heart sounds before murmur. First listen for S 1 and S2, then look for S3
and S4 and any other unusual heart sounds. Listen over the aortic (right upper sternal border 2nd ICS systolic = stenosis, diastolic =
regurgitation, continuous = BT shunt, right or left), pulmonary (left upper sternal border 2nd ICS systolic = ASD, pulmonary flow,
continuous = ductus, LBT shunt), tricuspid (left lower sternal border 5th ICS systolic = regurgitation ... Stills murmur, diastolic = stenosis)
and mitral (apex 5th ICS and midclavicular line systolic = prolapse, regurgitation; diastolic = stenosis) areas as well as over the right
clavicle, and both carotids. Listen for rub. To bring out an aortic murmur (typically aortic regurgitation), ask patient to lean forward, exhale
and stop breathing while you listen over the aortic area. To bring out a mitral murmur, ask patient to lie supine and roll partly onto the left
side while you listen over the apex. In general, murmurs are accentuated by increasing the dynamicity of the heart with mild exercise, such
as asking the patient to walk up a flight of stairs.
Murmurs are described in terms of where they are heard loudest, where the sound radiates, whether it occurs in systole or diastole, the pitch
(e.g. high, low), quality (e.g. harsh, blowing, musical), contour (e.g. crescendo, decrescendo or plateau) and its loudness graded out of six
(e.g. II/VI). The murmur of aortic stenosis is loudest over the aortic area, radiates to the clavicle or carotids, occurs in systole, has medium or
high pitch, is harsh and crescendo-decrescendo. A mitral regurgitation murmur by contrast, is loudest over the apex, also occurs in systole,
radiates to the axilla, is medium to high in pitch, blowing and plateau.
Innocent murmurs are <3/6 in intensity, peak early in systole, stop long before S2, are heard best at the base of the heart (aortic and
pulmonary areas), are not associated with clicks or heaves, and ECG and CXR are normal.
Patient lying supine
Auscultate for bruits over the renals on the abdomen. Observe for pulsations due to abdominal aortic aneurysm, palpate abdomen, femoral
pulses, and auscultate for femoral bruits, palpate the popliteal pulses, inspect the legs and feet for venous stasis or arterial insufficiency
ulcers, palpate the dorsalis pedis and tibialis posterior pedal pulses. Feel the ankles for pitting edema.
JVP: Raise the head of the bed 30 degrees and inspect the neck. A jugular venous pulsation higher than 4-5 cm above the sternal notch
(approximately the level of the right atrium) is abnormal. If no JVP is visible then you may have to raise or lower the bed for high or low
JVPs respectively. Check the hepatojugular reflux (supine, mouth open, breathing normally compress the liver, the jugular venous pulse
should either not rise or remain elevated only transiently, a sustained elevation > 1 cm in the JVP is pathological). This is used to assess high
jugular venous pressure and RV function.
14. 50 year old man with left-sided chest pain. Manage (means history, physical, investigations and treatment). Findings: bruise on
chest wall, normal CXR and ECG.
Differential Diagnosis for Chest Pain
Cardiac
Pulmonary

Non-Cardiac
GI

MSK/Neuro

Psychologic

Angina
MI
Pericarditis
Myocarditis
Dissecting aorta

Pneumonia w/ pleuritis
Pneumothorax
PE
Pulmonary hypertension

Esophageal reflux
Ulcer

Arthritis
Chondritis
Rib fractures
Herpes Zoster

Anxiety
Panic

History for chest pain: describe the pain, location, radiation, quality, time of onset, duration, intensity, circumstances under which it occurs,
aggravating and relieving factors, associated symptoms such as nausea, shortness of breath, dizziness, diaphoresis, dependent edema. Leg
pain. Respiratory symptoms: cough, sputum, fever, hemoptysis. GI symptoms: heartburn, dysphagia. Previous episodes, chronology of
these. History of trauma, asthma, bronchitis, COPD, pneumothorax, recent viral illness and previous chicken pox (Herpes Zoster can cause
chest pain), gastritis, peptic ulcer, reflux. Risk factors for heart and lung disease: smoking, hypertension, hyperlipidemia. Past medical
history, especially diabetes, heart disease including pericarditis, lung disease, GI problems, surgical history, and family history. Medications,
drug use, smoking, allergies, review of systems.
Physical exam: Cardiopulmonary exam as in question #13.
Investigations: CXR, ECG.
Treatment: given a normal CXR and ECG with a chest wall bruise as evidence of trauma send patient home, recommend non-prescription
pain medication (Tylenol and/or ibuprofen) and advise that the pain should subside gradually. Since the patient is at risk because of his age
group and male gender, explain the symptoms of myocardial infarct (MI) and advise to return immediately if these occur.
15. Young man with recent onset back pain and limp. Take history and physical.
A differential for low back pain is:
1. Degenerative (90% of all back pain)
Mechanical (degenerative, facet joint pain, muscle strain/spasm)
Spinal stenosis (congenital, osteophyte, central disc)
Peripheral nerve compression (disc herniation or rupture)
2. Cauda Equina syndrome
3. Neoplastic: primary or metastatic
4. Trauma: fracture (compression, distraction, translation, rotation)
5. Spondyloarthropathies: e.g. ankylosing spondylitis
6. Discitis/osteomyelitis
7. Referred: aorta (abdominal aortic aneurysm), renal (pyelonephritis), ureter (nephrolithiasis), pancreas (pancreatitis)
8. Malingering
Because discogenic and stenotic radiculopathy which have not improved over at least 4 weeks may be treatable surgically, the priority of a
history and physical for back pain is to differentiate radiculopathy from other causes and to identify the nerve root.
The most common disk herniation is a posterolateral L4-5, which compresses the L5 root. The herniation will also compress the L4 root if the
herniation is far lateral and the S1 root if it is more medial (central). The second most common herniation is a posterolateral L5-S1, which
compresses the S1 root. In the thoracic and lumbar spine, the nerve roots exit below the pedicles of the vertebra of the same number, while in
the neck the nerve root exits above the pedicle of the vertebra of the same number. L5 compression produces radiation from buttock to lateral
calf, lateral calf pain, numbness of the medial dorsum of foot (including web of great toe), and ankle dorsiflexion weakness, S1 compression
produces radiation posteriorly down leg to heel, posterior calf pain, lateral foot numbness and ankle plantar flexion weakness (with decreased
ankle jerk).
History: Red flags (BACKPAIN) B: bowel or bladder dysfunction; A: anesthesia (saddle); C: constitutional symptoms/malignancy; K:
chronic disease; P: paresthesias; A: age > 50; I: IV drug user; N: neuromotor deficits.
Describe the pain, location, radiation (L5 radiculopathy causes radiation from buttock to lateral calf, S1 radiates posteriorly down leg to heel),
quality, duration, frequency, intensity, circumstances under which it occurs, aggravating and relieving factors. Onset and chronology,
previous episodes. Previous investigations, treatment.

Pain worse lying down and bilateral leg weakness suggests spinal stenosis or ankylosing spondylitis. Spinal stenosis is characterized by
worsening of symptoms with standing and walking, with relief on bending and setting (a typical history of leaning on and bending over the
shopping cart for relief of pain while shopping is suggestive of spinal stenosis). Ankylosing spondylitis is characterized by morning stiffness
relieved by activity. Pain worse in back than in buttock or leg suggests mechanical back pain. Pain worse in buttock or leg than in back
suggests radiculopathy. Predominating symptoms of stiffness are suggestive of ankylosing spondylitis. Back pain is recurring and tends to
be nocturnal. Morning stiffness improves over the day. May be associated with weight loss, fever, fatigue, anemia. Focus on joint symptoms
(typically large joints), uveitis (occurs in one third of cases), and family history.
Has the patient had a fever, weight loss, night sweats (signs of cancer), urinary tract infection (sign of urinary retention), joint pain, uveitis
(inflammation of the uveal tract: iris, ciliary body, and choroids sign of ankylosing spondylitis)? Ask about effect on activities of daily
living, functional limitations. Associated numbness, weakness. Are the symptoms improving or worsening? What are the patients
conclusions about the pain and expectations of the physician? Medications, drugs and alcohol, smoking, past medical history, family history,
review of systems.
Cauda equina syndrome: Inquiry into bowel, bladder, and sexual function to reveal this rare syndrome is obligatory and a source of frequent
false alarms. Because these functions may not recover once lost, cauda equina syndrome due to a surgically treatable lesion is a surgical
priority if the time course is subacute and an emergency if the loss of function is acute. The syndrome consists of saddle anesthesia (perineal
numbness), lax anus, impotence, urinary retention and bowel incontinence. Note that this combination of signs is due to preservation of
sympathetic tone with loss of parasympathetic tone. Sympathetic tone is preserved because it is carried extra-spinally, while parasympathetic
signals are carried via the inferior spine and nerve roots. Note that bowel contraction and penile erection are parasympathetically driven.
Physical exam:
Standing
Assess gait, posture, range of motion including rotation, lateral and forward flexion, extension (pain worse on forward flexion and relief on
extension suggest discogenic pain, pain worse on extension suggests facet joint pain). For ankylosing spondylitis: Wright-Schober test
positive when distance between the lumbrosacral junction and a point 10 cm above (identified by palpation on the erect spine), distract by
less than 5 cm on full forward flexion of the spine. Modified Schober (i.e. detection of decreased forward flexion of lumbar spine). Lateral
flexion is impaired when the hand moves downward by less than 3 cm on the ipsilateral thigh. Look for scoliosis on standing (shoulder
heights equal?) and forward flexion; check for rib hump. Inspect back for spina bifida. Palpate for tender areas especially sacroiliac joints,
compress pelvis to elicit pain of sacroiliitis (hallmark of ankylosing spondylitis). Muscle tone, percuss costovertebral angles for renal pain.
Have patient walk on toes, heels. Ask patient to stand on one foot at a time and push up into tiptoe for ankle plantar flexor strength (S1).
Sitting
Knee jerks (L4) with quadriceps exposed, watch contraction. Ankle jerks (S1), rapidly dorsiflex each foot to test for clonus. Babinski.
Compare calf girths for wasting by measuring calf circumference 10 cm below tibial tuberosity. Test power of quadriceps, hamstrings, psoas
(raise knee up against resistance), ankle dorsiflexors. Ask patient to straighten both legs and compare this position to the degree of forward
flexion the patient was able to achieve on standing range of motion. Suspicion of malingering is raised if the patient claims to be unable to
bend from a standing position but is able to extend the knees from a sitting position.
Supine
Feel for lymph nodes at neck, clavicle, axillae, groin. Test hip extensors (patient presses leg into bed while you try to raise it). Sensation in
both legs: light touch, pin prick compare medial dorsum of foot (L5) with lateral foot (S1) and lateral calf (L5) with posterior calf (S1).
Vibration and position sense in big toes. Straight leg raise: raise patients heel on bed as far as patient will allow, note angle, note whether
this reproduced the patients ipsilateral or contralateral radicular pain. Bowstring test: flex hip to 90 degrees, extend knee to the point of pain
and press on the hamstring tendon, which is medial, note reproduction of pain. Peripheral vascular exam: inspect for venous stasis or arterial
insufficiency ulcers, check femoral pulses and auscultate for femoral bruits, feel popliteal, dorsalis pedis and tibialis posterior pulses.
16. 25 year old man wishes to refill a prescription of Fiorinal for tension headache. Manage.

History: description of headache pain, location (one-sided vs. bilateral/occiput-vertex Do you feel pain on one or both sides? If one-sided,
is it always the same side? If present on both sides, did the pain start on one side? Is it usually maximal on one side?), quality (pulsatile vs.
non-throbbing What kind of pain is it tightening, pressing, throbbing, pounding, pulsating, burning, etc? Do different types of pain occur
at different times in any one attack? If so, what types?), intensity, duration (at least 72 hours in migraine, if not treated), onset including time
of day (morning headache associated with raised intracranial pressure), previous episodes, aggravating/relieving factors (e.g. coughing and
straining worsen headache in raised ICP and chocolate or cheese can trigger migraines), associated symptoms (aura, nausea, vomiting,
photophobia light, phonophobia sound, osmophobia odors, nuchal rigidity, weakness, numbness, visual disturbances), medical history,
medication history (when was this prescribed, do you have the empty bottle, has it been prescribed before?), current meds, allergies, family
history, substance abuse inquiry, smoking, allergies, mood, stress, anxiety inquiry. Review of systems.
Red flags for headaches: must rule out headaches resulting from meningitis, trauma (subarachnoid hemorrhage, epidural hemorrhage),
tumor, temporal arteritis. History: new onset, headache worse at night, headache wakes patient at night, fever, neck stiffness, seizures,
trauma, changes in LOC/behavior, vomiting, severe, very young/old patients. Physical exam: fundi abnormal, Kernig/Brudzinski signs
(meningitis), focal neurological findings.
Given benign history with no suspicion of raised ICP or focal deficits and a description of headache consistent with the common tension
headache, a full neurological examination is not indicated. Suggest to the examiner that you would perform a brief neurological screening
exam. You will be told to move on.
Treatment: Explain that Fiorinal is a combination preparation of barbiturate (butalbital), caffeine and ASA which is properly used only for
the relief of occasional tension headaches. It is habit-forming, can precipitate a withdrawal syndrome including agitation, delirium and
seizures and has additive sedative effects with other CNS depressants. The fact that this patient has consumed an entire prescription in four
days suggests overuse due to dependence. He may also have analgesic headache syndrome in which inappropriately used analgesics actually
cause headaches. Suggest a drug holiday with weaning from caffeine and alcohol, proper sleep hygiene, diet, exercise and stress
management. Chronic headache may also be a symptom of depression or anxiety, arrange follow up to evaluate for these if the patient does
not improve.
17. Elderly woman in hospital post-op day 5 of total hip replacement. Acute chest pain, tachycardia, tachypnea, shortness of breath.
Manage.
Worry about: Life-threatening causes of acute chest pain: MI, PE, pneumothorax and tension pneumothorax, aortic dissection. Other causes
angina, gastritis, reflux, peptic ulcer, pericarditis, herpes zoster, musculoskeletal.
Differential Diagnosis for Chest Pain
Cardiac
Angina
Pulmonary
Pneumonia w/ pleuritis
MI
Pneumothorax
Pericarditis
PE
Myocarditis
Pulmonary hypertension
Dissecting aorta

Non-Cardiac
GI
Esophageal reflux
Ulcer

MSK/Neuro
Arthritis
Chondritis
Rib fractures
Herpes Zoster

Psychologic
Anxiety
Panic

History: Rapid cardiopulmonary history including any history of high blood pressure, heart problems, smoking, COPD.
Physical exam: Is a cardiopulmonary exam as in question #13 above with additional attention to risk factors for post-op complications (i.e.
inactivity/decreased mobility resulting in DVT/PE).
Homans sign: pain in the calf on dorsiflexion of the foot indicates thrombophlebitis. Check that trachea is midline. Inspect surgical
wound. Is the patient on DVT prophylaxis or anti-coagulation?
Treatment: Raise head of bed. Give oxygen 6 L/min by mask. Monitor oxygen saturation. Order stat CBC, lytes, glucose, INR/PTT, serial
CK-MB and Troponin, ABG, CXR, ECG. Give chewable ASA 160-325 mg immediately. Secure IV access, bolus IV lasix 40 mg, push if
fluid overload is suspected, and ventolin if wheezes are heard, give sublingual nitro spray or 0.3 mg sublingual nitro if blood pressure is
adequate and 1 mg morphine IV. Repeat nitro q5min x 3. May require additional morphine and nitro. Repeat CK-MB and Troponin q8h x 3.

ECG: if ECG shows significant ST elevation (more than one millimeter in two anatomically consecutive leads), or a new left bundle branch
block, then the patient is having an MI. Order stat Cardiology consult for possible lytic therapy or cardiac catheterization. If less severe
signs of ischemia are present (flipped T waves, ST depression), follow with repeat ECGs until resolved.
S1Q3T3: This classic pattern (wide S-wave in lead I, Q-waves in lead III, T wave inversion in lead III) with right axis deviation and RBBB are
signs of right heart strain seen in massive PE.
A-a gradient: An elevated A-a (Alveolar pO2-arterial pO2) gradient is a sign of pulmonary embolus but also occurs in any condition in which
there is a ventilation-perfusion mismatch (e.g. pneumonia, pulmonary edema, COPD). It is determined from the ABG:
A-a = 713 (FiO2) 1.25 (PaCO2) PaO2
[normal: 12 in child 20 in 70 year old]
Note that the inspired oxygen fraction (FiO2) is not known unless the patient is on room air, a ventimask or mechanically ventilated. This is
because the patient breathes in a proportion of room air which mixes with the oxygen delivered by face mask or nasal prongs thereby diluting
it by an unknown amount. Roughly, however, 2L/min gives 26% FiO2, 3L=30%, 4L=35%, and 6L=40%. 40% is considered the maximum
inspired oxygen obtainable without a high flow mask such as a ventimask.
ABG normal values: pH 7.35-7.45, pO2 80-100 mmHg, bicarbonate 24, pCO2 40
Indications for intubation: An ABG showing poor pO2 (in the 60s, or if less then 80 on high inspired O2 concentrations), elevated pCO2
(greater than 80), acidemia, or GCS < 8 (not able to protect airway) may indicate need for intubation if these are not quickly correctable.
Consult ICU.
CXR signs of PE: wedge-shaped infiltrate (Hamptons hump) or oligemic area, unilateral effusion, raised hemidiaphragm. A normal CXR is
also consistent, and usual, with PE.
Specific investigations for PE: CT chest (can only see PE which is large enough to be clinically significant), V/Q scan (conclusive when it
shows high or low probability), and serial (q2d) leg Dopplers for presence of DVT above the knee.
Treatment: if suspicion of PE is high, anticoagulate (before waiting for tests) with heparin 7500 U IV bolus, then infuse at 1200 U/h.
Measure PTT q6h, adjust dose for PTT 70-90s. If a diagnosis of PE is made, coumadin (warfarin) should be started, continue anticoagulation for 3 months.
18. Young mother with 6 week old baby has recently immigrated from Ghana. Poor English skills. Concerned about whether she
should have her baby immunized. Counsel.
Counseling: General principles of counseling are to be aware of communication barriers such as language difficulties, to understand the
patients objectives, fears, preconceptions, to deal with these in an empathetic, non-judgmental way and to normalize them to transmit
information in a way that is consistent with the patients expectations and understandable to them, and to invite further questions and
feedback.
History: Ask if the patient would prefer someone, perhaps a family member to translate. Ask about the patients concerns, what does she
want to know and why? Explain that vaccines protect children from diphtheria, tetanus, pertussis, polio (DPTP), mumps, measles, rubella
(MMR), influenza (Hib) and hepatitis B (Hep B). All of which were once common and caused serious, sometimes fatal illness in Canada,
and all of which are now hardly ever seen because of vaccines. Explain that because the vaccines stimulate the immune system, some
children have a temporary sore arm (with induration and tenderness) at the injection site, malaise, mild fever or rash. Allergic reactions to
vaccines also occur including urticaria, rhinitis, anaphylaxis. It is very rare to have a more serious reaction (seizures, encephalopathy have
been reported). Standard modern vaccines are not known to cause disease or to have long-term deleterious effects. Compare these risks with
the risk of not getting vaccinated. Explain the recommended immunization schedule (below), give the patient some information pamphlets,
invite further questions and ask her to return in two weeks for the childs first immunization.
Contraindications to vaccination: previous serious reaction to vaccine. Special contraindication to MMR, which is a live attenuated
vaccine suspended in egg white protein and preserved with neomycin: allergy to egg or neomycin, pregnancy and immunocompromised state

(except healthy HIV positive children). Special contraindications to the pertussis component of DPTP (which is thought to be the component
responsible for seizures and encephalopathic vaccination reactions when they occur): progressive neurologic disorder and epilepsy. Hib not to
be given after age 5. TdP should not be given in first trimester of pregnancy.
Recommended Immunization Schedule:
DPTP = diphtheria, tetanus, acellular pertussis and inactivated polio,
Hib = Hemophilus influenza type b, MMR = measles, mumps,
rubella, Hep B = hepatitis B, TdP = tetanus, diphtheria toxoid and
pertussis, Td = tetanus and diphtheria toxoid, NB: MMR is
administered SC while the others are IM.
2 months
DPTP, Hib (given as Pentacel)
4 months
DPTP, Hib
6 months
DPTP, Hib
1 year
MMR
18 months
DPTP, Hib
4-6 years
MMR, DPTP
12-13 years
Hep B (3 doses initial, 1 month, 6 months)
14-16 years
TdP (certificate of immunizations for high school)
q10yrs
Td
19. 40 year old woman, appears sad, requests sleeping pills. Manage.
History: Name, age, occupation. Social situation. Stressful life events. Depression commonly presents with sleep disturbance therefore
screen and treat for depression as in question #4 above. This should be in addition to a sleep history. Medications, allergies,
drugs/smoking/alcohol, past medical/surgical history, family history, review of systems.
Sleep history: usual requirements, chronology of sleep problems, stressor, sleep hygiene (when, where, regularity, shifts at work, quiet, late,
exercise, meals, alcohol, caffeine, prescription and non-prescription remedies, drugs and medications), sleep latency (time to fall asleep),
nocturnal awakening, early morning wakening, daytime somnolence, somnolence while driving, working or during conversation.
Proper sleep hygiene: regular bed and wake times, avoid daytime naps, regular exercise but not late in the evening, do not use the bed for
reading, TV, paperwork, etc., avoid caffeine, alcohol, smoking.
20. 60 year old woman with multiple pains investigated by several other doctors, all lab tests normal. Manage.
History of multiple pains: should address the differential for multiple pains.
Differential for multiple pains:
Depression with somatization: major depression presents with a somatic complaint; commonly headache, stomach pains, sleep disturbance,
eating disturbance, or bowel habit changes. This is a frequent presentation of depression in the elderly. Treat as in depression (see #19
above).
Somatization disorder: multiple non-intentional complaints in multiple organ systems beginning before age 30 that occur over several years,
with treatment sought and significant impairment in functioning. Diagnostic criteria: at least 8 physical symptoms that have no organic
pathology: 4 pain symptoms at 4 different sites, 2 GI symptoms other than pain, 1 reproductive or sexual symptom other than pain, 1 pseudoneurological symptom (e.g. temporary blindness). Complications: anxiety, depression, unnecessary medications or surgery. Often a
misdiagnosis for an insidious illness so rule out all organic illnesses (e.g. MS). Treatment: counseling, psychotherapy, close follow-up,
reassurance.
Conversion disorder: psychic perturbation presents as one or two neurological complaints affecting voluntary motor or sensory function.
Psychological factors thought to be etiologically related to the symptom as the initiation of symptoms is preceded by conflicts or other
stressors. La belle indifference patients inappropriately cavalier attitude towards a serious symptom. Treatment: anxiolytics (e.g.
lorazepam 1 mg PO q6h), relaxation therapy, counseling, close follow-up.
Pain disorder: e.g. chronic post-traumatic or post-surgical pain. Pain not fully accounted for by current tissue injury, exacerbated by psychic
factors and associated with functional impairment. Treatment: amitriptyline 25-75 mg PO qHS.

Hypochondriasis: exaggeration or misinterpretation of normal sensory phenomena to the point of functional disability. Evidence does not
support a physical disorder. Associated with obsessive fear of serious illness and doctor shopping despite reassurance. Belief is not
delusional as person acknowledges unrealistic interpretation. Treatment: counseling, reassurance, close follow-up.
Fibromyalgia: also called fibrositis and fibromyositis. 80-90% of cases occur in middle-aged females, may afflict 5% of adult women,
typically cardiovascularly unfit, depressed, previously normal life (onset often after car accident). Associated with absent or decreased nonREM stage 4 sleep, patients wake from sleep feeling unrefreshed. Constant, aching, axial pain with bilateral tender points (not trigger points,
at which referred pain is triggered due to myofaschial pain from overuse, e.g. tennis elbow). The disorder follows a waxing and waning
course ultimately without progression or resolution, and may become disabling. Characteristic reproducible tender points are located
bilaterally at lateral border of sternum, sternocleidomastoid, posterior neck, trapezius, rhomboids, over sacroiliac joints, lateral thigh,
posterior and medial knee. Patient should have eleven of the above eighteen tender points for a diagnosis. Treatment: amitriptyline 25-75 mg
PO qHS.
Chronic fatigue syndrome: similar to fibromyalgia but fatigue is the dominant feature and pain and tender points may be less prominent or
absent. Treatment: amitriptyline 25-75 mg PO qHS.
Factitious disorder or malingering: Factitious disorder involves misrepresentation of history and symptoms for the purpose of assuming the
sick role with its inherent secondary gains (attention and sympathy, justification for inadequacies). Munchausens syndrome is the type of
factitious disorder in which physical findings are faked by contamination of lab tests or ingestion of inappropriate medication or substances.
Typically the patient is a medical paraprofessional motivated by hostility toward the medical establishment, e.g. nurse takes coumadin to fake
hemophilia. Malingering is distinguished from factitious disorder by a motivation for secondary gain other than the sick role, such as
insurance benefits. Treatment: counseling.
History: Pain description, location, duration, chronology, aggravating and relieving factors, are pains linked to one another?
Somatoform disorders screen: How has your health been for most of your life? How have your pains affected your job, social life,
relationships, and your life generally? Are you often unwell, how often do you visit the doctor? Do you worry that you have a serious
illness? If a doctor tells you that there is nothing wrong, how does that make you feel? Do you believe him or her?
Associated symptoms: review of systems, medications, allergies, smoking, alcohol, drug use, family history, depression history as in #4
above.
Diagnosis and treatment: For non-specific pains with depressive symptoms the patient most likely has depression with somatization. Treat
for depression as in #4.
21. A young man presents to the Emergency Department having twisted his ankle. Manage.
History for ankle sprain: history of a plausible mechanism of injury involving significant inversion or eversion of the foot with pain and
swelling. Time of injury, onset of pain and swelling (may be delayed), noises heard at time of injury. Previous ankle or other injuries. Ability
to walk post injury (often preserved if ligaments are not ruptured). Past medical history, medications, allergies, family history.
Physical exam: inspect for gross deformity, erythema, swelling, bruising. Check distal circulation, sensation, active and passive range of
motion, palpate for tenderness at joints. Examine the joints above and below the affected joint. Identify sites of maximal tenderness. Point
tenderness over the area anterior (anterior talofibular ligament), inferior (calcaneofibular ligament), or posterior (posterior talofibular
ligament) to the lateral malleolus are signs of lateral ligamentous injury. Tenderness over the area medial and inferior to the medial malleolus
indicates deltoid (medial) ligament injury.
Talar drawer sign: Stabilize the tibia and pull forward on the heel, talar drawer sign is anterior movement of the talus. Greater than 3 mm
anterior movement may be significant. 1 cm is significant and indicates anterior talofibular ligament rupture.
Talar tilt: Stabilize the tibia, grasp the talus and tilt in inversion and eversion. Movement beyond the normal range compared with the
opposite side is a positive talar tilt and indicates lateral calcaneofibular ligament rupture if the tilt occurs in inversion or medial (deltoid)
ligament rupture if the tilt occurs in eversion.

Squeeze test: Pain in the ankle on squeezing the calf is a sign of ankle fracture.
Ottawa Ankle Rules: for taking ankle series x-rays (includes lateral and AP ankle with mortis view). X-ray if there is pain over the malleolar
zone AND tenderness on palpation of the posterior medial or lateral malleolus OR if the patient is unable to weight bear immediately and in
ER. For foot series (AP and lateral foot): X-ray if there is pain in midfoot zone AND bony tenderness over the navicular or base of 5 th
metatarsal OR unable to weight bear immediately and in ER. Calcaneal views if there is pain on palpation of heel.
Treatment for ankle sprain: (remember RICE) Rest, use crutches, avoid weight bearing but early mobilization. Ice for 20 minutes QID
for 2-3 days. Consider NSAIDs: Ibuprofen 400-600 mg PO q6h if no contraindications. Compression with tensor bandage or tape but not to
the point of pain. Elevate. Rehabilitation: start active range of motion exercises 2 days post injury, may weight bear after pain and swelling
have subsided. Full ligament healing may take 6 weeks in severe injury or more if re-injury occurs. Complete ligament rupture with joint
instability (positive talar drawer sign or talar tilt) should be evaluated by Orthopedics.
22. 16 year old known epileptic on Dilantin is having 3 seizures per month and requests better medication. Manage. Findings: not
taking meds, experiencing stress.
See question #3 above.
23. 50 year old woman with headache and normal vitals. Take a history. Q: Describe appropriate investigations and treatment for
temporal arteritis.
History: description of headache pain, location (one-sided vs. bilateral/occiput-vertex Do you feel pain on one or both sides? If one-sided,
is it always the same side? If present on both sides, did the pain start on one side? Is it usually maximal on one side?), quality (pulsatile vs.
non-throbbing What kind of pain is it tightening, pressing, throbbing, pounding, pulsating, burning, etc? Do different types of pain occur
at different times in any one attack? If so, what types?), intensity, duration (at least 72 hours in migraine, if not treated), onset including time
of day (morning headache associated with raised intracranial pressure), previous episodes, aggravating/relieving factors (e.g. coughing and
straining worsen headache in raised ICP and chocolate or cheese can trigger migraines), associated symptoms (aura, nausea, vomiting,
photophobia light, phonophobia sound, osmophobia odors, nuchal rigidity, weakness, numbness, visual disturbances). A history of
unilateral lancinating pain with swelling and tenderness in the temporal area should prompt inquiry after symptoms of polymyalgica
rheumatica (PR pain and stiffness in muscles of neck, shoulders, upper arms, hips, lower back and thighs no weakness or atrophy
increased ESR, anemia, normal CK responds to steroids immediately), which is related to temporal (giant cell) arteritis and may be a more
systemic variant of the same underlying disease. Symptoms of both PR and temporal arteritis include low grade fever, malaise, anorexia,
weight loss, bilateral proximal muscle weakness, aching and pain, as well as joint inflammation. Jaw claudication, stroke and blindness may
occur due to vasculitic occlusion of arterial supply. Ask about visual changes. Medical history, medication history (when was this prescribed,
do you have the empty bottle, has it been prescribed before?), current medications, allergies, family history, substance abuse inquiry,
smoking, allergies, mood, stress, anxiety inquiry. Review of systems.
Investigations for temporal arteritis: CBC (mild anemia with increased WBC), ESR (greater than 50 mm/h, [normal 30]), C-reactive
proteins, liver enzymes, temporal artery biopsy, may add temporal artery angiogram to guide biopsy.
Treatment: (in the absence of visual symptoms) without waiting for biopsy, start high dose oral prednisone 60 mg PO OD until symptoms
subside and ESR normal, then 40 mg PO OD for 4-6 weeks, then taper to 5-10 mg PO OD for 2 years (relapses occur in 50% if treatment is
terminated before 2 years). Treatment does not alter biopsy results if the sample is taken within 2 weeks. Monitor ESR regularly. If visual
symptoms are present, or develop during treatment, the patient is admitted and given prednisolone 1000 mg IV q12h for 5 days.
24. HIV positive man. 1 week of shortness of breath, cough, fatigue. Perform a physical exam. Q: Give a differential diagnosis for a
CXR showing a fine reticular pattern in the left lower lobe. Manage.
Physical exam: A physical exam for query pneumonia consists of the cardiopulmonary exam as in question # 13 with additional attention to
the particular signs and symptoms of HIV infection.

Examination for lobar consolidation: In general, pulmonary effusion decreases transmission of breath and vocal sounds to the chest wall,
while consolidation (seen in pneumonia) increases it. Four maneuvers bring out the effect of increased transmission: tactile fremitus is
increased transmission of palpable fremitus to the chest wall while the patient repeats ninety-nine, bronchophony is enhanced
transmission of spoken words such as ninety-nine, egophony is a change from an ee to an ay sound over the affected area while the
patient sustains an ee sound, and whispered pectoriloquy is a marked increase in audibility through the chest wall over the affected area
while the patient whispers the words ninety-nine or one-two-three.
Signs of consolidation: increased tactile fremitus, percussion dullness, crackles, bronchial breath sounds, increased voice transmission
(bronchophony, egophony, whispered pectoriloquy).
Signs of HIV Infection: (and possible impending AIDS) check entire skin surface for Kaposis sarcoma, examine pharynx for thrush or oral
hairy leukoplakia (Epstein-Barr virus-related epithelial proliferation causing raised white plaques on the sides of the tongue), palpate neck,
clavicle, axillae, and groin for lymph nodes enlarged by non-Hodgkins lymphoma. Examine abdomen for hepatic or splenic enlargement.
Differential diagnosis of unilateral lobar reticular pattern on CXR: pneumocystis carinii pneumonia (PCP), Kaposis, lymphoma,
cytomegalovirus (CMV), tuberculosis, Cryptococcus neoformans, Hemophilus, Streptococcus, mycoplasma, chlamydia. The classic CXR of
PCP, an AIDS-defining illness, is bilateral hilar infiltrates, but X-ray findings are variable and may be alveolar or interstitial.
Investigations: O2 sats/ABG, CBC with differential and CD4 count, LDH (elevated in 95% of PCP pneumonias and not in other
pneumonias), blood cultures, sputum for cytology/gram stain/culture/TB stain if sputum available (cough usually non-productive and induced
sputum may fail), bronchoscopy with cytology, gram stain and culture of bronchial washings and brushings (may see bronchial Kaposis).
Treatment: Septra DS 2 tabs q8h x 14 days outpatient with 1 tab OD or BID 3/week continued as prophylaxis. More specific therapy with
results of diagnostic tests. In severe illness, admit to hospital, give IV Septra at same dose and Prednisone 40 mg BID x 5 days, then OD x 5
days, then 20 mg OD for prophylaxis. Patient should be referred to an AIDS specialist for antiviral and possible experimental therapies.
Counseling and referral to support organizations. Follow up appointment.
25. A 2 day old infant has serum bilirubin 220 mol/L (ref. Max 200 mol/L). Take a history from the mother. Q: What are the
possible causes for this abnormality? Give investigations and treatment.
50% of term infants develop visible jaundice (> 85-120 mol/L or 5-6 mg/dL)
Mothers obstetrics history: GTPAL (number of gestations, term pregnancies, premature births, abortions, live children), history of previous
pregnancies including neonatal jaundice, maternal medical history esp. liver disease, illness during pregnancy esp. diabetes (large birth
weight, pre-eclampsia), rubella (teratogenic), toxoplasmosis (from cats, infects fetal brain), herpes (infects fetus, frequently fatal), CMV
(damages fetal liver), teratogenic medications taken during pregnancy, drug and alcohol use, maternal blood type, complications of present
pregnancy including gestational hypertension or diabetes, hyper/hypothyroid, hypercoagulation. Family history of neonatal jaundice, liver
problems.
Newborn history: gestational age at birth, caesarean, induction, rupture of membranes artificial or prolonged, fetal distress, forceps or
vacuum delivery, meconium, APGARs, was resuscitation required? Initial blood work, breast feeding? How often and how well, color of 1 st
stool, color of urine, vomiting, neonate muscle tone, behaviors, fever, irritability, lethargy.
Causes of neonatal jaundice: unconjugated = physiologic neonatal jaundice OR pathologic: hemolytic ABORh incompatibility, neonatal
sepsis, splenomegaly, hereditary spherocytosis, G6PD etc.; non-hemolytic breast milk jaundice, breakdown of cephalohematoma,
polycythemia, sepsis, Gilberts, Crigler-Najjar, hypothyroidism. Conjugated: GI obstruction in fetus (increases enterohepatic circulation), bile
duct obstruction, drug-induced and multiple other less common causes.
Investigations: Use blood obtained by venipuncture, not heal prick, as sludging of heel prick blood in skin capillaries distorts cell counts and
concentrations. Measure direct (conjugated) and indirect (unconjugated) bilirubin, neonatal and maternal blood types, Coombs test (see
below), blood smear, CBC with reticulocyte count. Septic work-up, urinalysis, blood cultures, CXR, AXR, CSF.
Coombs test: a two-part test. 1. Direct antiglobulin test: detects presence of anti-red cell autoimmune antibodies attached to red cells.
Patients red cells agglutinate when mixed with a solution containing anti-human immunoglobulin antibodies, i.e. multiheaded anti-

immunoglobulin antibodies bind to the constant chains of antibodies on the red cell surface, sticking red cells together. 2. Indirect
antiglobulin test: detects presence of anti-red cell antibodies in patients serum. Patients serum incubated with red cells of the same blood
group, test is positive if agglutination occurs.
Treatment: The aim of treatment is to correct anemia and decrease hyperbilirubinemia. Severe hyperbilirubinemia may lead to kernicterus
(deposition of bilirubin in the brainstem and basal ganglia leading to mental retardation, cerebral palsy, hearing loss and paralysis of upward
gaze). Indications for intervention: In general, serum total bilirubin level greater than 300 um/L is an indication for phototherapy, while levels
greater than 400 um/L call for exchange transfusion or plasmaphoresis.
Therapeutic modalities: In phototherapy photoisomers of bilirubin are produced which are more water-soluble and can be excreted without
conjugation, while exchange transfusion replaces the babys blood with donor blood. Plasmaphoresis is the replacement of blood plasma
with donor plasma while retaining the patients own red and white cells.
Specific treatment of jaundice depends on the cause:
Physiologic jaundice: occurs on day 2-3 in 50% of term infants (NEVER within 1st day) and resolves by day 7. It is more common in
preterm infants where it occurs later, up to day 6, and lasts longer. Due to transient limitation in bilirubin conjugation (i.e. the
hyperbilirubinemia is unconjugated or indirect). Usually requires no treatment.
ABORh incompatibility jaundice: Rh incompatibility (Rh negative mother with Rh positive fetus) tends to be more severe than other
incompatibilities (ABO incompatibility: e.g. O mother with A or B fetus) and may cause hydrops fetalis (generalized edema, including
pulmonary edema, with high output heart failure). Treatment for all types is exchange transfusion or plasma electrophoresis.
Sepsis-related jaundice: treat underlying cause and use phototherapy.
Breast milk jaundice: occurs day 4-7 and peaks at 2nd to 3rd week, rare (1:200), long chain fatty acids in breast milk competitively inhibit
glucuronyl transferase activity. Treatment is to substitute formula for breast milk for 2-4 days, then resume.
26. A 50 year old man is denied life insurance because of abnormal liver function tests. AST > ALT very elevated, ALP slightly
elevated, Bili normal. Take a history. Q: Give a differential diagnosis. What investigations would you order?
History: Name, age, occupation. History of jaundice, hepatitis, foreign travel, blood transfusions, recreational IV drug use. Dark urine, pale
stool, abdominal pain, fever/chills, decreased appetite, weight loss, night sweats, nausea and vomiting, pruritis, easy bruising, gynecomastia,
hemorrhoids (from portal hypertension), alcohol use & CAGE questionnaire (see below). Sexual history: number of past and present partners,
genders of same, sexually transmitted disease. Medications, drug use, smoking, allergies, past medical history, family history, review of
systems.
CAGE questionnaire: Control have you tried to cut down on your alcohol? Anger have you ever felt angry when someone suggested you
decrease your alcohol intake? Guilty have you ever felt guilty about your drinking? Eye opener do you sometimes have a drink to get
started in the morning?
Liver Enzymes: AST & ALT are sensitive but non-specific markers of hepatocellular damage hepatitis (inflammation) vascular injury
(ischemia). AST > ALT = alcoholic liver disease; ALT > AST = viral hepatitis. ALP & GGT are markers of cholestatic disease
intrinsic (toxic, infectious, inflammatory), systemic (sepsis, pregnancy), infiltrative (tumor, fat, lymphoma), mass lesions (stone, tumor,
abscess). AST/ALT > 2 (AST usually < 300) alcoholic liver. Serum transaminases > 1000 due to 1) viral hepatitis, 2) drugs, 3) passage of
common bile duct stone, 4) hepatic ischemia.
Differential: alcoholic liver disease, viral hepatitis, liver malignancy (metastatic or primary).
Investigations: Viral serology (Hep A, B, C antibody and B antigen presence of B antigen for > 6 months indicates chronic carrier state),
GGT, AST, ALT, AlkPhos, LDH, bilirubin, INR/PTT, albumin, glucose (cirrhosis), serum ceruloplasmin, serum copper (Wilsons disease),
serum ferritin, total iron binding capacity (TIBC, for hemochromatosis), ANA, anti-smooth muscle antibody (autoimmune hepatitis, also
called chronic active hepatitis), abdominal ultrasound, liver biopsy.

27. 19 year old female with vaginal discharge. Take a history. Q: Give three possible diagnoses, what investigations would be
helpful?
History: Name, age, occupation, description of discharge, onset, chronology, previous episodes, volume, color, consistency, odor, timing
(related to menses?). Associated symptoms: pain including abdominal, burning, fever, itch, dyspareunia, dysuria, urgency, frequency,
aggravating and relieving factors. Sexual history: number of past and present partners, gender, type of contraception (condoms), possibility
of pregnancy, past history of sexually transmitted disease. Obstetrics/gynecology history: (GTPAL) pregnancies, abortions/miscarriages,
births, pap smears (normal?), menstrual pattern. Medications (especially antibiotics), oral contraceptives, other drug use, allergies. Past
medical history including diabetes. Family history, review of systems.
Causes of discharge: Physiological: normal midcycle discharge, increased estrogen states. Infectious: candidiasis, bacterial vaginosis
(Gardnerella vaginalis), trichomonas infection, chlamydia, gonorrhea (NOTE: gonorrhea and chlamydia can cause cervicitis, PID and
urethritis, but do not cause vaginitis but Toronto Notes includes them in the differential for vaginal discharge), bartholinitis or Bartholin
abscess, PID. Neoplastic: vaginal intraepithelial neoplasia (VAIN), vaginal squamous cell carcinoma, invasive cervical carcinoma, fallopian
tube carcinoma. Other: allergic/irritative vaginitis, foreign body, atrophic vaginitis, enterovaginal fistulae.
Investigations: speculum exam, swab and culture, saline slide microscopy and KOH whiff test (add KOH to vaginal secretions on a slide).
These give results as follows:
Candidiasis: inflamed appearance, lumpy white discharge, spores and pseudohyphae seen under microscope. Treatment: miconazole vaginal
suppository.
Bacterial vaginosis: non-inflamed, thin gray secretions, clue cells under microscope (epithelial cells with obscured borders due to adherence
of bacteria), fishy odor on KOH test. Treatment: metronidazole 500mg PO BID x 7 days (in pregnancy use Amoxicillin 500mg TID x 7
days).
Trichomonas: inflammations, frothy yellow-gray-green discharge, motile trichomonads seen under microscope. Treatment: metronidazole 2
g x 1 or 500mg PO BID x 7 days (in pregnancy use Clotrimazole vaginal suppositories).
28. 60 year old man with microscopic hematuria on routine urinalysis. Take a history. Q: Give a differential diagnosis, what
investigations would be helpful?
Etiology of Hematuria by Age Group
Age
Etiology (in order of decreasing frequency)
0-20
Glomerulonephritis, UTI, congenital anomalies
20-40
UTI, stones, bladder tumor
40-60
Male: bladder tumor, stones, UTI
Female: UTI, stones, bladder tumor
> 60
Male: BPH, bladder tumor, UTI
Female: bladder tumor, UTI
History: patient ID, suprapubic pain, pain on urination, frequency, urgency, frank blood in the urine (globular clots from bladder or string
shaped clots from ureters), color of urine, difficulty initiating or maintaining urinary stream, renal pain, back pain, groin pain. Provoking
factors (e.g. exercise, trauma). History of recent UTI, STDs, TB exposure, pelvic irradiation, bleeding diathesis, smoking. Fever, chills,
nausea, fatigue. Previous renal colic/diagnosed nephrolithiasis? History of hypercalcemia, hypertension. Malignant symptoms: night
sweats, weight loss, fatigue. Medications, drugs (NSAIDs, anticoagulants)/alcohol, smoking, anticoagulants and salicilates, past medical
history, past surgical history, family history (polycystic kidney disease?), review of systems.
Differential diagnosis: BPH, transitional cell carcinoma of bladder, UTI, nephrolithiasis, hydronephrosis, prostatitis, prostate cancer, renal
cell carcinoma, glomerulonephritis, trauma (heart valve, muscle injury/breakdown), essential hematuria (tends to occur in children). NOTE:
microscopic hematuria is normal in 10% of population.
Investigations: prostate specific antigen (PSA), CBC (to rule out anemia, leukocytosis), urine for microscopy (casts, crystals, culture and
sensitivity, cytology), abdominopelvic ultrasound, cystoscopy, intravenous contrast urography, intravenous pyelogram (IVP).

29. 25 year old male with a history of dyspepsia and binge drinking has abdominal pain. Perform a focused physical exam. Q: What
radiological investigations would you order and why?
Physical exam for abdominal pain: vitals, posture (unmoving in fetal position suggests peritonitis while writhing suggests renal colic),
jaundice, nutritional status, buccal mucosa, teeth, breath (hepatic fetor), parotid hypertrophy, glossitis, inspect chest for telangectasia,
gynecomastia, loss of axillary hair. Hands: palmar erythema, clubbing, Dupuytrens contracture, wasting of hand intrinsics.
Abdominal exam (supine): inspect for caput medusa, Cullens sign (umbilical bruising intraperitoneal bleed), Gray-Turners sign
(bruising of flank retroperitoneal bleed), pulsations, auscultate for bowel sounds, renal bruits. Estimate size of liver and spleen by
percussion. Palpate for liver edge, Murphys sign (examiner presses on liver edge after patient has exhaled, patient catches breath on
inspiration), splenic enlargement (begin palpation at right lower quadrant to catch very large spleen and percuss at Traubs space), hard stool
in bowel. Note cough tenderness, rigidity, rebound, guarding, tenderness, pain at McBurneys point (one third of the way along the line
between the right anterior iliac crest and the umbilicus). Rovsings sign: palpation of the LLQ produces RLQ pain. Psoas sign: pain on
passive or active flexion at the hip, indicates peritoneal irritation over the psoas or psoas abscess. Obturator sign: pain on internal or external
rotation of the hip, indicates bowel herniation into the obturator canal. Assess for ascites. Ask patient to roll onto side and pound
costovertebral angles lightly with fist, CV angle tenderness indicates kidney pain due to pyelonephritis or nephrolithiasis. Palpate groins for
hernias.
Rectal: palpate prostate, rectal shelf, check for gross or occult blood.
Radiological Investigations: Abdominal 3 views: supine, upright, left lateral decubitus. Dilated bowel with multiple air/fluid levels
indicates ileus. Dilated proximal bowel with collapsed distal bowel indicates obstruction. Check for gastric distension. Small bowel has
circular plica: lines go all the way across. Large bowel has interrupted haustra: lines go halfway across. Check for calcified kidney stone,
fecolith and appendiceal air/fluid level. Can see gallstones and abdominal aortic aneurysm if calcified. Abdominal ultrasound: gallstones,
cholecystitis, pancreatitis, appendicitis, hydronephropathy, kidney stones, abdominal aneurysm. CXR: check for free air under the diaphragm
in an upright film.
30. 70 year old male with neck pain and left arm weakness. Perform focused physical exam. Findings: decreased sensation over left
index and middle finger, mild wrist extensor and triceps weakness. Q: Describe a cervical spine film of the patients neck (shows
narrowing of C6-7 disk space). Diagnosis and treatment?
Physical exam for neck pain: guided by differential for the causes of neck pain: musculoskeletal, discogenic, stenotic, malignant or
brainstem infarct. Take vitals, cranial nerves: pupillary reflexes, extra-ocular movements, visual fields, facial muscles (frown, raise
eyebrows, show teeth, protrude tongue), facial sensation, gag, Horners triad (ptosis, anhidrosis, miosis on side of sympathetic palsy?),
sternocleidomastoid and trapezius power. Cerebellar testing: finger-nose, heel-shin, dysdiadocokinesis, gait, Rhomberg, Pronator drift.
Neck: inspect for lesions, asymmetry, muscle wasting, especially sternocleidomastoids, palpate for nodes, masses, palpate dorsal vertebral
spines, range of motion.
Shoulders, arms, hands: inspect for symmetry, wasting, fasciculations, skin lesions. Power: Test deltoids (C5), biceps (C6), triceps and
wrist extension (C7), hand intrinsics (C8). Note that each muscle group actually has mixed nerve root innervation, i.e. deltoids and biceps
(C5,6), triceps (C6,7,8), wrist extension (C6,7), hand intrinsics (C8,T1). Sympathetic outflow occurs at C8, T1. Sensation: check pinprick,
vibration, light touch over the shoulder (C5), thumb (C6), index and middle finger (C7), ring and little finger (C8). Deep tendon reflexes at
biceps, triceps, brachioradialis, Hoffmans sign (the Babinski of the upper limb: flick relaxed index finger dorsally, thumb abducts for positive
test). Tone/rigidity: check for increased tone by rapid supination and rapid extension of elbow.

Lateral Cervical Disc Syndrome


C4-5
Root involved C5
Motor
Deltoid
Supraspinatus
Biceps
Reflex
Supinator
Sensory
Shoulder

C5-6
C6
Biceps

C6-7
C7
Triceps

C7-T1
C8
Digital flexors
Intrinsics

Biceps
Thumb

Triceps
Middle finger

Finger jerk
Ring finger
Little finger

Peripheral nerves: Check two point discrimination at each fingertip. To determine peripheral nerve damage: Median nerve territory is the
palmar surface of the thumb, and the palmar surface and dorsal tips of the index, middle and thenar side of the ring fingers. Sensation to the
ulnar side of the hand is the ulnar nerve, and the dorsal surface of the thenar side of the hand is radial nerve innervated. The median nerve
also innervates most muscles of the thenar eminence, and the 1st and 2nd lumbricals. The thumb is weak in abduction at 90 degrees to the
plane of the hand in median nerve dysfunction. Tinels sign: tapping the palmar surface of the wrist elicits shooting paresthesia in median
distribution. Phalens sign: maximally flexing both wrists by pushing the dorsi of the hands together elicits median nerve distribution
numbness or paresthesias after 30-60 seconds.
C-spine X-rays: Lateral: an adequate lateral shows the top of the T1 vertebra: look for alignment of the anterior and posterior margins of
vertebral bodies as well as spinous processes. Spinous processes may have abnormal separation in injury. The maximal normal distance
between the posterior aspect of the anterior arch of C1 and the dens is 3mm in adults and 5mm in children. Look for regularity of disk space
height, gas in the disk space (suggests degeneration), osteophytes, pre-vertebral swelling greater than one third of the vertebral body width (7
mm from C1-4, 22mm from C5-7). Hangmans fracture: coronal plane fracture through the base of both pedicles of C2, caused by
hyperextension injury, separates the posterior elements of C2 from its body. AP: check alignment of processes and vertebral bodies, distance
between spinous processes should be regular, pedicles should be seen in cross section (erosion of a pedicle can cause the winking owl sign
where the pedicles are the eyes and the spinous process, the beak). Odontoid view: trace bone cortex around the outline of the dens,
misalignment of this outline indicates odontoid fracture, articular spaces of atlantoaxial joints on either side of the dens should be equal.
Note: odontoid fracture type I tip, type II base, type III through body of C2.
Diagnosis: A narrowed C6,7 disk space suggests disk degeneration at that level. C6,7 disk herniation would impinge on the C7 nerve root
(cervical roots exit above the vertebra of the same number) which is consistent with clinical C6,7 nerve root dysfunction on sensory and
motor exam.
Treatment: most patients respond to conservative therapy: soft collar, NSAID, acetaminophen. If symptoms persist for two weeks or
neurologic symptoms progress, refer to Neurosurgery for myelogram, CT neck and possible CT myelogram, MRI, or EMG, nerve conduction
studies. May require decompressive laminectomy or anterior discectomy with bone graft fusion.
31. A 30 year old patient with type I diabetes presents to the emergency department with abdominal pain and vomiting. Take a
history. Q: Labs: Glucose 25, K 6.0, pH 7.22, Bicarb 14. What is your diagnosis and management?
History for abdominal pain and vomiting: quality of the pain, location, onset, chronology, radiation, associated symptoms, aggravating and
relieving factors. Number of episodes of vomiting, description of vomit, presence of blood and bile. Associated prodromal illness, fever,
malaise, sore throat, cough, urinary symptoms, diarrhea. Foods eaten, other people sick? Previous similar episode? Polydipsia, polyuria,
lethargy, anorexia, hyperventilation? Other precipitants of DKA, recent surgery, recent trauma, pregnancy, MI, non-compliance or wrong
insulin dose, infection.
Diabetes history: time since diagnosis, medications, blood sugar monitor at home? Diabetic control, polyuria, polydipsia, diet, exercise,
drugs, alcohol, smoking, complications of diabetes (retinopathy, neuropathy, nephropathy, infections). Who follows patients DM? Has
patient taken insulin since feeling unwell, last insulin dose? Past medical history, current medications, allergies, family history, review of
systems.
Diagnosis: diabetic ketoacidosis.

Management: Foley, IV, lytes, glucose, ABG, serum ketones. Septic work-up: CBC, CXR, blood cultures, urinalysis, ECG if K is critically
elevated. 1 L NS per hour x 2-3 hours or until tachycardia and BP normalize, then 500 cc/hr x 2 hours, then 250 cc/h x 5 hours. Insulin drip
at 2 U/hr. Check glucose and lytes q2h. When glucose drops to 15, switch fluids to maintenance 2/3-1/3 D5W/NS + 20 mEq KCl/L (4:2:1
rule). Begin diet and regular insulin regimen. If the DKA was the result of non-compliance close follow-up and education such as diet and
diabetes management counseling with a dietitian are required.
32. A mother with her 6 week old who has been vomiting for three days. Take a history. Q: Investigations show a palpable mass in
the right epigastrium, metabolic hypochloremic alkalosis. What is the diagnosis? Give a differential diagnosis for vomiting in an
infant.
History of infant vomiting: age of onset, duration, severity, chronology, association with feeding or body position, description of force,
volume, color, composition (bilious, fecal, blood, regurgitant), getting worse or better, is child still hungry afterward, or does he settle.
Coughing or gagging with feeds (tracheoesophageal fistula). Associated diarrhea, constipation, fever, weight loss, abdominal distention,
urination. Are other children sick? Has child been in contact with an infected person.
Mothers obstetrics history and newborn history: as in question #25 above
Development history: age and weight normograms, feeding history: quantity, frequency, breast vs. bottle (which formula), colic, feeding
difficulties. Past medical history, medications, family history.
Diagnosis: pyloric stenosis.
Differential diagnosis for infant vomiting: Newborn: congenital malformation (pyloric stenosis, tracheoesophageal fistula, duodenal atresia,
malrotation of the intestine). Post newborn period: gastroenteritis, peritonitis, appendicitis, hepatitis, ulcers, pancreatitis, overfeeding, reflux,
food allergy, milk protein intolerance, systemic infection.

1995
33. 79 year old female collapses in the mall. Patient is drowsy, unresponsive to verbal stimuli. Manage. Findings: HR 40, BP 80/40,
ECG complete heart block.
Rescusitation: ATLS/ACLS format as in question #6 above.
Management of complete heart block: (P waves seen on ECG not related to QRS complexes). Transcutaneous pacing (atropine 1 mg IV
may be tried but is rarely effective). Patient will require sedation (midazolam 2 mg IV) and analgesia (morphine 2 mg IV) before starting
external pacing. Will require placement of a transvenous pacer until a permanent pacer can be placed. Consult Cardiology/CCU/ICU.
Causes of AV conduction abnormalities: calcification of the conducting system (Levs and Lenegres disease), inferior MI, coronary spasm,
digitalis overdose, tricyclic antidepressant overdose, -blockers, calcium channel blockers, viral rheumatic fever, Lyme disease, sarcoid,
amyloid, hemochromatosis, cardiac tumor, congenital.
34. 25 year old male with tricyclic antidepressant overdose. Manage.
Resuscitation: ATLS/ACLS as in question #6 above ABCD, orders, secondary survey, second orders.
History (from patient and family/friends): ask for the empty pill bottles to confirm the drug (this may be a pass criterion), how many pills,
when taken, concurrent ingestions of alcohol or other drugs. Where was the patient found? Was there a period of unconsciousness, how long
did this last? Other symptoms (visual blurring, seizure). Did the patient give any warning of the attempt (note, phone calls, giving away
possessions), was there a preceding depression or strange behavior, problems at work or with a relationship. Previous attempts? Medications,
drugs, alcoholism, smoking, allergies, past medical and psychiatric history, family history, review of systems.
Investigations: CBC, lytes, urea, creatinine, glucose, INR/PTT, ABG, CK, serum osmolality, ALP, AST, ALT, total bili, GGT, Toxicology
Screen (ASA, acetaminophen, TCA level, barbiturates, benzodiazepines, EtOH), ECG. CXR (for aspiration pneumonia).
Treatment: Gastric lavage (rare) if less than 1 hour since ingestion. Activated charcoal 10 g/g drug ingested or 1 g/kg body weight NG.
Hydrate with normal saline to promote diuresis for excretion of TCA and possible myoglobinuria (occurs due to muscle breakdown following
prolonged coma lying on a hard surface). Alkalinize with 1 amp bicarb IV (or 1-2 mEq/kg) and hyperventilation if the patient is intubated.
Follow ABGs or venous gases, aim for pH 7.45-7.55. Remaining treatment is symptomatic: treat seizures with lorazepam 2 mg IV, treat
cardiac dysrhythmias, hypotension, agitation and coma as they arise. Consult ICU for 24 hours minimum monitoring. QRS > 0.1s indicates
increased risk of seizures and dysrhythmias. Psychiatric consult after patient is medically cleared.
TCA Toxicity:
Therapeutic levels are 2-4 mg/kg. Life-threatening symptoms usually occur at levels > 10 mg/kg.
Anticholinergic effects: hyperthermia, tachycardia, mydriasis (dilated pupils), decreased sweating and secretions, vasodilation, constipation,
urinary retention, ileus. CNS effects are generalized seizures, myoclonus, ataxia, hyperreflexia, confusion, agitation, hallucinations, acute
psychosis, decreased level of consciousness, respiratory depression (mnemonic: Hot as a hare, blind as a bat, dry as a bone, red as a beet, mad
as a hatter, the bowel and bladder lose their tone and the heart goes on alone).
Quinidine effects: conduction delay (prolongation of QRS, PR, QT, T wave flattening), heart block, bradycardia, asystole, ventricular
dysrhythmias and resultant hypotension.
35. 16 year old female in hospital for ASA overdose. Medically cleared. Take a history.
History: patient name, age, occupation. Circumstances surrounding the attempted suicide. Preceding conflicts at work or with family or in a
relationship. Recent loss of employment or loved one. Warning signs: suicide note, giving away prized possessions. Describe the attempt,
how many pills taken, what kinds, concurrent alcohol or drug use. Did the patient really want to die or was the attempt a cry for help? Gauge
the lethality of the attempt in terms of the means used and the chances of discovery. Previous attempts, describe these. Is patient now
actively suicidal or remorseful. If the patient actively suicidal, what is the current plan? Medications, drug/alcohol use, allergies, past
medical history, family history (esp. psychiatric), social supports: review of systems.

Psychiatry: History for depression/mania, mental status exam, multiaxial diagnosis as in question #4 above.
36. Father with 3 year old child who is not speaking. Take a history. Findings: not speaking well, recurrent ear infections, poor
hearing. Make a diagnosis.
History: of not speaking should determine whether the problem is primary (never spoke) or secondary (stopped speaking). Secondary causes
of mutism are psychological upset (due to family discord, etc.) and rare inherited neurodegenerative conditions. Primary mutism may be part
of a global developmental delay or related to hearing problems which are either congenital (inherited, intrauterine infections), ototoxic drugs
(e.g. streptomycin) or trauma.
Pregnancy and birth history: GTPAL (number of gestations, term pregnancies, premature births, abortions, live children), history of
previous pregnancies including neonatal jaundice, maternal medical history, illness during pregnancy, rubella (teratogenic), toxoplasmosis
(from cats, infects fetal brain), herpes (infects fetus, frequently fatal), CMV (damages fetal liver), teratogenic medications taken during
pregnancy, drug and alcohol use, family history of deafness or late speaking. Newborn history: gestational age at birth, caesarean, induction,
rupture of membranes artificial or prolonged, fetal distress, forceps or vacuum delivery, meconium, APGARs, was resuscitation required?
Initial blood work, breast feeding? How often and how well, color of 1st stool, color of urine, vomiting, neonate muscle tone, behaviors, fever,
irritability, lethargy.
Developmental history (from parent): See question #9. Growth: expected height and weight for age? Speech, has child ever spoken words
or phrases, are these used appropriately, has the child made sounds, chronology and description of these. How does the child communicate if
not through speech? Gross Motor: when did the child start walking, running. Fine motor: when did you notice pincer grasp, turning pages in
a book. Social behavior.
Hearing: does the child wake up in response to sounds? Startle to loud sounds? Come when called? Understand spoken instructions?
History of ear infections, wax problems. Ask about swimming. Past medical history, medications, allergies, family history, review of
systems.
Diagnosis: given recurrent otitis media with poor hearing the most likely diagnosis is retarded speech development due to poor hearing.
Refer to ENT for hearing tests, possible tubes (tympanic drainage).
37. 40 year old female with fatigue. Take a history. Findings: cold intolerance, weight gain. Make a diagnosis.
Fatigue history: onset, chronology, past episodes, functional limitations, associated with exertion? Recent viral illness (mononucleosis), cold
intolerance, weight gain, dry skin, brittle hair, hoarseness (hypothyroidism), associated muscle aches (fibromyalgia), chest pain (angina),
shortness of breath (congestive heart failure). Sleep history: usual requirements, chronology of sleep problems, stressor, sleep hygiene
(when, where, regularity, shifts at work, quiet, late, exercise, meals, alcohol, caffeine, prescription and non-prescription remedies, drugs and
medications), sleep latency (time to fall asleep), nocturnal awakening, early morning wakening, daytime somnolence, somnolence while
driving, working or during conversation. Depression screener: as in question #4 above. Must fully explore suicidal ideation: does patient
intend to harm self, reason for suicidal thoughts, current plan, lethality of plan, access to lethal means, has patient given away prize
possessions or written final notes to loved ones, previous attempts. Medications (especially TCAs, sedatives, antihypertensives), allergies,
drug/alcohol use, smoking, past medical history, family history, review of systems.
Differential Diagnosis: Chronic fatigue, CHF, ischemia, thyroid disease, sleep disturbance, depression.
Diagnosis: most likely hypothyroidism given cold intolerance and weight gain.
38. 35 year old male with back pain and stiffness. Take history and perform a focused physical exam. Findings: 10cm separation
between lumbar spines while erect increases by less than 5 cm when back is flexed forward (positive Wright-Schober test), lateral
flexion impaired. Q: Give the diagnosis and two associated conditions.
History and Physical: see question #15.
Diagnosis: based on typical history of back pain, lumbar spine X-rays showing fusion of the sacroiliac joints or sacroiliitis and
syndesmophytes (disk spaces undergoing fusion), elevated ESR and HLA-B27 tissue antigen positive this is probably ankylosing spondylitis.

Associated conditions: inflammatory arthritis, uveitis, psoriasis, IBD, amyloidosis, radiculopathy, pericarditis, angina, conduction
abnormalities.
Treatment: No cure. Regular therapeutic exercise to prevent deformity/disability (esp. swimming and back extension exercises). To control
pain and stiffness: indomethacin (100 mg PO OD), naproxen (250 mg PO BID-TID), etc. Surgery helpful in severe cases: e.g. total joint
replacement.
39. 35 year old male with 1 year history of back and leg pain. Do a physical exam. Findings: right foot dorsiflexion weakness with
dorsal foot numbness. Make a diagnosis and identify the level of lesion.
History and Physical exam: see question #15.
Diagnosis: L4,5 disk herniation with L5 radiculopathy.
40. 47 year old male with impotence. Wants a pill for this. Fears losing his girlfriend. Take a history and counsel.
Background: The causes of erectile dysfunction are subdivided in to psychic and organic categories. Current literature favors an
approximately 1:1 ratio of organic to psychic etiology. The problem is rarely primary (never had ability to sustain erection). The organic
causes are: drugs (-blockers, thiazides, H2 blockers, antidepressants, antipsychotics, Digoxin, clofibrate, sedatives, alcohol, heroin),
hormonal (diabetes, gonadal dysfunction, prolactin-secreting pituitary tumor, associated with loss of libido and testicular atrophy), neurogenic
(stoke, MS, spinal cord injury), iatrogenic (drugs, surgery, radiation), vascular (peripheral vascular disease, DM, HTN). Impotence is grossly
defined as inability to have satisfactory intercourse due to erectile dysfunction in at least 25% of encounters.
History: Current partners. Problems in these relationships. Why is the patient seeking medical attention for this now? Onset of erectile
dysfunction and chronology. Description of the problem: no erection at all, cannot sustain erection, ejaculate too quickly to satisfy partner,
cannot achieve orgasm or orgasm without ejaculation, retrograde ejaculation. Circumstances under which impotence occurs: only with
certain partners, only at certain times or locations, what percentage of the time? Is impotence related to lack of sexual desire? Presence and
firmness of morning or nocturnal erections. Does the patient sustain erections in masturbation? Associated problems: anxiety attacks,
anhedonia/depression, perineal or peripheral numbness, poor peripheral circulation. Exercise, medications, contraceptive use, drug/alcohol
use, smoking, cholesterol, allergies, past medical history, family history, review of systems.
Counseling: Discuss causes of impotence in terms of organic vs. inorganic etiology and that it tends to cause great anxiety (normalize
patients feelings). Erectile dysfunction can often be improved with lifestyle changes: exercise, weight loss, improved diet, decreased alcohol
intake, smoking cessation, stress management, sleep hygiene, better diabetic control, and joint counseling with partner to decrease anxiety.
Improvement of patients relationship with partner: address sexual boredom. Review medications: suggest changes. Explain that many
organic causes of impotence are unfortunately not reversible. Describe therapeutic options: counseling with partner on alternative means of
sexual gratification, testosterone preparations or bromocryptine (for prolactinoma) if patient is shown to have hormonal disturbance on blood
work (measure testosterone and gonadotropins), sildenafil (viagra), yohimbine and trazodone preparations for impotence (poorly effective
and expensive), penile self-injection with phentolamine, papavarine & PGE1 or PGE1 alone (30 gauge needle, last 30-60 min., quite
popular), vacuum-rubber ring device, penile prostheses. Arrange follow up with both partners.
41. Young woman with tunnel vision. Negative investigations by a neurologist and ophthalmologist. Take a history. Findings:
concerned that her husband is having an extramarital affair. Counsel.
History: Description of visual problem, functional limitations, onset duration, chronology, aggravating/relieving factors, associated headache,
eye pain, nausea, anxiety, palpitations, tremor. Previous episodes of eye problems or other unusual phenomena (blindness, paralysis,
numbness, abdominal pain). What doctors has patient seen, what did they say? Problems at work, home, with relationships. Who can the
patient go to for support in her life? Psychiatric problems in the past? Depression screener and sleep history as in questions #3 and #19.
Medications, drug and alcohol use, allergies, past medical history, family history, review of systems.
Conversion disorder: psychic perturbation presents as one or two neurological complaints affecting voluntary motor or sensory function.
Psychological factors thought to be etiologically related to the symptom as the initiation of symptoms is preceded by conflicts or other

stressors. La belle indifference patients inappropriately cavalier attitude towards a serious symptom. Treatment: anxiolytics (e.g.
lorazepam 1 mg PO q6h), relaxation therapy, counseling, close follow-up.
Counseling: Normalize this problem and validate the patients feelings: the fact that several specialists have said there is nothing wrong with
the patients vision does not mean that there is not a subtle medical problem which may become apparent later. For this reason it is important
to stick with one doctor who knows the patient well and can coordinate further referrals if necessary. Many people who are faced with the
possibility of marital infidelity automatically activate a defense mechanism which gives them time to adjust, and which is not under
conscious control: Such a reaction also helps the patient to enlist needed support from others. This is a normal reaction for these people.
These symptoms can vary widely from paralysis, to numbness, to pains, to inability to speak, and visual problems including blindness and
tunnel vision. Outline a plan for management: address possible sources of anxiety and stress: a frank discussion with the patients husband
about fidelity is required and may be done privately or if both partners are willing, in consultation with you or a marital therapist. The patient
should try to consolidate a support network: parents, friends, etc. Consider depression, sleep or anxiety medications as appropriate. Arrange
follow up with both partners.
42. 30 year old male from another city. Wants Fiorinal prescription for chronic headaches renewed. Manage.
See question #16.
43. 23 year old female with 24 hour abdominal pain. Perform focused physical exam. Findings: peritoneal signs, point tenderness at
McBurneys point. Q: Give a differential diagnosis, order investigations. What further history would help confirm diagnosis?
Physical exam for abdominal pain: see question #29.
Rectal: rectal shelf, check for gross or occult blood. Also include a pelvic manual and speculum exam. (wont be asked to perform this at the
LMCC II). Check for pain with cervical motion (seen in PID), pain on palpation of ovaries, mass, cervical discharge. Take swabs (see
investigations).
Differential diagnosis: appendicitis, ovarian cyst, rupture or ovarian torsion, mittelschmerz, ectopic pregnancy (life-threatening), hepatitis,
cholecystitis, gastroenteritis, peptic ulcer, pelvic inflammatory disease (PID), urinary tract infection (UTI), pyelonephritis, kidney stone,
inflammatory bowel disease, intestinal obstruction due to volvulus or IBD.
Investigations: ABX 3 views, abdominal-pelvic ultrasound, CBC, lytes, urea, creatinine, INR/PTT, glucose, hCG. Urinalysis. Stool for
occult blood. Cervical swabs for culture and pap smear. If OR is imminent order type and cross for 2 units, CXR.
Helpful further history: a history of gradual onset vague periumbilical or LLQ pain migrating to a sharper, more localized pain in the RLQ
over several hours associated with anorexia, nausea, and controlled by still fetal posture suggests appendicitis.
44. 62 year old female with left lower quadrant pain. Perform physical exam. Findings: low grade fever, some abdominal distention,
LLQ tenderness without rigidity, poorly defined left lower quadrant mass. Q: Abdominal series shows multiple air/fluid levels.
Describe. Give differential diagnosis with most likely diagnosis. Order further investigations.
Physical exam for abdominal pain: see question #29 and #43.
Differential diagnosis: diverticulitis, diverticular abscess, constipation with obstruction, GI malignancy with perforation, gallstone ileus,
obstruction due to volvulus (usually RLQ pain), Crohns, mesenteric ischemia or infarct, ovarian tumor, PID, uterine perforation.
Most likely diagnosis: diverticulitis.
Investigations: abdominal-pelvic CT (ultrasound if CT unavailable), stool for occult blood, urinalysis, cervical swabs and pap, CBC, lytes,
urea, creatinine, INR/PTT, glucose, for possible preop: ECG, CXR, type and cross 4 units.
45. 24 year old female with left lower quadrant abdominal pain who has an IUD. Perform physical exam. Findings: signs of
peritoneal irritation. Q: Give a differential diagnosis, order investigations.

Physical exam for abdominal pain: see question #29 and #43.
Differential diagnosis: uterine perforation by IUD, pelvic inflammatory disease (PID more common with IUD), ovarian cyst with torsion or
rupture, tubo-ovarian abscess, ectopic pregnancy (also more common with IUD), gastroenteritis, inflammatory bowel disease, intestinal
obstruction due to volvulus or IBD, appendicitis.
Investigations: ABX 3 views, abdominal-pelvic ultrasound, CBC, lytes, urea, creatinine, INR/PTT, glucose, hCG. Urinalysis. Stool for
occult blood. Cervical swabs for culture and pap smear. If OR is imminent order type and cross for 2 units. CXR.
46. 20 year old female at 36 weeks gestation with hypertension and proteinuria. Counsel patient on etiology and risks to fetus and
mother. Q: Outline a plan for management.
Background: Hypertension may be pregnancy induced or pre-existing hypertension can be worsened by pregnancy. Pre-eclampsia is
pregnancy-induced or worsened hypertension (systolic BP increased by 30 mmHg, and diastolic by 15mmHg over non-pregnant/T1
pressures) with renal impairment (proteinuria > +1 on dipstick or > 300 mg/24 hour collection) and/or non-dependent edema (e.g. face,
hands) onset > 20 weeks. Eclampsia is pre-eclampsia with CNS involvement, usually decreased level of consciousness and seizures. Other
end organs may be affected, particularly the liver and placenta. May progress to death through multi-organ failure. Imbalance of
thromboxane (vasoconstrictor) and prostaglandin (vasodilator) causes arteriolar constriction capillary damage protein extravasation and
hemorrhage. Both pre-eclampsia and eclampsia fall under the new heading of pregnancy-induced hypertension (PIH).
Mild PIH: no CNS involvement or criteria for sever PIH.
Severe PIH: 2 of the following:
1. BP > 160/110
2. pulmonary edema or cyanosis
3. CHF
4. proteinuria/oliguria/ Cr
5.
6.
7.
8.
9.

LFTs/RUQ or epigastric pain


visual disturbances/hyperreflexia/clonus/headache/convulsions
N&V
thrombocytopenia/microangiopathic hemolysis
IUGR

Eclampsia: grand mal seizures in woman with preeclampsia.


Counseling: You have pre-eclampsia. Define as above. Condition is common: 5% of pregnant population, more common in first
pregnancies. Cause seems to be imbalance of thromboxane (vasoconstrictor) and prostaglandin (vasodilator) which causes arteriolar
constriction capillary damage protein extravasation and hemorrhage. Severity varies. Risks for mother: end organ dysfunction
(kidneys, liver), cerebral hemorrhage (50% of deaths), left ventricular failure/pulmonary edema, loss of pregnancy due to abruption (often
with DIC), seizures, HELLP syndrome, death. Risks for fetus: malnutrition, hypoxia, incomplete maturation, which can result in low birth
weight (IUGR), prematurity, death. Overall treatment strategy is to slow progression of hypertension until the baby can be delivered.
Delivery is curative.
Management plan:
Initial exam and investigations: vitals, body weight, examine for edema, check for RUQ tenderness, reflexes, CBC, lytes, creatinine, urea,
uric acid, urinalysis with microscopy, 24 hour urine protein/creatinine clearance, LFTs, INR/PTT. Fetal HR, non-stress test, biophysical
profile (ultrasound with 5 criteria), Doppler flows. Bed rest in left lateral decubitus position (to reduce abdominal vessel compression),
normal dietary salt/protein intake. No use of diuretics/antihypertensives.
Follow up: daily BP, daily weight, daily reflexes, fetal movement counts at home (if patient lives reasonably close to a hospital and can get
transportation fast), frequent follow-up visits for blood work, urinalysis and fetal monitoring, bed rest (preferably left side). Instruct patient
on worsening signs: rapid weight gain, liver pain, visual disturbance, persistent headache, drowsiness or seizures.

Delivery: early hospital admission (at 36 weeks) for close monitoring and administration of IV MgSO4 4 g over 20 minutes, if signs of CNS
involvement are present (hyperreflexia, decreased LOC, seizure). Possible induction of early delivery or Cesarean section. Consider IV
hydralazine (first line treatment) 5-10 mg IV over 5 minutes q15-30min, second line: labetolol 20-50 mg IV q10min, third line: nifedipine 1020 mg PO q20-60min if delivery is not imminent or if delivery is imminent to decrease BP after conservative measures tried (diuretics and
salt or fluid restriction not useful and may be harmful).
47. 30 year old woman with vaginal bleeding at 30 weeks gestation. Take a history. Q: Give a differential diagnosis. Order
investigations.
History: patient ID (name, age, occupation), GTPAL, weeks of gestation. Onset of bleeding, duration, estimate quantity (number of pads
soaked), color and consistency of blood, associated fever, chills, abdominal discomfort (pain), contractions, fetal movement, lightheadedness,
last sexual intercourse (may cause spotting due to friable cervix). Problems with previous pregnancies, problems in this pregnancy, medical
visits to this point, investigations done. Associated abdominal trauma (accident or abuse), drug use (cocaine), father and mothers blood type,
medications, alcohol, smoking, past medical history, family history, review of systems.
Differential diagnosis: placenta previa (placenta covers internal os of cervix the most common cause of painless bleeding in the third
trimester), bloody show (shedding of cervical mucous plug), abruptio placenta (separation of placenta from uterine wall usually painful),
vasa previa (fetal bleed due to root vessels of umbilical cord overlying the cervical os extremely dangerous to the fetus). Other causes:
uterine rupture, coagulopathy (DIC), molar gestation, vaginal tear, vaginal infection, cervical polyp, cervicitis, cervical cancer, placenta
accreta, bleeding from bladder or bowel. (NOTE: no vaginal exam until previa has been ruled out by U/S).
Investigations: maternal vitals, CBC, INR/PTT, fibrinogen, type and cross-match if bleeding is severe, Rh status (may need Rhogam gamma
globulin to prevent formation of antibodies against fetal blood if mother is Rh negative and father is Rh positive), fetal monitor, fetal
ultrasound, maternal monitoring, IV access, pelvic speculum and manual exam with digital cervical exam (do these only after ultrasound to
rule out placental previa can cause further bleeding in previa), Apt test for fetal hemoglobin in vaginal blood, test maternal blood for
presence and amount of fetal hemoglobin (determines amount of Rhogam required to neutralize fetal blood antigenicity).
48. 31 year old female with right hand numbness and weakness. Take a history and perform focused physical exam. Q: Differential
diagnosis, investigations and treatment.
History: name, age occupation, amount of work done with hands, description of symptoms, onset, duration, chronology, time of day,
aggravating and relieving factors. Previous episodes, investigations. Ask about pain at night, difficulty turning a key or opening jars (signs
of weakness). Functional limitations. Associated injury, neck pain, numbness or weakness in other areas, visual problems, headache, nausea.
Medications, drug/alcohol use, smoking, allergies, past medical history (especially DM, hypothyroidism, rheumatoid arthritis, pregnancy),
surgical history, family history, review of systems.
Physical exam: see question #30.
Differential diagnosis: carpal tunnel compression of median nerve, cervical radiculopathy, stroke/TIA, diabetic peripheral neuropathy,
brachial plexus injury or tumor.
Investigations: nerve conduction studies.
Treatment: modify manual work, wrist splint (often worn at night), NSAIDs, local corticosteroid injections, control underlying systemic
contributors (e.g. diabetes, hypothyroidism, arthritis), surgical decompression via flexor retinaculum release (Neurosurgery or Plastic surgery
consult).
49. 53 year old female with incidental solitary lung nodule on CXR. Take a history. Findings: non-smoker, textile worker, no
symptoms. Q: CXR shows homogeneous round 2 cm opacification in right upper lobe. Give a differential diagnosis and identify the
most likely diagnosis. Order further investigations.
History: name, age, occupation, living conditions, hobbies, pets, esp. birds, cats, travel history, contact with hazardous substances (e.g.
asbestos). Positive TB skin test. History of pneumonia, TB etc. Malignant symptoms: weight loss, fatigue, change of bowel habits,

anorexia, night pain Smoking. Alcoholism. Lung symptoms: cough, sputum, shortness of breath/dyspnea, hemoptysis, wheeze, orthopnea,
PND, chest wall pain, Medications: drugs/alcohol, allergies, past medical history, family history, review of systems.
Differential diagnosis: less than 3 cm is more likely to be benign but greater than 3 cm is more likely malignant: neoplasm (45%) primary
= 70%, benign (hamartoma, lipoma) = 15%, metastatic = 10%; infection (53%) TB, histoplasmosis, coccidiomycosis; other (2%)
granuloma (scar tissue from old pneumonia, TB granuloma, histoplasmosis, silicosis, sarcoid), vascular (A-V malformation, infarct),
congenital (cyst), round pneumonia, round atelectasis, loculated effusion. Percentages are for lesion greater than 3 cm.
Most likely diagnosis: granuloma.
Investigations: old CXR for comparison (if lesion is old and unchanging, interventions are less aggressive, calcification is also associated
with benign lesions such as old granulomas), CT chest with CT guided needle biopsy, sputum for cytology and acid-fast staining (TB), TB
skin test, bronchoscopy with biopsy and washings if lesion seen, open biopsy or lobectomy.
Algorithm: solitary nodule previous CXR benign or unchanged (repeat in q3-6months for 2 years if unchanged observe, if changed at
any time continue), malignant or changed CT thorax: cancer (stage and treat), calcification (observe), no diagnosis bronchoscopy or
transthoracic needle aspiration still no diagnosis (resect for diagnosis), inflammatory (treat cause), cancer (stage and treat).
50. Mother with 6 month old child who has diarrhea. Take a history. Finding: recently switched from breast milk to milk powder
formula. Q: Give a differential diagnosis. Recommend treatment.
History: Distinguish between acute diarrhea, chronic diarrhea with or without failure to thrive. Name, age, age of onset and chronology of
diarrhea, consistency, color, quantity and frequency of diarrhea, blood in stool, concurrent illness, vomiting, fever, anorexia, difficulty
breathing, lassitude, dry mouth, eyes, low urine output, illness affecting other children in the family or adults. Feeds and feeding history (esp.
fruit juice). Growth pattern: weight loss? Recent immunization, travel, antibiotics. Medications, past medical history, allergies, birth history,
pregnancy problems, maternal illness during pregnancy, family history, review of systems.
Differential diagnosis: infection (bacteria: campylobacter, salmonella; antibiotic induced: c. diff colitis; parasitic: giardia; post infectious:
secondary lactase deficiency), intolerant of formula change (component of lactose intolerance), toddlers diarrhea (6 months to 36 months,
resolves spontaneously by age 2-4 years lots of juice overwhelms small bowel disaccharide malabsorption), overfeeding, osmotic
diarrhea due to high osmolality liquids such as juice, gastroenteritis. With weight loss consider malabsorption syndrome: celiac disease,
lactose intolerance, milk protein allergy, cystic fibrosis (with chest infections), IBD (unusual at this age), enzyme deficiencies, liver disease,
biliary atresia, thyrotoxicosis.
Treatment: try non-milk (soy based) formula. Arrange follow up.
Note: Treatment for Toddlers diarrhea reassurance, self-limiting, 4 fs fiber, normal fluid intake, 35-40% fat, discourage excess fruit
juice.
51. 50 year old male alcoholic. Vomited bright red blood 1 week ago. Perform physical exam. Q: Give differential diagnosis. What
is your treatment plan?
Physical exam: see question #29.
Differential diagnosis: esophagitis, gastritis, duodenal ulcer, peptic ulcer, Mallory-Weiss (partial thickness) tear, esophageal varices, gastric
or esophageal cancer, lung tumor, aorto-enteric fistula (rare can occur after previous aortic surgery).
Treatment plan: place on omeprazole 20 mg PO OD, patient to refrain from alcohol, discontinue NSAIDs, draw blood for INR/PTT, AST,
ALT, ALP, GGT, total bili, CBC, lytes, urea, creatinine. CXR (check for perforation air under diaphragm), refer to gastroenterology for
endoscopy. Consider admission if patient unreliable or transportation is a problem.
52. Telephone rings: hysterical mother says her child swallowed a cleaning agent. Manage over the phone. Q: What do you do after
hanging up the phone? What investigations do you order in the ER?

Over the phone: Establish calm, reassurance, obtain caller name, phone number, address. Childs age, weight, medical problems,
medications, allergies. Identify agent ingested, have caller read hazard label if possible. How much was taken and when? What is childs
present condition? Any other agents taken? Have child drink 2-3 glasses of milk if alert (works for alkali or acid). Do not induce vomiting
(exposes esophagus and pharynx to the corrosive agent again). If amount ingested was very small, may not need to come in, warn that child
may vomit. If amount unknown or significant, child may come to ER immediately. Mom may drive if calm, otherwise you will send an
ambulance. Have child lie on his side in case of vomiting.
After hanging up: send ambulance, call the poison center with the description of the agent and ask for direction as to treatment etc.
Investigations: CBC, lytes, glucose, urea, creatinine, serum osmolality, serum ketones, ABG, CXR, toxicology screen if ingested agent
unknown.

1994
53. 60 year old male with 20 minutes chest pain. Diaphoretic, ECG shows 2 mm ST elevation in V1, 2, 3. Manage.
History and Physical: see question #13, #14 and #17.
Treatment: Raise head of bed. Give oxygen 6 L/min by mask. Monitor oxygen saturation. Order stat CBC, lytes, glucose, INR/PTT, serial
CK-MB and Troponin, ABG, CXR, ECG. Give chewable ASA 160-325 mg immediately. Secure IV access, bolus IV lasix 40 mg, push if
fluid overload is suspected, and ventolin if wheezes are heard, give sublingual nitro spray or 0.3 mg SL nitro if blood pressure is adequate and
1 mg morphine IV. Repeat nitro SL q5min x 3. May require additional morphine and nitro. Repeat CK-MB and Troponin q8h x 3.
Diagnosis: severe anterior wall ischemia evolving to infarct. Required thrombolytics or cardiac catheterization. Urgent Cardiology consult if
available.
54. 28 year old male arrives in the ED having fallen 6m from a scaffold. Assess.
Rescusitation: see question #6.
History: if available: Nature of fall, preceding (seizure, etc.) and subsequent events, has patient ever experienced similar symptoms before,
did patient lose consciousness, were there seizure-type phenomena, injuries during fall, duration of unconsciousness, post-ictal drowsiness,
medications and drugs, alcohol/illicit drug involvement, smoking, allergies, past medical history, family history, review of systems.
55. 22 month old female child brought to emergency by her mother with fractured left humerus. Two previous fractures in past 3
months, bruises seen on forehead. Manage.
Warning signs of child abuse: explanation doesnt match injury, delay in seeking treatment, recent family crisis, injuries of varied
ages/recurrent/multiple injuries, distinctive marks, atypical pattern of injury, unrealistic expectations of child behavior by caregivers.
Risk factors for child abuse: environmental (social isolation, poverty, domestic violence), caregiver (substance abuse, jealousy between
boyfriend and father, parents abused themselves, personality/character disorder or mental illness, poor social and vocational skills/below
average intelligence), child factors (disability, difficult child i.e. temperament, premature).
History: how did the injury happen? Who was looking after the child when it happened? Who are the childs care givers, and who lives in
the house or comes in contact with the child. How did the child get the bruises? What happened with the other fractures? Any other injuries
in the past? Is the child accident prone or difficult to handle? What is the childs personality: open vs. withdrawn. Are there other children in
the house? Have they had broken bones or other injuries? Was this child a planned pregnancy, problems with pregnancy, birth history.
Developmental milestones as in question #9. Where does the child live (isolation), income level of parents, problems with the law,
alcoholism, drug use, smoking by caregivers or other adults in the home, what is the typical response of caregivers when the child cries or
misbehaves? Were the caregivers abused as children? Is there spousal abuse, sexual abuse or incest? Has the Childrens Aid Society been
involved with this child or other children? Interview relatives, friends. Childs medical history, medications, allergies, review of systems.
Physical exam: observe childs behavior, constant watchfulness associated with child abuse. Inspect for malnutrition, bruises, scars, burns,
especially on the flexor surfaces. Inspect oral cavity, perineum, anus, genitalia. Ophthalmoscopy for retinal hemorrhages (shaken baby
syndrome). Evaluate for development, neurological exam for possible brain injury.
Investigations: X-rays for old fractures, if records not available, CBC, lytes, urea, creatinine, INR/PTT, albumin (malnutrition).
Treatment: admit for the childs safety and investigations, consult child psychiatrist and pediatric orthopedic surgeon. Obligated to report
suspicion of child abuse to Childrens Aid Society. Family therapy, frequent follow up to monitor development.
56. 35 year old male. 1 week hyperactivity, histrionic, spending spree, bizarre behavior. Take a history.

Differential: Causes of one week of bizarre behavior: manic episode (bipolar mood disorder), depression, drug-induced (steroids,
amphetamines, alcohol), organic (hypothyroidism, frontal lobe tumor, MS, dementia), schizophrenia.
History: Description of symptoms: onset, duration, chronology, aggravating and relieving factors (drug use). Is patient a danger to himself or
others (suicidal or homicidal)? Does patient have alternating up and down periods, how long do these last? How frequent? Are they cycling
faster than before? Ask about mood, sleep, interest, guilt, energy, concentration, appetite/weight, psychomotor, suicide/morbid ideation.
Paranoia, ideas of reference (thought broadcasting; special messages, mind reading), special powers, magical thinking, secret identity, voices,
visual or tactile hallucinations. Current life events, stress, relationship problem, bereavement. Previous psychiatric problems, family history
or psychiatric disorders, substance abuse, relationship problems, problems at work. Work and relationship histories. Ask about
hypothyroidism, adrenal dysfunction, hypercalcemia, mononucleosis. Medications, drugs, alcohol, smoking, allergies, past medical history
including psychiatric history and history of abuse. Family history, review of systems.
Manic episode: Expansive, elevated or irritable mood x 1 week with 3 of following: GSTPAID grandiosity (or inflated self esteem), sleep
(less need for), talkative, pleasurable activities (with painful consequences), activity increased (goal directed or psychomotor), ideas (flight
of), distractibility. Not mixed episode. Severe enough to cause psychotic features/impaired social/occupational functioning. Not substance
abuse or GMC.
Diagnosis of major depression: see question #4.
Mental status: see question #4.
57. 22 year old male. Hears voices. Take a history.
History as in question #56 and #4 with special attention to the chronicity of symptoms, and work, school, and relationship histories.
DSM IV Criteria for the diagnosis of schizophrenia:
A: 2 or more of the following characteristic symptoms occurring for a significant portion of a 1 month period: delusions, hallucinations,
disorganized speech, catatonic or grossly disorganized behavior, negative symptoms (flat affect, alogia poverty of speech, avolition,
anhedonia, apathy, affectional impairment). Only 1 symptom if delusions are bizarre or hallucinations consist of a voice keeping running
commentary or 2 voices conversing.
B: Social or occupational dysfunction.
C: Continuous sign of some disturbance for at least 6 months including the month of more severe disturbance in A.
D: Schizoaffective or mood disorder excluded.
E: Substance abuse or general medical disorder excluded.
F: If there is a history of autistic disorder or pervasive developmental disorder, then the diagnosis of schizophrenia is made only if delusions
or hallucinations are prominent for at least 1 month.
Note that schizophrenic symptoms < 1 month = brief psychotic disorder, < 6 months = schizophreniform disorder, > 6 months =
schizophrenia.
58. 16 year old male with three episodes of sudden loss of awareness lasting < 1 minute, wants information on epilepsy. Counsel.
History: see question #3.
Counseling: Cause of seizures disturbed electrical activity in the brain, often with a tiny focus of abnormal tissue from previous infection,
trauma (including birth trauma) or inherited. About 2% of the population have epilepsy. Your seizures appear to be absence or petit-mal,
which start in young people. One third of cases resolve spontaneously with age. Seizures do not damage the brain unless they are prolonged
(> 30 minutes) and absence seizures are not associated with decreased intelligence or learning ability. Most people with this type of epilepsy
are well controlled on medication and have no limitations in their activities, careers or relationships.
Further counseling and treatment: see question #3.
59. Young man in the emergency department with a stab wound to the belly. Manage.
See question #6.

60. 18 year old female wants oral contraceptive. Manage.


See question #2.
61. 35 year old woman feels depressed. Manage.
See question #4.
62. 5 year old girl with pain on urination. Mother concerned about UTI and sexual abuse. Order investigations and counsel.
Algorithm: Possible UTI Is immediate antimicrobial therapy indicated? (e.g. infant, toxic, dehydrated) if yes (urine specimen for
culture via SPA/catheter initiate Abx therapy [TMP/SMX or cephalosporins i.e. cefixime, cefprozil]: consider hospital if toxic, vomiting
and cannot take PO meds or if < 6 months); if no (U/A on specimen bag U/A positive for LE, nitrites, WBC? no? unlikely UTI if
no symptoms yes? Obtain urine for culture SPA/catheter) is the culture positive? no? (no UTI) yes? (7-14 days of Abx;
prophylaxis [TMP/SMX or nitrofurantoin] until imaging completed) clinical response in 48 hrs? no? (immediate urinary tract U/S)
yes? (U/S as soon as convenient) ultimately VCUG as soon as convenient.
Investigations: midstream clean catch voided urine specimen (or bag urine). May need to catheterize or aspirate suprapubically (SPA) to
obtain a good specimen. Urine dipstick. Microscopy, culture and sensitivity most common pathogen is E. coli serotypes from bowel flora
others include Klebsiella, Proteus, enterococci, S. saprophyticus. If child appears systemically ill, take blood for cultures, CBC, urea,
creatinine, lytes. Renal ultrasound for major malformations and voiding cystourethrogram (VCUG) should be done in all children 2 months
to 2 years old with UTI. Radiological investigations may be postponed until the second UTI in girls over 2 years old due to higher rate of
benign UTI. Postpone the VCUG 3-6 weeks to allow normalization of flow after UTI.
Counseling: Ask why the mother is concerned about sexual abuse. UTI alone is not a good indicator. Explain that, in girls UTIs are
common because of short urethra and proximity to anal area. Describe front to back wiping after urination, and general hygiene. Give
prescription for TMP/SMX (Septra) 2/10 mg/kg/d PO OD or Nitrofurantoin 2 mg/kg/d PO OD x 7-10 days if urine dip is positive for white
cells and patient not allergic. Arrange follow up in > 2 weeks for re-culture of urine. Explain need for ultrasound and VCUG to rule out flow
abnormalities which may threaten kidney function (this may be postponed until second UTI in girls over the age of 2 due to increased
incidence of benign UTI), arrange these. Consider hospitalization for pyelonephritis, rehydration or child < 6 months old.
63. Elderly lady on Digoxin and Lasix with syncopal attacks. Take a history. Q: Give a differential diagnosis. What investigations
would you order?
History: onset, duration, chronology, description of events. Have the patients episodes been witnessed? Does the patient lose
consciousness, are there warning signs of post-ictal symptoms, can the patient prevent an episode by sitting down or other means. If
dizziness is a feature, is this light-headedness or true vertigo (vertigo means that the patient senses actual movement of either the room or
themselves). What is the patient doing when these episodes occur? Are muscle jerks a feature? Associated fatigue, weakness,
nausea/vomiting, chest pain, shortness of breath, palpitations, focal neurological symptoms. Past medical history (e.g. diabetes, heart
disease), medications, drug use, alcohol, smoking, allergies, family history, review of systems.
Differential diagnosis: Medication induced bradycardia (Digoxin), hypovolemia (Lasix). Cardiovascular arrhythmia, valvular disease,
subclavian steal. Metabolic: hypoglycemia. Central nervous system: seizures (e.g. narcolepsy, tumor), stroke/TIA, cervical spondylosis,
anxiety with hyperventilation, middle ear (benign positional vertigo, acoustic neuroma, Menieres disease). Autonomic: vagal, orthostatic
hypotension. Infection in elderly can present in many ways: UTI etc.
Digoxin overdose: anorexia, nausea, vomiting, bradycardia, visual effects: yellow, green or white halo around objects, decreased level of
consciousness, abdominal pain and diarrhea. ECG shows junctional tachycardia, PVCs, AV block, and sometimes PSVT.
Physical exam should include: vitals, orthostatic BP, check for signs of dehydration (thirst, mucous membrane moistness, HR, urine output,
skin turgor, BP), cardiopulmonary exam (see question #13), neurologic exam (see question #5) and mini-mental status exam (see question
#4).

Investigations: Digoxin level, CBC, lytes, urea, creatinine, INR/PTT, glucose, ECG, 24 hour Holter monitor, echocardiogram, EEG, CT
head, carotid Doppler.
64. 30 year old male, married with 2 children. Brought in by police for violent and dangerous behavior. Take a history. Q: Would
you admit this patient? What are the criteria for a Form 1?
History: Attempt to determine whether patient is sad (depressed), bad (antisocial, reaction to stressful or frustrating events, poor anger
management), or mad (mania, schizophrenia). Is the episode related to drugs of abuse or organic (brain tumor, metabolic disturbance)?
Cover history for depression (see question #4), mania (see question #4) and schizophrenia (see question #57) with mental status exam (see
question #4) and mini mental (see question #4).
Criteria for admission: patient requires observation or medication in a controlled, safe setting for diagnosis, patient appears to be a danger to
himself or others, environment at home unsuitable for the patient at this time, patient requires medical work up for organic causes, patient in
need of detoxification.
Criteria for a Form 1: Forcible admission for assessment without right to appeal, maximum 72 hours, can be administered by any licensed
physician who has seen the patient within a week, both criteria must be met:
1. Patient appears to be danger to himself/herself or others.
2. Patient appears to be currently suffering from a mental illness.
65. 65 year old male with dysphagia. Take a history. Q: Differential diagnosis and investigations.
History: see question #10.
Investigations: Barium swallow (liquid and with marshmallow for transfer), endoscopy with biopsy, CT chest, esophageal manometry, 24
hour pH reflux study.
66. 65 year old male outpatient with shortness of breath, cough, sputum. Take a history and perform a physical exam. Findings:
Lobar consolidation with yellow-green sputum. Q: Given a diagnosis of pneumonia, recommend treatment.
History: Name, age, occupation, travel history, pets. Cough (acute, chronic, worse in any position/season/night, anything relieve), sputum
(what color, quantity, frequency, quality), hemoptysis (quantity, frequency, quality e.g. blood tinged/clots), dyspnea (constant, duration, onset,
frequency, severity, exercise tolerance, triggers, alleviating), fever, chills, malaise, fatigue, increase in asthmatic symptoms (wheeze, cough),
preceding viral illness. Onset, chronology of symptoms, positional factors (orthopnea), chest pain, ankle swelling. History of COPD?
Medications, compliance with meds (observe use of puffers), drugs of abuse (alcohol), smoking, allergies, past medical history, family
history, review of systems.
Physical Exam: Cardiopulmonary exam as in question #13 and #24.
Treatment: for diagnosis of community-acquired pneumonia, admit if patient is systemically ill (may have septicemia), if the patient is
debilitated or hypoxia is a feature (send blood cultures and give oxygen). Start IV cefuroxime 750 mg IV q8h. Switch to a more specific oral
antibiotic when culture results become available. For outpatient therapy: Clavulin (amoxicillin + clavulanate) 500/125 mg PO BID. Follow
up in 1 week. Discontinue therapy after 3 afebrile days.
Note: this therapy does not cover atypicals. Practices vary according to the treatment population:
Antibiotic therapy for pneumonia
Presentation
Community acquired, no COPD
Community acquired with COPD
Alcoholics & debilitated patients
Diabetics or hospital acquired
Therapies:

Likely Organisms
Streptococcus pneumonia (typical) or Mycoplasma pneumoniae & chlamydia (atypical)
add Haemophilus influenza
add Gram negatives, Legionella and anaerobes (in aspiration)
add Staphylococcus aureus and in very ill patients: Pseudomonas aeruginosa

1.
2.
3.
4.
5.
6.
7.
8.

IV cefuroxime 750mg IV q8h covers: Strep, Staph (unless it is MRSA), gram negatives, anaerobes. Change to more specific oral
agent when organism known.
Clavulin (amoxicillin + clavulanate) 500/125mg PO BID. Covers Strep, Staph, gram negatives and anaerobes.
Septra (trimethoprim + sulphamethoxizole) 2 tabs PO BID (or 1 DS tab BID) covers Strep, Staph and gram negatives.
Erythromycin 500mg PO/IV QID covers Strep and atypicals.
Penicillin G 1-2 million units IV q4h effectively covers Streptococcus only. Oral version is penicillin V 250-500mg PO q6h.
Piperacillin 3g IV q6h + Tobramycin 2mg/kg IV q8h is a standard therapy for Pseudomonas aeruginosa. Piperacillin also covers Strep,
gram negatives and anaerobes, while Tobramycin adds further gram negative coverage with synergy.
Clindamycin 300mg IV q8h or PO q6h covers Strep, Staph and anaerobes.
Vancomycin 1g IV q12h or 125mg PO q6h for MRSA also covers Strep and anaerobes.

Sanford Guide: Rx influenced by local prevalences.


Presumed viral
Influenza, parainfluenza,
For influenza A or B:
For influenza A:
pneumonia in adults:
adenovirus, RSV, hantavirsu
zanamivir 10mg inhaled BID rimantadine 100mg PO
cought, no sputum,
x 5d or oseltamivir 75mg PO
2x/d or amantadine
dyspnea/hypoxia,
BID x 5d. Start within 48
100mg PO 2x/d.
interstitial infiltrates
hours of symptoms onset.
Adults over age 18:
Smokers: S. pneumo, H.
Azithro 0.5gm PO x1 then
FQ with enhanced
Community acquired;
influenzae, Moraxella catarhalis.
0.25 gm/d or clarithro 500mg activity vs. S. pneumo or
non-hospitalized.
Post-viral bronchitis: S. pneumo,
PO BID or clarithro ER 1gm O Ceph 2 or AM/CL
rarely S. aureus. No co-morbidity: PO OD.
875/125mg PO BID or
Adults over age 18:
Mycoplasma, Chlamydia
doxy 100mg PO BID or
Community acquired;
pneumoniae, viral, rarely S.
Rx duration varies: rx until
telithro 800mg PO OD
hospitalized. (Note:
pneumo. Alcoholic stupor: S.
afebrile
3-5d.
x7-10d.
Non-ICU bed: [P Ceph 3 IV
In ICU: (P Ceph 3 IV +
severe S. pneumo in
pneumo, anaerobes, coliforms.
(erythro 15-20 mg/kg/d q6h
azithro 500mg IV OD) or
post-splenectomy,
Epidemic: Legionaires. Birds:
or azithro 500mg IV OD)] or
(FQ with enchanced
myeloma, lymphoma
Psittacosis. Rabbits: Tularemia.
(cefuroxime + erythro) or
activity vs. S. pneumo);
pts.)
Parturient livestock or cats:
(FQ with enhanced activity vs. add P Ceph 3 for dual
Coxiella burnetii. Airway
S. pneumo)
coverage if suspect Gmobstruction: Anaerobes.
neg enteric.
O Ceph 2: cefdinir 300mg PO q12h, cefpodoxime proxetil 200mg PO q12h, cefprozil 500mg PO q12h, cefuroxime axetil
250-500mg PO q12h.
P Ceph 3: cefotaxime 2gm q4-8h IV, ceftriaxone 1-2gm IV OD, ceftrizoxime not as active in vitro vs. S. pneumo.
67. Young female with malaise, tender lymph nodes in the neck, left upper quadrant abdominal pain. Perform a physical exam. Q:
Give a differential diagnosis.
Physical exam: see question #29 and #43. Also: Palpate for lymph nodes in the neck, supra and infra-clavicular, axillae, groin. Examine the
oral cavity and pharynx. Check for rashes.
Differential diagnosis: neoplastic: lymphoma, leukemia. Viral infection: mononucleosis, HIV, EBV. Bacterial: syphilis. Inflammatory
autoimmune disease: sarcoidosis, lupus. Liver disease with portal hypertension. Serum sickness, allergic reaction.
68. 65 year old man with ataxia, dizziness, macrocytic anemia. Take a history. Finding: poor diet. Q: Give a differential diagnosis.
What is the most likely diagnosis? What investigations would you order?
History: onset, chronology, description of symptoms. Setting in which symptoms occur. Functional limitations (driving, walking, stairs,
reaching upward). Differentiate light-headedness from true vertigo (room or self spinning). Peripheral numbness, psychiatric features: mild
depression, irritability, paranoia (seen in B12 deficiency). Weakness, eye symptoms, tremor. TIA or stoke phenomena: sudden neurologic
deficit (loss of vision, speech, motor or sensory changes). Check for heart problems, hypertension, diabetes. CAGE alcoholism screen as
below, history of syphilis, MS, hypothyroidism (decreases secretion of intrinsic factor) use of chemotherapeutic agents (interfere with DNA
synthesis). Diet, weight loss or gain, chronic diarrhea (malabsorption), abdominal pain. Symptoms of hypothyroidism. Previous gastric
surgery. Signs of intracranial hypertension (hydrocephalus): morning nausea, vomiting, headache. General signs of malignancy: anorexia,
fatigue, night sweats. Past medical history, medications, drugs/alcohol, smoking, allergies, family history, review of systems.

CAGE questionnaire: Control have you tried to cut down on your alcohol? Anger have you ever felt angry when someone suggested you
decrease your alcohol intake? Guilty have you ever felt guilty about your drinking? Eye opener do you sometimes have a drink to get
started in the morning?
Differential diagnosis: Anemia due to vitamin B12 deficiency: usually due to malabsorption (Crohns, celiac disease etc.), lack of intrinsic
factor pernicious anemia (auto-antibodies against gastric parietal cells associated with thyroid and adrenal insufficiency), post
gastrectomy, long term use of antacids, pancreatic insufficiency or malnutrition (vegan diet). Severe hypothyroidism. Due to Folate (B6)
deficiency: Poor nutrition (alcoholism, poverty, infancy found in green leafy vegetables), malabsorption, medication or drug-induced
(alcohol, anticonvulsants, antifolates e.g. MTX, BCP), increased need (pregnancy, prematurity, hemolysis, hemodialysis, psoriasis, exfoliative
dermatitis). Ataxia, dizziness: Wernickes encephalopathy, hepatic encephalopathy, inner ear problem (benign positional vertigo, acoustic
neuroma, Menieres), postural hypotension, brainstem stoke or TIA, intracranial tumor.
Most likely diagnosis: Vitamin B12/folate deficiency secondary to poor diet.
Investigations: CBC and differential with blood smear, lytes, urea, creatinine, INR/PTT, GGT, AST, ALT, ALP, serum folate, screen for
serum B12, RBC folate, serum ferritin. Schillings test: measures absorption of B12. Bone marrow Barium enema if suspect pernicious anemia
(pernicious anemia is associated with bowel cancers).
69. 42 year old man found unconscious in the street. Appears to have been struck in the head. Perform physical exam. Findings:
GCS 11, unilateral body weakness. Q: What is your differential? Evaluate C-spine film. Describe your initial treatment and
investigations.
Initial management: rescusitate as required using the ATLS/ACLS format (see question #6).
History: if available: Nature of collapse, preceding and subsequent events e.g. trauma, has patient ever experienced similar symptoms before,
did patient lose consciousness, were there seizure-type phenomena, injuries during fall, duration of unconsciousness, post-ictal drowsiness,
medications and drugs, smoking, allergies, past medical history, family history, review of systems.
Differential diagnosis: concussion, subdural bleed, epidural bleed, brain contusion, seizure or post-ictal weakness, brainstem or spinal cord
injury.
Clearing C-spines: The principle of clearing C-spines is to rule out both bony fractures and ligamentous injury, either of which can make the
spine dangerously unstable. Most emergency physicians will clear the cervical spine in the case of an alert patient who has no pain on
palpation of the dorsal spinous processes and a normal cross-table lateral C-spine X-ray. If the patient has neck pain, flexion/extension plane
films are done. These involve gently flexing, then extending the neck and taking views at each extreme. The patient must be alert enough to
warn the examiner of paresthesias in the hands or increased neck pain on movement during this procedure, which may indicate compromise
of the neural elements. Flexion/extension views may be done under fluoroscopy if the patient is not alert.
Treatment: normalize vitals, oxygen saturation, ABGs, hydrate to maintain BP, give blood if necessary, correct coagulopathy. Immobilize
cervical spine. Consider intubation (careful of the neck). Control ICP, load with dilantin 1g IV to prevent seizures, give 20% mannitol 50g
IV, rapid sequence intubate with sux, fentanyl, etomidate/propofol/ketamine, spray cords with lidocaine and hyperventilate to pCO 2 35mmHg.
Consult Neurosurgery. CT head and neck. May need MRI for delineation of brainstem or spinal cord injuries.
70. 4 year old boy with cough for 6 weeks. No improvement on antibiotics 3 weeks ago. Take a history. Q: What is your differential
diagnosis? Give the most likely diagnosis and describe a treatment plan.
History: Name, age. Prodromal illness, fever, malaise, rhinorrhea, sore throat, shortness of breath, wheeze. Is cough productive? Color of
sputum, quantity. Any chest pain? Aggravating and relieving factors. Onset of cough, chronology, time of day (night), worse with exposure
to polluted air, cold air, dust, smoke, exercise, in grassy areas, weedy areas, forests, certain rooms, in bed. Did symptoms improve with
Amoxicillin? Allergic symptoms: red eyes, itching, itch in back of throat. Family pets. Air conditioning. Type of bedclothes and pillows
(feather, synthetic, foam). Recent change in the childs environment, different bedclothes, new room, change of season. Are there smokers in
the house? Past medical history, medications, allergies, family history (asthma, allergy, CF), review of systems.

Differential diagnosis: asthma, bronchitis, bronchiolitis (upper respiratory tract infection), chronic sinusitis, rhinitis, TB, recurrent
pneumonia, collapsed lung, cystic fibrosis.
Approach to cough:
1. Productive: bronchiectasis, bronchitis, abscess, bacterial pneumonia, TB
2. Nonproductive: viral infections, interstitial lung disease, anxiety, allergy
3. Wheezy: suggests bronchospasm, asthma, allergy
4. Nocturnal: asthma, CHF, postnasal drip, GERD, or aspiration
5. Barking: epiglottal disease (croup)
6. Positional: abscess, tumor
7. Differential diagnosis:
a. Airway irritants: inhaled smoke, dusts, fumes, aspiration (gastric contents, oral secretions, foreign body), postnasal drip
b. Airway disease: URTI including postnasal drip and sinusitis, acute or chronic bronchitis, bronchiectasis, neoplasm, external
compression by node or mass lesion, asthma, COPD
c. Parenchymal disease: pneumonia, lung abscess, interstitial lung disease
d. CHF
e. Drug-induced
Investigations: CXR, CBC, lytes, INR/PTT, urea, creatinine, pulmonary function tests (> 4 years of age).
Most likely diagnosis: asthma.
Treatment: Acute: O2 to keep SpO2 > 92%. Fluids. 2-agonists: Salbutamol (Ventolin) 0.03 cc/kg in 3 cc NS q20min via mask until
improvement then q1hour as necessary. Ipratropium bromide (Atrovent) if severe: 1 cc added to each of first 3 Ventolin masks. Steroids:
prednisone 2 mg/kg in ER, then 1 mg/kg PO OD x 4 days. Chronic: Sodium cromoglycate (Intal) 2 puffs QID. Add ventolin 2 puffs PRN if
acute breathlessness with wheeze is a feature. Modify the home environment to decrease contact with common allergens: dust mites, pollen,
pet hair (especially cats). Control dust with thorough and regular cleaning. Boil bedclothes, plastic undercovers on mattress and pillows,
remove rugs, install air conditioner. Remove pets. No smoking in the house (second hand smoke is a cause of childhood asthma). Warn
parents of the symptoms of a severe asthma attack, status asthmaticus and when to come to the ER. Discuss treatment strategy, a regular antiimmune medication (sodium cromoglycate or inhaled steroid) with PRN bronchodilator.
71. A 60 year old woman with a history of atrial fibrillation and congestive heart failure returns to the office for the results of her
Digoxin level, which is subtherapeutic. Take a history and counsel. Q: What are the effects of a Digoxin overdose?
History: Name, age, occupation. Ask about symptoms of atrial fibrillation: sudden fatigue, palpitations, general weakness, light-headedness,
CNS embolic phenomena (TIA, stroke). Symptoms of Digoxin overdose which the patient may have had in the past: anorexia, nausea,
vomiting, bradycardia, visual effects: yellow, green or white halo around objects, decreased level of consciousness. Symptoms of congestive
heart failure: ankle edema, shortness of breath, orthopnea, paroxysmal nocturnal dyspnea. Cough, wheeze, hemoptysis. Is patient taking her
Digoxin regularly, if not, why? Medications and whether patient is taking these, drugs and alcohol, smoking, allergies, past medical history,
family history, review of systems.
Counseling: explain action of Digoxin: prevents fibrillation by slowing the heart rate while increasing its force of contraction. Helps both
atrial fibrillation and congestive heart failure. Must be taken regularly. Discuss reasons why patient may not have been taking medication.
Memory problem? Concern about the side effects? Feels she no longer needs the medication? Emphasize the need to discuss these concerns
with you before altering medications. Discuss symptoms of overdose which the patient might watch for. Consider alternative medication if
side effects are a problem. Develop a follow-up plan to monitor compliance and cardiovascular status. Arrange follow-up.
Effects of Digoxin overdose: anorexia, nausea, vomiting, bradycardia, visual effects: yellow, green or white halo around objects, decreased
level of consciousness.
72. 60 year old woman with hypercalcemia on an insurance physical. Take a history and perform a physical exam. Finding: sister
had a parathyroid adenoma. Q: Give a differential diagnosis for hypercalcemia. What investigations would you order?

History: Symptoms of hypercalcemia: Bones, Stones, psychosis-based Moans, and abdominal Groans fatigue, muscle weakness,
arthralgias, renal colic due to nephrolithiasis, emotional lability (can progress to psychosis and coma), bone pain, abdominal pain, nausea,
vomiting, constipation, ileus, polyuria, polydipsia, nocturia. Onset, duration of these. General malignancy symptoms: weight loss, night
sweats, fatigue. Orthostatic hypotension (Addisons). Heat intolerance, hyperactivity (hyperthyroid). Diet, especially amount of milk and
use of calcium supplements and antacids. Medications, drugs and alcohol, smoking, allergies, past medical history (especially heart burn,
reflux, gastritis, peptic ulcer), family history (especially of multiple endocrine neoplasia MEN), review of systems.
Physical exam: Trousseaus sign (inflate BP cuff, leave on 1-2 minutes, distal arm goes into titanic flexion, indicates hypercalcemia). Inspect
for signs of Addisons: bronze skin tone, orthostatic hypotension, or of Cushings: moon facies, striae, buffalo hump. Chest: palpate sternum
and ribs for bone pain, examine breasts for signs of malignancy: dimpling, mass. General cardiopulmonary exam. Abdominal exam, palpate
liver carefully for masses, percuss kidneys for pain. Rectal: test stool for occult blood. Examine long bones for straightness and tenderness
(Pagets disease of bone).
Differential diagnosis: parathyroid adenoma (hyperparathyroidism) due to solitary adenoma, hyperplasia, carcinoma, inherited MEN
(multiple endocrine neoplasia may also have pituitary adenoma causing Addisons or Cushings), malignancy (myeloma, lung, breast,
squamous in any site), high bone turnover (Pagets disease of bone, hyperthyroidism, Vit A excess), vitamin D in pharmacologic doses, milkalkali syndrome (large ingestion of milk and alkali, usually for gastric hyperacidity), aluminum intoxication, drugs (thiazides, lithium,
CaCO3), familial hypocalciuric hypercalcemia.
Investigations: CBC, lytes (including Ca & Ca++, Mg, PO4), urea, creatinine, albumin, AST, ALT, ALP, GGT, INR/PTT, serum cortisol,
serum PTH, TSH, serum protein electrophoresis (for monoclonal gammopathy of myeloma). Plane X-rays of tender or malformed bones,
including skull (see salt and pepper lesions). CT head, thyroid, adrenals.

1993
73. 50 year old woman states that the Russians are leaking radiation into her house. Take a history with mental status exam.
History should focus on depression, mania and schizophrenia. Mental status exam. See questions #4 and #57.
74. Obese patient wants help with weight loss. Counsel.
History: Name, age, occupation, weight history, note ups and downs if present, past attempts to lose weight, successes, obstacles, goals.
Dietary habits: frequency of meals, snacking, eating at night, foods eaten, binge eating, guilt about food, hoarding, concealing eating from
others. Estimate daily caloric intake. Alcohol intake. Smoking, drug use. Exercise history. Overweight relatives? Diseases associated with
weight gain: hypothyroidism, DM type II, Cushings, major depression, anxiety disorder, some medications (TCA, steroids, OCP). Problems
associated with overweight: gout, sleep apnea, cholecystitis, back pain, cardiovascular disease, hemorrhoids, lower limb joint pain and
osteoarthritis. Why is patient seeking medical help for this now? Crisis in patients life, stress, anxiety? Assess patients self-image: does
patient feel underweight, overweight or normal? Does patient feel that weight interferes with health? With activities? Screen for eating
disorders. Medications, drugs and alcohol, allergies, past medical history, family history, review of systems (include sleep habits, apneic
spells, OTC medications).
Physical exam: vitals, calculate BMI (body mass index) = weight (kg)/height2 (m2). Ideal is 20-25 for a male, but varies depending on frame.
25-30: overweight; 30-35: obese; > 40: morbidly obese. Use standard height-weight tables for males and females to gauge percent overweight
(>20% = obese). Direct physical to pertinent positives from history. Inspect for Cushingoid features: moon facies, buffalo hump, striae,
visual field defects. Fat distribution: centripetal fat associated with greater heart disease and diabetes risk. Fundoscopic exam for
retinopathy. Cardiopulmonary exam, abdominal exam (not liver size). Check for signs of hypothyroidism: hypothermia, goiter, dull facial
expression, lid lag absent, lids droop, periorbital swelling, hair is sparse, coarse and dry, skin is also coarse, dry scaly and thick. Patients are
forgetful, show change in personality which may progress to psychosis, deep tendon reflexes show slow return phase (brisk contraction and
slow relaxation), leukonychia (whitened nails), orange palms and soles due to carotene deposition, bradycardia, pericardial effusion, pleural
effusions, myxedema, non-pitting edema, carpal and tarsal tunnel nerve compression due to myxedema, causing paresthesias and numbness
in the hands and feet.
Counseling: Motivation: how would being at ideal body weight improve the patients life? Emphasize health, lifestyle, self esteem,
relationship benefits. Discuss nutrition-related problems: heart disease, obesity, hypertension, osteoporosis, anemia, dental decay, cancer,
gastrointestinal disorders, respiratory compromise, high lipids, diabetes, sleep apnea, osteoarthritis. Discuss diets tried and why these failed.
Fad diets involve unusual or extreme eating patterns and are not designed to be maintained for a lifetime therefore these should be
discouraged. Weight loss agent Ponderal no longer available. SSRIs such as Paxil may assist with weight loss, unfortunately, when the drug
is discontinued, most people regain weight. Explain that the brain has a satiety set point which can be reset over time with reduction in
caloric intake. Warn that the bodys ability to determine caloric content is very good, and will not be fooled by so-called diet products.
Recommend a balanced diet consisting of ordinary foods, with three distinct meals per day of small size. No eating at night and be careful of
snacks. Inform patient that he will be hungry for at least the first two weeks of reduced intake. Suggest visualization techniques, redirection
of interests, and to think of hunger as a sign of positive progress on weight loss. Group support can be beneficial too: Weight watchers,
overeaters anonymous etc. behavior modification.
Dietary recommendations: reduce fat to 20% of caloric intake. Ideal caloric intake can be estimated at 8-10 Cal/lbs (ideal weight) for
females and 10-12 Cal/lbs (ideal weight) in males. Emphasize that caloric intake is more important for weight loss than food composition
(i.e. excessive calories lead to weight gain even if they are non-fat).
Exercise recommendations: sudden intense exercise in sedentary patient unwise. Exercise tends to provide a good excuse for overeating
and may be dangerous in an obese patient. Recommend mild daily exercise such as 1 hour walking per day. More vigorous exercise can be
initiated when weight is lost.
Arrange regular follow-up for body mass monitoring and counseling.
75. Alcoholic smoker with cough, sputum, shortness of breath. History and physical exam. Order investigations.

See question #66.


76. 70 year old man with dysphagia. Manage.
See question #65.
77. 64 year old woman with resting tremor. Perform focused physical exam.
Hallmarks of Parkinsons disease: TRAP: Tremor (rest, pill-rolling, 4-7 Hz, can be suppressed by voluntary movement), Rigidity (lead pipe
and cog-wheeling), Akinesia/bradykinesia, Postural instability (festinating gait, retropulsion, falls). Remember to explain to the examiner
what signs of Parkinsons you are looking for at each step in the exam. Other features include: mask-like face, lack of blinking,
blepharoclonus (fluttering of closed eyelids), dysphagia, drooling, hypophonia, micrographia, gait: start hesitation, small shuffling steps, loss
of arm swing, subcortical dementia (apathy, forgetful, poor ability to use knowledge).
Physical exam: vitals, observe patient at rest: look for pill-rolling tremor in the upper limb which is worst at rest, may also have head tremor
(titubation), stooped posture, open-mouthed, mask-like face, generally hypokinetic with decreased blinking and drooling. Mini mental status:
dementia associated with Parkinsons (50% of patients), may find poor short term memory, poor concentration, abstraction, micrographia.
Depression may also be a feature, screen for depression: MSIGECAPS. Cranial nerves, body power, pronator drift, deep tendon reflexes and
Babinski are normal in Parkinsons. Tone: bilateral lead pipe (constant) rigidity with possible cog-wheeling due to tremor superimposed on
passive motion. Test elbow, forearm rotation and knee by applying rapid passive motion while feeling the muscle tendon. Cerebellar testing:
finger-nose and heel-shin tests show improvement of tremor with intention (i.e. a resting tremor rather than an intention tremor), rapid
alternating movements are poor bilaterally in Parkinsons and Rhomberg is positive due to postural instability. The Parkinsonian gait is
unsteady and shuffling with small steps, decreased arm swinging, and a tendency to fall forward or backward. Patients may try to increase
forward speed to keep from falling (festination).
Mini-Mental Status: orientation to time/place (5 pts, year, season, month, day, day of week; 5 pts, country, province, city, hospital, floor),
memory (3 pts, honesty, tulip, black; 3 pts, delayed recall), attention/concentration (5 pts, serial 7s, WORLD backwards), language tests:
comprehension (3 pts, three point command), reading (1 pt, close your eyes), writing (1 pt, complete sentence), repetition (1 pt, no ifs,
ands or buts), naming (2 pts, watch, pen), spatial ability (1 pt, intersecting pentagons)
78. 12 year old female with fever, photophobia, neck stiffness. Manage.
History: name, age, onset of symptoms, duration, increasing or decreasing in severity, fever, nausea, vomiting, photophobia, phonophobia,
neck stiffness, headache, rash. Immunocompromised? (HIV, asplenia, prematurity), parameningeal infection? (sinusitis, mastoiditis),
environmental risk factors? (day-care centers, household contact, travel to endemic regions). Medications or other interventions tried?
History, severity, chronology of migraines. Premonitory visual disturbance? Recent neurosurgery, head trauma, other illness? Contacts with
meningitis at school or work. Medications, drugs/alcohol, allergies, past medical history, family history, review of systems.
Physical exam: vitals, GCS, note general appearance of patient if patient is very ill, a resuscitation approach (ATLS/ACLS) may be
appropriate, orientation. Inspect for meningococcemial rash. Cranial nerves: pupillary reflexes, note photophobia if present, extraocular
muscle movement, check for double vision, visual fields, facial sensation and movement, gross hearing, sternocleidomastoid and trapezius
power. Tone: passive rapid movement at elbows, rotation of forearms and flexion/extension of knees. Pronator drift. Cerebellar testing:
finger-nose, heel-shin, rapid alternating movements of forearms. Power of deltoids, triceps, biceps, wrist extension and flexion, finger
abduction and adduction, psoas, quadriceps, hamstrings, ankle dorsiflexion and plantar flexion. Deep tendon reflexes at triceps, biceps,
brachioradialis, knee, ankle, Babinski. Light touch, pin prick over limbs and body, vibration sense at joints. Signs of meningismus: Kernigs:
pain in the neck on extension of the knee with the hip in 90 degrees of flexion. Brudzinskis: pain on passive flexion of the neck. Nuchal
rigidity. Opisthotonos: spasm in which head and heels are bent backward and body bowed forward.
Investigations: CBC, lytes, INR/PTT, urea, creatinine, glucose, blood cultures, ABG, CT head followed by lumbar puncture if history and
physical are suspicious for raised intracranial pressure (lumbar puncture may, rarely, precipitate brain herniation in the presence of raised
ICP).
Treatment: isolation, initiate IV antibiotics immediately (before CT and LP) if the clinical picture is suspicious for meningitis. Cefotaxime 2
g IV q4h + ampicillin 50 mg/kg IV q6h. Consult ICU. Consider intubation and intensive management of ICP. Monitor glucose, acid-base &
volume status and manage as needed. Steroids for Hib meningitis (give early). Anticonvulsants for seizures. Report to public health.

79. 17 year old female with chronic diarrhea. Take a history.


Background: Causes of chronic diarrhea in a 17 year old: Crohns, ulcerative colitis, irritable bowel, malabsorption (Celiac disease, tropical
sprue), lactose intolerance, intestinal infection (C. difficile, giardia, amoebiasis), pancreatic dysfunction, unusual but may be laxative abuse.
History: onset of diarrhea, duration, consistency and color of stools, do they float? Is there blood or mucous? Frequency of BMs/day, weight
loss, appetite. Dietary history. Is diarrhea worse with milk (lactose intolerance tends to produce explosive diarrhea after milk ingestion
hereditary)? Laxative use. Use of antibiotics in the past 6 weeks. Travel history. Fever, nausea, vomiting, infectious illnesses. Associated
abdominal pain, fatigue, uveitis, mouth or anal ulcers, ankylosing spondylitis, sacroiliitis, renal problems (due to malabsorption), arthritis
(these are associated with Crohns). Malnutrition signs/symptoms: lassitude, weakness, hair falling out, skin rash, easy bruising, weight loss,
anemia, neurologic findings (carpal and tarsal nerve compression, confusion, emotional lability, loss of corticospinal vibration and position
sense), glossitis. Medications, drugs and alcohol, smoking (decreases risk and symptoms of inflammatory bowel disease), past medical
history (IBD, abdominal surgery), family history, review of systems.
Classification of Chronic Diarrhea
Type
Inflammatory
Ulcerative colitis
Crohns disease
Malignancy: lymphoma,
adenocarcinoma
Osmotic
Ingestion
Lactose intolerance
Medications, laxatives
Maldigestion and Malabsorption
Pancreatic insufficiency
Bile salt deficiency
Celiac Sprue
Whipples disease
Bowel resection
Secretory
Bacterial enterotoxins
Secretagogues VIP, gastrin,
carcinoid
Functional
IBS

Characteristics
Fever, hematochezia, abdominal pain; usually weight loss
with carcinoma

Stool volume decreases with fasting


Increased stool osmotic gap:
fecal [Na+] + [K+] < serum osmolality 25
mmol/L
Weight loss, fecal fat > 7-10 g/24h stool collection
Anemia, hypoalbuminemia

Large volume (> 1 L/d); little change with fasting


Normal stool osmotic gap:
Secretory: fecal [Na+] + [K+] = serum osmolality

80. Young mother with black eye, hit by her boyfriend. Manage.
Warning signs of domestic violence: obsessive need to control victim by controlling money, restrictions on going out, not allowed to see
certain people. The Spanish Inquisition where were you, who were you with, what did you do? Social isolation. Threats. Verbal abuse
aimed at decreasing self-esteem of victim. Cycle of violence followed by remorse, then increased violence.
Risk factors for domestic violence: social isolation, poverty, substance abuse, partners parents had abuse relationship, personality/character
disorder or mental illness.
History: Describe violent episode, what triggered it? Were objects used as weapons? Injuries? Was the boyfriend remorseful afterward?
History of previous episodes of violence or loss of temper by boyfriend. What is patients response? Has patient been in an abuse
relationship before? Were the patients parents in an abusive relationship? Is boyfriend controlling, does he restrict her activities, question
her excessively after she has been out, engage in verbal abuse or threats? Is the violence increasing in severity? Are there children in the
house? Who are the biological parents? Ask about violence to the children, sexual abuse. Does the patient or her partner abuse alcohol or
other drugs? Is money a problem? Is the boyfriend willing to seek help?

Counsel: Explain that the boyfriend hitting the patient is a criminal assault and an example of domestic violence. Domestic violence tends to
increase over time unless the victim leaves, or the abuser and couple seek therapy. Very often, women dont leave their abusive partner until
they are seriously hurt or before they are killed. Domestic violence between adult partners tends to be reflected in future behavior of children
who are exposed to it and there is a risk of violence to the children. Child abuse is a criminal act and if suspected, is reportable to police by
law. Spousal abuse is also a criminal act but is not reportable by law. Recommend that the patient not return to the abuser if there is risk to her
safety (e.g. not the first assault, abuser not remorseful). If the patient does return, an exit plan should be developed to ensure patient safety.
Document all evidence of abuse (pictures, sketches) and related visits; quote patient directly in chart.
Alternatively, the patient can contact the police to obtain a restraining order on the abuser. Develop a plan with the patient to seek alternate
living arrangements (womens abuse shelter), enlist the help of patients support structure (friends, other family members), contact the police
(patient should be informed that, if contacted, the police will lay charges whether the patient wants to or not). Counsel patient on how to
enter into controlled, safe, contact with the abuser to discuss possible therapy for anger management and controlling behaviors, with therapy
as a couple for relationship problems. Social worker referral. Arrange follow up.
81. Patient kicked by horse, now hypotensive in emergency department. Manage.
Rescusitation: The ACLS and ATLS format is useful: see question #6.
History: if available: Where was the patient kicked, when, was there mechanism of action for other injury, i.e. collapse/fall, preceding and
subsequent events, did patient lose consciousness, duration of unconsciousness, did the patient fall, injuries during fall, medications and
drugs, smoking, allergies, past medical history, family history, review of systems.
Management: For unstable blood pressure, blood loss is the most likely cause (CBC may be normal with a large acute blood loss), type and
cross for 4-6 units depending on estimated severity and hang blood as soon as possible. Is the patients abdominal wound the first priority?
Examine for head injury, other injuries. Check for abnormalities on abdominal exam suggestive of splenic rupture. With clear surgical
abdomen (rigidity, rebound, absence of bowel sounds), consult general surgery and prepare patient for immediate OR. If less severe
abdominal bleeding is suspected, consider CT abdomen or diagnostic peritoneal lavage.
82. Suturing station. Suture laceration on a rubber forearm. Choose suture type. Is a tetanus booster required?
Suturing station: past years have included a point for introducing yourself to the rubber forearm.
History: Name, age, occupation, mechanism of injury, environment in which injury occurred. How long since the injury. Any distal loss of
sensation, motor power? Other injuries. Past medical history, medications, drug/alcohol use, smoking, allergies, review of systems.
Choice of suture: use non-absorbable monofilament such as 3-0 Prolene or Ethylon. Braided sutures can harbor bacteria and absorbables
cause more inflammatory reaction in the skin. Given a choice between 3-0 silk (a braided non-absorbable) and chromic gut (a braided
absorbable), choose silk.
Technique: for small wound use interrupted sutures starting at the middle of the wound. Anesthetize with lidocaine without epinephrine,
cleanse and irrigate wound beforehand and drape, glove and observe sterile technique.
Tetanus immunization status: Dose Td 0.5 mL IM. Usual schedule of immunizations for tetanus (prepared as diphtheria-tetanus toxoid plus
pertussis vaccine, i.e. DTP) is 2, 4, 6, 18 mo., 4-6 years, Td (diphtheria-tetanus toxoid) at 14-16 years and repeat q10y.
Last tetanus immunization treatment
0-5 years ago
none
5-10 years ago
boost (Td)
>10 years ago
boost and give immunoglobulin (passive)
uncertain
boost and give immunoglobulin (passive)
Follow up: Warn of signs of wound infection. Remove sutures in 7 days (5 days on the face to minimize scarring, the face heals faster and is
less likely to become infected due to better blood supply). Recommend Tylenol plain if pain is a problem.

83. Female, 7 weeks pregnant with lower abdominal pain and vaginal bleeding. Take a history. Finding: previous spontaneous
abortion at 6 weeks gestation. Q: Give a differential diagnosis, which is most likely? What three findings on vaginal examination
would confirm this diagnosis?
Definitions:
1. Threatened abortion = any uterine bleeding or cramping in the first 20 weeks of gestation.
2. Inevitable abortion = intolerable pain or bleeding x 1 week, cervix open. Life-threatening to the mother.
3. Incomplete abortion = membranes ruptured, part of products of conception passed, cervix open.
4. Complete abortion = uterus empty, bleeding and complete sac and placenta passed, cervix open.
5. Missed abortion = fetal death and retention of products; presents as pregnancy not progressing, cervix closed.
6. Habitual = 3 or more consecutive spontaneous abortions.
20-30% of pregnancies have uterine bleeding or cramping in the first 20 weeks. Half of these abort. Most spontaneous abortions are
associated with abnormal fetus.
History: patient ID (name, age, occupation), GTPAL, weeks of gestation. Onset of bleeding, duration, estimate quantity (number of pads
soaked), color and consistency of blood, associated fever, chills, abdominal discomfort, light headedness. Problems with previous
pregnancies, problems in this pregnancy, medical visits to this point, investigations done. Associated abdominal trauma (accident or abuse),
drug use (cocaine), father and mothers blood type, medications, alcohol, smoking, past medical history (diabetes, lupus), family history,
review of systems.
Differential diagnosis: threatened abortion, incomplete abortion, non-uterine bleeding source trauma: post-coital vs. lesion: cervical
polyp, neoplasm etc. (vaginal, cervical, vulvar), abnormal pregnancy (ectopic, molar), physiologic bleeding (due to placental development).
Most likely diagnosis: incomplete abortion. Three findings which would confirm this: ruptured membranes, products of conception passed,
cervix dilated (os open).
84. 60 year old male with hemoptysis and shortness of breath. Take a history. Findings: history of CAD, HTN, hasnt been taking
antihypertensives for six weeks. Q: describe CXR, (shows enlarged heart, upper lobe vascular redistribution, Kerley B lines,
bilateral interstitial infiltrates and bilateral small effusions). Read ECG: shows Q waves and inverted T waves in V1-4. What is the
diagnosis?
History: name, age, occupation. Onset of symptoms, duration, time of day. Has patient had these before? Smoking history. Prelude of
fatigue, ankle edema, orthopnea, paroxysmal nocturnal dyspnea, palpitations, chest pain/heaviness/tightness, pain in left arm or jaw/teeth.
History of angina, other cardiac problems. History of GI bleeds, reflux, varices, gastritis, peptic ulcer, COPD history, cough, sputum,
wheeze. History of immobilization, leg pain or swelling, previous DVT, PE. Medications has patient been taking them? Drugs, alcohol,
allergies, smoking, surgical history, family history, review of systems.
CXR: consistent with pulmonary edema and CHF.
Diagnosis: ECG: anterior wall myocardial infarct.
85. 2 year old child with 9 week history of cough, on Amoxicillin for 2 weeks. Take a history. Q: Give a differential diagnosis. What
investigations would you order?
See question #70.
86. Young female with secondary amenorrhea for 6 months. Take a history. Q: What investigations would you order. Give a
differential diagnosis. What is the most likely diagnosis, what results would confirm this diagnosis?
History: age of menarche, regularity of previous menses, flow, duration, accompanying cramps, bloating, psychic disturbance. Headache,
visual field disturbance (for sellar tumor). Signs of virilization: increased quantity and coarseness of body hair and facial hair, acne,
increased sexual drive, increased muscle bulk. Galactorrhea? Diet history: has patient lost/gained weight lately? Thyroid symptoms: energy
levels, emotional lability/depression, cold intolerance, or feels hot, jumpiness. Exercise history: is patient engaging in vigorous exercise such

as running? Sexual history: contraception, is pregnancy a possibility? Medications, drugs and alcohol use, smoking, allergies, past medical
history with surgical history, family history, review of systems.
Approach: History and physical pregnancy test TSH and Prolactin (high/low hypo/hyperthyroid; high, > 100, or symptoms of
hyperprolactinemia CT to rule out tumor) progesterone challenge (+ve withdrawal bleed anovulation; no withdrawal bleed
end-organ failure or outlet obstruction) FSH, LH (high ovarian failure; low hypothalamic dysfunction)
Investigations: hCG, TSH, serum LH, FSH, serum prolactin, serum testosterone, sex-hormone binding globulin (SHBG), DHEA-S,
progesterone trial for uterine bleeding (indicates functional endometrium).
Differential diagnosis: pregnancy, polycystic ovary syndrome (Stein-Leventhal syndrome), hypothalamic dysfunction, excessive exercise,
stress, weight loss, adrenal dysfunction (e.g. Cushings), thyroid dysfunction, prolactinoma, hypopituitarism.
Most likely diagnosis: PCOS
Confirmed by: elevated LH, low or normal FSH with well estrogenized vaginal mucosa, increased serum androgens, ovarian cysts seen on
ultrasound.
87. Mother with low birth weight baby, just delivered. Take a history. Q: On physical exam of the baby you find emaciation with
wrinkled yellow skin and yellow tears. What is the problem? Give three underlying causes for this. Give two potential problems
which may arise in the next 48 hours.
Mothers obstetrics and newborn history: see question #25.
Causes of intra-uterine growth restriction (IUGR)
1. symmetric IUGR (normal head to body size), familial, maternal gestational infections (mnemonic ToRCH), toxoplasmosis (carried
in cat feces), rubella, cytomegalovirus (CMV), herpes
2. Asymmetric IUGR (small body): placental insufficiency due to maternal malnutrition, smoking, drugs and alcohol, illness during
pregnancy (e.g. Crohns), hypertension.
3. Jaundiced, emaciated baby: hyperbilirubinemia. Three underlying causes: ABO or Rh incompatibility, neonatal liver insufficiency
(CMV) and sepsis (TORCH).
All causes of neonatal jaundice: Unconjugated = physiologic neonatal jaundice OR pathologic: hemolytic ABORh incompatibility,
neonatal sepsis, splenomegaly, hereditary spherocytosis, G6PD etc.; non-hemolytic breast milk jaundice, breakdown of
cephalohematoma, polycythemia, sepsis, Gilberts, Crigler-Najjar, hypothyroidism. Conjugated: GI obstruction in fetus (increases
enterohepatic circulation), bile duct obstruction, drug-induced and multiple other less common causes.
Two potential problems arising in the next 48 hours: kernicterus (hyperbilirubinemic seizures and brain damage: deposition of bilirubin in
the brainstem and basal ganglia leading to mental retardation, cerebral palsy, hearing loss and paralysis of upward gaze), hydrops fetalis
(generalized edema, including pulmonary, with high output heart failure).
88. A young man fell while inebriated and sustained a laceration to the right wrist. Perform a focused physical exam. Findings:
numbness on ulnar side of hand, Allens test shows no ulnar artery refill, FDS impaired in little and ring finger. Q: What four
structures were lacerated? Management?
Physical Exam:
Vascular: capillary refill, is hand pink and warm? Allens test: compress ulnar and radial arteries at the wrist, have patient open and close
hand to remove blood, then release one side the hand should flush pink due to blood supply from the side released. (Usually used to
demonstrate function of ulnar artery). Doppler probe.
Sensory: digital nerves: check two point discrimination on either side of each digit. Pin-prick sensation: Median nerve territory is the palmar
surface of the thumb, and the palmar surface and dorsal tips of the index, middle and thenar side of the ring fingers. The ulnar side of the
hand is ulnar nerve, and the dorsal surface of the thenar side of the hand is radial nerve innervated. The median nerve also innervates most

muscles of the thenar eminence, and the 1st and 2nd lumbricals. The thumb is weak in abduction at 90 degrees to the plane of the hand in
median nerve dysfunction. The ulnar portion of the palm is supplied by the cutaneous branch of the ulnar nerve, while the thenar portion is
supplied by the cutaneous branch of the median nerve.
Neuromotor examination of the Hand
Median
Sensory
Redial aspect of index finger pad
Motor Extrinsic Flex DIP of index finger (FDP)
Motor Intrinsic

Ulnar
Ulnar aspect of little finger pad
Flex DIP of little finger (FDP,
extensor carpi radialis)
Abduct index finger (first dorsal
interosseous)

Thumb to ceiling with palm up


(abductor pollicis brevis)

Radial
Dorsal webspace of thumb
Extend wrist and thumb
(extensor pollicis longus)

Motor: (dont ask patient to apply force against resistance as this may rupture a partially severed tendon test active ROM only). Median
nerve: thumb abduction. Ulnar nerve: finger spread against resistance. Radial nerve: wrist extension. FDS flexes the MCP and PIP joints of
the fingers, while FDP (flexor digitorum profundus) flexes the DIP. To test FDP function hold the MCP/PIP joints in extension and ask
patient to flex DIPs. Both the FDS and FDP can flex the entire finger, but the FDP tends to flex them all at once, while the FDS can flex
fingers in isolation. To test FDS: hold all fingers except the one you are testing in extension and ask patient to flex the remaining finger.
Tendon Examination of the Hand
MCP
Extensor tendons
Communis
Flexor tendons
Intrinsics (lumbricals)

PIP
Extensor digitalis (lateral bands)
Flexor digitorum superficialis

DIP
Intrinsics (lateral bands)
Flexor digitorum profundus

Structures superficial to the flexor retinaculum (in order from ulnar to thenar): ulnar nerve, ulnar artery, cutaneous branch of ulnar nerve,
palmaris longus tendon, cutaneous branch of median nerve.
Structures immediately deep to flexor retinaculum (ulnar to thenar): 4 flexor digitorum superficialis tendons (FDS), median nerve, palmaris
longus tendon, cutaneous branch of median nerve.
Structures lacerated: given diminished ulnar territory sensation, Allens test shows no refill from ulnar circulation, and FDS weakness in
little finger and ring finger, the following structures were included in the laceration: ulnar nerve, ulnar artery, flexor retinaculum, ulnar two
divisions of FDS.
Treatment: Clean and explore wound under local anesthetic and sterile conditions. Consult plastic surgery for microvascular repair. If at
night, may suture skin closed and arrange for patient to be seen by plastic surgeon the next day.
89. A young man sustains a head injury on falling from his bicycle. Patient has been hemodynamically stabilized. Perform a focused
neurological exam. Q: Lateral skull and lateral C-spine X-rays provided. Are they adequate? Are they normal? The patient has
continuing sanguinous discharge from his nose. What is the likely cause of this. Treatment?
Begin by assessing the level of consousness using the Glasgow Coma Scale:
Glasgow Coma Scale (GCS):

Glasgow Coma Scale


Eye Opening
Spontaneous
To speech
To Pain
never

(E)
4
3
2
1

Verbal Response
Oriented and converses
Confused conversation
Inappropriate words
Incomprehensible sounds
None

(V)
5
4
3
2
1

Best Motor Response


Obeys commands
Localizes pain
Withdrawal to pain
Abnormal flexion (decorticate)
Abnormal extension (decerebrate)
Nil

(M)
6
5
4
3
2
1

NB: Standard painful stimulus is rubbing the knuckle on the sternum. For withdrawal, apply pressure on the base of the nail bed with a pen.
Decorticate posture is arm flexion with leg extension on the same side of the body, may be unilateral or bilateral. Indicates a lesion above the

brainstem. Decerebrate posture is arm and ipsilateral leg extension, may be unilateral or bilateral, indicates brainstem involvement. A GCS
of 8 or less is considered an indication for intubation because of the risk of poor protection of the airway from aspiration.
Head and Neck: inspect for lacerations and contusions, cranial nerves: extra-ocular movements, visual fields by confrontation, pupillary
reactivity, dolls eyes (careful of neck, may not be able to turn head enough to do this), and accommodation, corneal reflex and facial
sensation, palpate facial bones for stability, look in nose and ears for blood or CSF leaks, hemotympanum, facial muscle power, gross
hearing, sternocleidomastoid power and trapezius power. Check oral cavity, gag reflex, palpate dorsal cervical spines for pain and alignment,
is the trachea midline? Pronator drift, cerebellar tests: finger-nose, heel shin, rapid alternating movements (dysdiadocokinesis). Body power,
tone, sensation. Deep tendon reflexes, Babinski. Log roll patient onto back, inspect, palpate spine.
If GCS is significantly less than 15: Head and Neck: inspect for lacerations and contusions, cranial nerves: pupillary reactivity, dolls eyes
(careful of neck, may not be able to turn head enough to do this), corneal reflex, palpate facial bones for stability, look in nose and ears for
blood or CSF leaks, hemotympanum. Check oral cavity, gag reflex, palpate dorsal cervical spines for alignment, is the trachea midline?
Brainstem (breathing pattern). DRE for sphincter tone. Deep tendon reflexes, Babinski. Log roll patient onto back, inspect, palpate spine.
Differential diagnosis: concussion, subdural bleed, epidural bleed, brain contusion, seizure or post-ictal weakness, brainstem or spinal cord
injury.
Clearing C-spines: The principle of clearing C-spines is to rule out both bony fractures and ligamentous injury, either of which can make the
spine dangerously unstable. Most emergency physicians will clear the cervical spine in the case of an alert patient who has no pain on
palpation of the dorsal spinous processes and a normal cross-table lateral C-spine X-ray. If the patient has neck pain, flexion/extension plane
films are done. These involve gently flexing, then extending the neck and taking views at each extreme. The patient must be alert enough to
warn the examiner of paresthesias in the hands or increased neck pain on movement during this procedure, which may indicate compromise
of the neural elements. Flexion/extension views may be done under fluoroscopy if the patient is not alert.
Treatment: Continuous sanguinous nasal discharge after head injury is likely a leak of CSF mixed with blood. This represents a break of the
meninges and can, rarely, lead to meningitis (< 5% of cases). Prophylactic antibiotics not indicated as this selects for resistant organisms. If
meningitis results, it is usually due to less virulent organisms than in other settings. > 90% of leaks resolve spontaneously within 4 weeks. If
leak does not subside spontaneously, a surgical repair may be necessary. Consult neurosurgery. CT head and neck. May need MRI for
delineation of brainstem or spinal cord injuries.
Key Points:
1. Never do lumbar puncture in head injury.
2. All patients with head injury have C-spine injury until proven otherwise.
3. Dont blame coma on alcohol: there may also be a hematoma.
4. Low BP after head injury means injury elsewhere.
5. Must clear c-spine both radiologically AND clinically.
90. Older man with 50 RBC/HPF on routine urinalysis. Take a focused history. Q: What is the likely diagnosis? Give three other
possible diagnoses. What two investigations would you order?
History: see question #28.
Likely diagnosis: benign prostatic hyperplasia.
Differential diagnosis: transitional cell carcinoma of bladder, UTI, nephrolithiasis, hydronephrosis, prostatitis, prostate cancer, renal cell
carcinoma, essential hematuria (tends to occur in children).
Two investigations: prostate specific antigen (PSA), cystoscopy, renal, bladder and prostate ultrasound, intravenous pyelogram (IVP).
91. 20 year old female with hypertension. Perform a physical exam. Q: Give a differential diagnosis. What investigations would you
order?
See question #11.

92. 67 year old male complains of bladder distension, inability to urinate and dribbling of urine from the urethra. Take a history. Q:
What investigations would you order?
History: patient ID. Onset of symptoms, chronology, previous episodes. Associated constipation, perineal numbness, leg weakness, diabetic
neuropathy. Is patient on a new medication? Suprapubic pain, pain on urination, frank blood in the urine, color of urine, difficulty initiating
or maintaining urinary stream, renal pain, groin pain. Previous renal colic or diagnosis of nephrolithiasis? Known prostatic hypertrophy or
cancer? Diabetes? B12 deficiency? Recent surgery? History of hypercalcemia, hypertension. Malignant symptoms: night sweats, weight
loss, fatigue. Medications, drugs/alcohol, smoking, past medical history, past surgical history, family history, review of systems.
Investigations: urinalysis, urine microscopy and culture with sensitivities, cystoscopy, PSA, renal and pelvic ultrasound.

1992
93. 60 year old female feeling depressed. Complains of stomach pain. Perform focused mental status exam.
Mental status: appearance, behavior (dress, grooming, posture, gait, apparent age, physical health, body habitus, expressions, attitude cooperative?, psychomotor activity, attention, eye contact), speech (rate, rhythm/fluency, volume, tone, quantity, spontaneity, articulation),
mood (subjective emotional state in patients own words), affect (Quality euthymic, depressed, elevated, anxious; Range full, restricted;
Stability fixed, labile; Appropriateness; Intensity - flat, blunted), suicidal ideation (low, intermediate, high poor correlation between
clinical impression of suicide risk and probability of attempt), thought process (coherent, flight of ideas, tangentiality, circumstantiality,
thought blocking, neologisms, clanging, perseveration, word salad, echolalia), thought content (delusions bizarre vs. non-bizarre,
obsessions, preoccupations, phobias, recurrent themes), perceptual disturbances (illusions, hallucinations, depersonalization, derealization),
insight, cognition, judgment.
94. 16 year old girl brought to the office by a classmate for weight loss over the past six months. The classmate is worried about
anorexia nervosa. Take a history and counsel.
History: amount of weight lost, time frame. How did the patient lose the weight? What is the patients diet now? Still losing weight? How
often does the patient weigh herself? Are you proud of this weight loss? Do you think you need to lose more? Are you afraid of becoming
fat? Do you admire women who are smaller than you? Binge eating, post-prandial vomiting, laxative or diuretic abuse, excessive exercise,
diet pills. Wearing baggy clothes to conceal fatness, unable to look at self in a mirror or to be touched by others. Ask about the home
environment, is there a problem with expressing conflict openly? Signs of malnutrition, amenorrhea (> 3 consecutive menstrual cycles
missed), sallow skin, rash, easy bruising, dry and sparse hair, lassitude, weakness, anemia, neurologic findings (carpal and tarsal nerve
compression, confusion, emotional lability, loss of corticospinal vibration and position sense), glossitis, heart burn, teeth erosion, GI bleeding.
Counsel: Determine ideal body weight using standard height/weight charts (BMI = weight (kg)/height 2 (m2), ideal is about 20-25 for
females). Show patient her position on the chart. Explain that anorexia nervosa is a modern disease of highly motivated young women.
These women exercise extreme control over their bodies, often as a means of sublimating their inability to express conflict at home. Warn
patient that excessive weight loss has led to the deaths of many young women who were unable to correct their anorexia. Explain that proper
body weight is essential for health and mental function, including learning and performing well at school/career. You understand that the
patient may be proud of her weight loss. Being underweight may show a great deal of self control and will power, but being at ideal weight
shows more. Invite patient to develop a healthy body image by not equating soft or fatty body areas with overweight. Emphasize that
attractiveness and good health depend on a good balance of fatty tissues as well as lean. Contract with the patient to gain a certain number of
pounds per week. Discuss how she will do this.
Contract for specific weight gain goals (2 lbs/week). Involve dietician. Halt diuretics, laxatives, diet pills. Close monitoring of weight,
vitals, heart rhythm, potassium. Arrange follow up with patient and her family to discuss family dynamics, expression of conflict in the
home.
95. 2 year old child with history of fever and 1 seizure. Counsel parents.
See also question #9.
Most likely diagnosis: benign febrile seizure (febrile seizures usually occur 6 months to 6 years, associated with initial rapid rise in
temperature, no neurologic abnormalities/evidence of CNS infection/inflammation before or after, no history of non-febrile seizures, most
commonly generalized tonic-clonic, < 15 minutes duration, no recurrence in 24 hours, atypical may show focal origin/> 15 minutes/> 1/24
hours/transient neurologic defect).
Counsel parents regarding febrile seizure: A typical febrile seizure is a brief generalized tonic-clonic seizure related to high fever (at least
39 degrees Celsius) and occurring between the ages of 3 months and 7 years. The post-ictal stage is characterized by improvement in
confusion, lethargy, limpness. The greatest risk factor for febrile seizures is a history of febrile seizures in the parents. This is the most
common seizure in children (3-5% of children, M > F). Occur between the ages of 6 months and 6 years. Thought to be due to initial rapid
rise in temperature. These seizures may come about as a result of fever from any cause, including post immunization. In the absence of an
abnormal developmental history (CP, developmental delay), and an otherwise well child, they are usually benign. Seizures do not cause
mental impairment unless they are prolonged (> 30 min) but can be a symptom of brain damage.

Prognosis after a single febrile seizure: 65% will never have another seizure, 30% will have further febrile seizures, 3% will go on to have
seizures without fever, and 2% will develop lifelong epilepsy (risk factors for this are: developmental and/or neurological abnormalities of
child prior to seizures, family history of non-febrile seizures and an atypical initial seizure).
Treatment of recurrence: control fever with antipyretics (Tylenol), tepid bath, fluids for comfort only and use Ativan (lorazepam) 1 mg
SL/PO (or diazepam 5-10 mg PR) if a seizure occurs at home. Turn patient onto his/her side, do not force objects or fingers into mouth.
Bring to ER if seizure does not stop within 10 minutes. Seizures do not cause mental impairment unless they are prolonged (> 30 min),
although seizures can be a symptom of brain damage. Patient should be investigated with CT head and EEG. Prophylactic anticonvulsant
therapy is a consideration with repeated seizures.
96. Operating room nurse sustained needle stick injury. Worried about hepatitis and AIDS. Counsel.
History: Name, age, occupation. Determine severity of exposure: hollow bore needle? Needle gauge? Depth of penetration? Did needle
contain blood from a patient? Was any blood injected? Is the HIV and hepatitis status of the patient known? Is the nurse immunized against
Hep B?
Patient known to have:
HIV
Hep B
Antigen Hep B
Antibody Hep C

Odds of Transmission
0.3%
40%
10%
5%

Counsel: HIV has a high mortality rate within 5 years of testing positive, and is eventually fatal in most cases, however patients with HIV
have a much longer life expectancy than in the past due to improved anti-retroviral therapy. Hepatitis B causes fulminant hepatic necrosis in
1% of those infected, which is fatal in 60% of cases. 5% of those infected with Hep B remain in a chronic carrier state, which is associated
with a 25-40% risk of cirrhosis and 2-5% per year risk of hepatocellular cancer. There is a > 50% risk of chronic liver disease once infected
with Hep C, the risks of cirrhosis and hepatocellular cancer are similar to those for Hep B. The overall risk of transmission is as described
above. Recommend baseline testing for HIV, Hep B, and C in nurse and patient (require consent for HIV testing). If the patient was recently
infected, he/she may not become positive on antigenic testing for up to 3 months. Therefore nurse and patient should be retested.
Recommend HIV prophylaxis (AZT + 3TC x 4 weeks and consult Infectious Disease specialist) for significant needle stick from a patient
with known HIV or who is at high risk (multiple partners, IV drug use, anal intercourse, recent immigrant from endemic area). If the nurse is
not effectively immunized (i.e. antibody titers tested) against Hepatitis B, recommend immunization. If patient is found to be Hep B or C
positive, give the nurse passive immunity gamma globulin, Hep B gamma globulin (HBIG) within 7 days of exposure, has been proven to be
effective in preventing transmission.
97. Demonstrate and counsel a patient on breast self-examination and mammography screening.
Breast exam: Sitting: Inspect in four separate positions: 1) sitting with arms at her side, 2) sitting with both arms raised above her head, 3)
sitting with hands pushing on hips and elbows out, 4) patient leaning forward: inspect for size and symmetry, visible masses/contour
changes, skin retraction, erythema, dimpling, nipple retraction/inversion/ulceration/size & shape, peau dorange around nipple and elsewhere.
Palpation of axillary, infraclavicular, supraclavicular nodes. Supine (with pillow under shoulder), each breast examined separately, drape
other breast, small circular motions covering an area of approximately 1 square inch divide into light, medium and deep palpation and
perform in all four quadrants. Can denote position of lumps by clock position with cm distance from nipple. Nipple squeeze to try to exude
any fluid from the nipple (ask patient to squeeze nipple herself). Watch for dimpling, bloody nipple discharge and inflammation.
Mammography: Yearly mammography screening of proven benefit from age 50. Benefit as a screening test equivocal from age 40 in the
general population but is recommended if there is a positive family history of breast cancer. Breast cancer in two first degree relatives
(parents, siblings, children) is an indicator for yearly mammography starting at 5-10 years before youngest family members presentation.
98. 60 year old male, difficulty walking. Perform a neurological exam.
See neurological exam in question #5.
99. 37 year old well G1P0 female, 9 weeks pregnant. Manage.

History: Patient ID: Planned pregnancy? Status of any relationships at present including relationship with the childs father. Social supports
(family, friends, boyfriend), do they know? Are they helping? Employment/financial/educational status of the patient, does the patient feel
prepared to raise a child? Provisions for care of child when born? GTPAL (number of gestations, term pregnancies, premature births,
abortions, live children), history of problems, if any, with previous pregnancies. Current pregnancy, establish gestational age (GA) by last
menstrual period (LMP) if regular periods and sure dates (if unsure a dating ultrasound would be needed). The GA is the number of weeks
from the first day of the LMP. The EDC is first day of LMP + 7 days 3 months. Smoking (prepared to quit?), alcohol (no alcohol during
pregnancy), illicit drugs, diet, exercise, medications (avoid during pregnancy: including over the counter). Diabetes, family history of
inherited disorders, heart disease, circulatory problems, renal disease, hypertension. Menstrual history, regularity of cycles, how long has
patient not used contraception. Any morning sickness, vaginal bleeding? Past medical and surgical history, medications, drugs/alcohol,
smoking, allergies, family history, review of systems.
Psychiatric: Cover mnemonic for major depression. MSIGECAPS: mood (depressed), sleep (increased or decreasedif decreased, often
early morning awakening), interest (decreased), guilt/worthlessness, energy (decreased or fatigued), concentration/difficulty making
decisions, appetite and/or weight increase or decrease, psychomotor activity (increased or decreased), suicidal ideation positive diagnosis of
major depression requires five of these over a 2 week period, one of the five must be loss of interest or depressed mood. Symptoms do not
meet criteria for mixed episode, significant social/occupational impairment, exclude substance or GMC, not bereavement.
Physical: vitals, weight, height, palpation of neck and thyroid gland, fundoscopic exam, check lid lag, reflexes, cardiopulmonary exam,
breast exam, abdominal exam. Palpate uterus, measure symphysis-umbilicus distance. Doppler for fetal heart (may not detect until 10
weeks). Vaginal bimanual and speculum exam (cervix should be closed). Pap smear (if none in last 6 months, use speculum, not brush in
os), swab cervix for cultures (GC, chlamydia).
Investigations: CBC, lytes, INR/PTT, urea, creatinine, urinalysis, ECG. Blood group and type, Rh antibodies, VDRL and HbsAg routine,
rubella titer, HIV serology offered, serum folate, urine dip, microscopy and culture, TB skin test in patients from an endemic area, genetic
testing as indicated on history or for sickle cell in blacks. Triple screen (MSS) MSAFP, hCG, uE3 Trisomy 18, Trisomy 21, NTD (at
16 weeks). Amniocentesis at 15-16 weeks for alpha-fetoprotein and acetyl cholinesterase. Chorionic villus sampling (10-12 weeks) should be
offered given the patients age. Fetal ultrasound.
Counseling: Discuss risk of Downs syndrome due to maternal age, value of fetal genetic testing. Recommend daily pregnancy vitamin
preparation, milk and healthy diet. Do not increase food intake dramatically excessive weight gain not recommended, 2-3 lbs per month for
a total of 25-30 lbs gain in weight ideal. Do not diet during pregnancy. Continue normal activities and customary exercise. No alcohol, no
smoking, no medications of any kind unless discussed with MD. Control morning sickness with small meals and bland foods. Lying on side
decreases swelling and discomfort. Hemorrhoids, backache, heartburn, increased vaginal discharge are common. Follow-up every 4 weeks
until 32 weeks, then increase to every 2 weeks. Call if any concerns or troubling symptoms, especially abdominal pain, vaginal bleeding,
persistent headache, illness or infection.
100. 60 year old female with bloody vaginal discharge. Take a history.
History: Name, age, occupation. Think about: blood dyscrasias, thyroid dysfunction, malignancy, PCOS, endometriosis, PID, fibroids,
unopposed estrogen, or polyps. Onset of bleeding, frequency, estimate quantity (number of pads), color, consistency of discharge, associated
pain, vaginal discomfort, cramping. Previous episodes, history of fibroids, polyps, PID, PCOS. Post coital and rectal bleeding. Weight loss,
night sweats, fatigue. History of easy bruising/bleeding, inherited blood coagulation disorders. Age of menarche, age of menopause, age of
first sexual activity. Use of hormonal replacement therapy, which preparation? History of fibroids, reproductive tract cancers, last Pap smear.
Pregnancy history. Medications, drugs/alcohol, smoking, past medical history, surgical history, family history, review of systems.
101. 30 year old man with hematemesis and abdominal pain in the emergency department. BP 80/50, tachycardia. Manage.
Resuscitate as in question #6. Consult gastroenterology for immediate endoscopy.
102. Pregnant woman, 36 weeks gestation, has proteinuria and BP 150/85 (pre-gestational BP 110/65). Manage.
See question #46.
103. A mother is worried that her 1 year old looks pale. Take a history. Finding: breast fed for the first 2 months, then 2% milk. Q:
What is the most likely diagnosis? What investigations would you order?

History: Name, age. Feeds and feeding history (esp. fruit juice, excess milk). Growth pattern: weight loss? Diarrhea? (consistency, color,
quantity and frequency), blood in stool, melena stools, concurrent illness, vomiting, fever, anorexia, difficulty breathing, lassitude, dry
mouth/eyes, low urine output, illness affecting other children in the family or adults. Recent immunization, travel, antibiotics. Medications,
past medical history, allergies, birth history, pregnancy problems, maternal illness during pregnancy, family history, review of systems.
Most likely diagnosis: Iron deficiency anemia (most common cause of childhood anemia). Typically in bottle-fed infants (6-24 months)
receiving large volumes of cows milk should add iron-fortified cereal and iron rich foods starting at 6 months.
Investigations: CBC with peripheral smear, lytes, urea, creatinine, INR/PTT, serum ferritin, albumin.
104. Elderly man with creatinine 1000. Take a history. Q: Give a differential diagnosis. What investigations would you order?
History: Patient ID. suprapubic pain, pain on urination, frequency, urgency, frank blood in the urine (globular clots from bladder or string
shaped clots from ureters), color of urine, difficulty initiating or maintaining urinary stream, renal pain, groin pain. Provoking factors.
Associated symptoms including saddle anesthesia, loss of bowel control. History of recent UTI, STDs, TB exposure, pelvic irradiation,
bleeding diathesis, smoking. Fever, chills, nausea, fatigue. Previous renal colic/diagnosed nephrolithiasis? History of hypercalcemia,
hypertension. Malignant symptoms: night sweats, weight loss, fatigue. Medications, drugs (NSAIDs, anticoagulants)/alcohol, smoking, past
medical history, past surgical history, family history (polycystic kidney disease?), review of systems.
Differential diagnosis:
1. Pre-renal: Hypovolemia, poor cardiac output, renovascular disease, NSAID/ACEi use, liver failure.
2. Renal: Vascular malignant HTN, cholesterol emboli, HUS/TTP; Tubulo-interstial ATN (ischemic/toxin endogenous/exogenous),
AIN; Glomerular (< 5%). Causes: X-ray contrast, myoglobinuria, acute glomerulonephritis, DIC, pyelonephritis, intrarenal precipitation in
hypercalcemia, myeloma.
3. Post-renal: Obstruction: upper (clot, tumor, stone, external compression), lower (BPH, clot, stone, stricture, autonomic dysfunction).
Investigations: CBC, lytes, urea, creatinine, phosphate, ionized Ca++, magnesium, INR/PTT, AST, ALT, ALP, GGT, prostate specific antigen,
CK-MB, troponin, ABG. Urinalysis: microscopy, dip, culture and sensitivity. Abdominal x-ray, abdominal pelvic ultrasound. Post-void
catheterization. (Avoid IVP due to dye).
105. 1 year old boy with 6 months diarrhea. Take a history. Q: Give a differential diagnosis.
See question #50.
106. 58 year old lady in hospital 4 days post-op hysterectomy for fibroids. Agitated, had tactile hallucinations the previous night.
Take a history. Finding: history of alcoholism. Q: What is the most likely diagnosis?
History: onset of hallucinations, duration, description. Tactile hallucinations or bugs crawling on skin or on ceiling suggest alcohol
withdrawal. Associated fever, agitation, sweating, tremor, decreased level of consciousness, seizure? Any problems with surgical recovery,
wound healing, mobilization? Amount of alcohol consumed at home. History of alcoholism, leg swelling, SOB, chest pain. Current state.
Post-op medications (morphine, Demerol) previous bad reactions to these or to antibiotics? Previous episode like this one? Past medical
history, medications, drug and alcohol use, smoking, allergy, family history, review of systems.
Most likely diagnosis: alcohol withdrawal.
107. Young man with a swollen cervical lymph node. Perform a focused physical exam. Q: CXR shows mediastinal widening with
perihilar nodes. Describe. Give five features on history which would be helpful for diagnosis.
Physical exam: vitals, jaundice, nutritional status, buccal mucosa, teeth, breath (hepatic fetor), parotid hypertrophy, glossitis, inspect chest
for telangectasia, gynecomastia, loss of axillary hair. Hands: palmar erythema, clubbing, Dupuytrens contracture, wasting of hand intrinsics.
Palpate for lymph nodes in the neck, supra and infra-clavicular, axillae, groin. Examine the oral cavity and pharynx. Check for rashes.
Abdominal exam (supine): see question #29.

Differential diagnosis: lymphoma, leukemia, viral infection (mononucleosis, HIV, EBV), inflammatory autoimmune disease (sarcoidosis,
lupus), serum sickness (severe allergic reaction short of anaphylaxis), TB, liver disease with portal hypertension.
Five features on history helpful for diagnosis: viral prodrome, family history of sarcoid, lymph nodes painful, bone pain, pruritis, weight
loss.
108. Female patient found to have a nodule on routine CXR. Perform a focused physical exam. Q: Give a differential diagnosis.
What investigations would you order?
Cardiopulmonary exam as in question #13 and #24. See also question #49.
Investigations: old CXR for comparison (if lesion is old and unchanging, interventions are less aggressive, calcification is also associated
with benign lesions such as old granulomas), CT chest with CT guided needle biopsy, sputum for cytology and acid-fast staining (TB), TB
skin test, bronchoscopy with biopsy and washings if lesion seen, open biopsy or lobectomy.
Algorithm: solitary nodule previous CXR benign or unchanged (repeat in q3-6months for 2 years if unchanged observe, if changed at
any time continue), malignant or changed CT thorax: cancer (stage and treat), calcification (observe), no diagnosis bronchoscopy or
transthoracic needle aspiration still no diagnosis (resect for diagnosis), inflammatory (treat cause), cancer (stage and treat).
109. 60 year old male slipped and fell 6 days ago. Comes to you because of hemoptysis. Perform a focused physical exam. Finding:
positive Homans sign. Q: What is the most likely diagnosis? Give a plan form management.
Cardiopulmonary exam as in question #13, plus additional attention to calf size, tenderness, redness and pleuritic chest pain.
Homans sign: pain in the calf on dorsiflexion of the foot indicates thrombophlebitis. Check that trachea is midline. Is the patient on DVT
prophylaxis or anti-coagulation?
Most likely diagnosis: pulmonary embolus.
Specific investigations for PE: CT chest (only shows clinically significant PE), V/Q scan (conclusive when is shows high or low
probability), pulmonary angiogram (gold standard but invasive), ECHO, and serial (q2d) leg Dopplers for presence of DVT above the knee.
Others: CXR (often normal, Hamptons hump, Westermarks sign, rarely dilatation of proximal PA), ECG (sinus tachycardia, S 1Q3T3), ABG
(PaO2 usually decreased, PaCO2 decreased due to increase in overall minute ventilation, increased A-a gradient), D-dimer. See also question
#17.
Treatment: if suspicion of PE is high, anticoagulate before waiting for these tests with heparin 7500 U IV bolus (80 U/kg), then infuse at
1200 U/hr (18 U/kg/h). Measure PTT q6h, adjust dose for PTT 70-90s (2.5-3 times normal baseline). Start coumadin, to INR 2-3, continue
coumadin for 3 months.
110. Telephone in room. Mother calls because her child has just ingested a caustic cleaner. Manage over the phone. Q: What do you
do after hanging up the telephone? Give a plan for management.
See question #52.
111. Telephone in the room. Physician in a peripheral center calls wishing to transfer an unstable patient who has been in a motor
vehicle accident. Manage over the phone. Q: CXR shows opacification of the right lung. What is your diagnosis? Give the
immediate management of this problem.
Over the telephone: Physicians name, name of center, patients name. Injuries, investigations done, vitals, lab values. GCS, is patient
intubated? Peripheral physician must not transfer patient until he is stabilized, i.e. good BP, good oxygen sats, bleeding controlled, blood
products given as needed. Estimated time of arrival? Physician accompanied?
CXR: likely hemothorax.

Treatment: Chest tube in ER, drain hematoma and connect to suction through a bubble chamber. Consult thoracic surgery, prepare patient in
case of immediate OR.
112. Young woman with bilateral migratory arthritis of recent onset. Take a history. Q: Give a differential diagnosis. What
investigations would you order?
Note: Migratory arthritis suggests gonococcal infection.
History: Patient ID. Onset of arthritic symptoms, durations, joints affected, chronology. Associated fever, malaise, fatigue, rash, abdominal
pain and cramps, vaginal discharge, pain with urination, dyspareunia (painful intercourse). History of arthritis (rheumatoid, osteoarthritis),
psoriasis, Lyme disease (camping trips), Reiters syndrome, ankylosing spondylitis, sexually transmitted diseases including PID. Sexual
history: present partners, number of partners, fidelity of partner(s), use of condoms. Medications, drugs/alcohol, smoking, allergies, past
medical history, family history, review of systems.
Differential diagnosis: gonococcal arthritis, psoriatic arthritis, Lyme disease, Reiters syndrome, ankylosing spondylitis, rheumatoid
arthritis, osteoarthritis, gout.
Investigations: CBC, ESR, lytes, urea, creatinine, INR/PTT, blood cultures. Cervical swab for culture and sensitivity. Joint aspirate for
microscopy and culture.

Sample Qs
55 year old woman with joint pain - history
joint pain: location, onset/duration, swelling, function of joint, ADLs
characteristics: morning stiffness, worse after rest/activity
course: have you ever had it before (chronic/subacute/acute)
pattern of joint involvement: other joints
extra articular features: skin, eyes, bowel, kidney, lung involvement, urethritis, cervicitis, SI joints, enthesitis
constitutional symptoms
past medical history
family history (arthritis, collagen vascular diseases)
medications, allergies
social history: smoking, alcohol, sexual history
PEP
comment on x-rays of her hands
RA: periarticular osteopenia, symmetrical erosions at ulnar styloid, 1st, 2nd MCP & PIPs
OA: joint space narrowing, subchondral cysts, subchondral sclerosis, osteophytes, at the DIP, PIP and 1 st CMC
discuss initial management
single joint: synovial fluid aspiration for crystals, cells, C&S
colour
viscosity
WBC/mm3
%PMN

Normal
clear
high
<200
<25%

Non-Inflam
clear
high
<2,000
<25%

Inflammatory
opaque
low
>2,000
>25%

Infectious
opaque
low
>50,000
>50%

multiple joint
OA: conservative (weight loss, PT, OT, rest); acetaminophen, NSAIDs for inflammation, intraarticular steroids;
surgical options later
RA: conservative (education, OT, PT), NSAIDs for short term and start on a DMARD with appropriate baseline tests;
corticosteroids for acute flares or bridging to onset of DMARD

31 yo man with 10 year history of IDDM presents with blood sugar book - history
History
look at blood glucose book
ask about symptoms of hypoglycemia - is he aware?
ask about meals and snacks - does he know about the Canada Food Guide
exercise routine and daily activities
monitoring his glucose
past medical history
family history
medications: especially new ones
allergies
social: smoking, alcohol
PEP
4 suggestions to correct blood sugar levels (pre-lunch hypoglycemia, late evening hyperglycemia)
Insulin
regular
NPH/lente
ultralente

Duration
short
intermediate
long

Onset
-1
2-4
4-5

Peak
1-3
8-10
-

Duration
5-7
18-24
25-36

reduce Humulin R at breakfast (pre-lunch hypoglycemia)


ensure mid morning snack (pre-lunch hypoglycemia)
increase Humulin R at dinner (late evening hyperglycemia)
increase Humulin NPH (late evening hyperglycemia)

29 yo man with fall from height, ankle and abdominal injury, focused hx and physical
Address ABC with c-spine control
History
details of accident: when, how did it happen, how high, what did you land on
areas of pain aside from abdomen and ankle (spine, ribs, etc.)
associated symptoms: address abdomen first, then ankle if no other injuries
abdomen: pain, distension, hematuria, hip pain
ankle: open wounds (open #), able to weight bear
past medical history especially with respect to abdomen and ankle
medications, allergies
smoking, alcohol
last meal
Physical
vitals
chest exam
abdomen: inspect for wounds, palpate, percuss, auscultate BS +, RECTAL
pelvis: stability (compress on pelvis anteriorly, laterally and palpate suprapubic area)
ankle: neurovascular status of legs, inspect for swelling, open wounds, check for tenderness at knee, look, feel, move
PEP
4 indications for laparatomy: peritonitis, pneumoperitoneum (free air), gun shot wound in abdomen, hemodynamic instability with no
other source of blood loss, evisceration, blood in NG tube
3 causes of distended abdomen: (air/blood/fluid) pneumoperitoneum, hemorrhage, ascites, bowel obstruction
45 yo man struck by a car and knocked off his bicycle - primary assessment, mgmt of tension pneumothorax
Primary assessment
Airway with c-spine control
is the airway patent? blood, edema, decreased LOC, teeth, dentures
can the patient protect airway? Decreased LOC, facial #, cough/gag reflex
assess: look, listen (talking/stridor), feel facial skeleton
options:
1) jaw thrust/chin lift
2) suction for blood
3) pharyngeal airway: oral/nasal (for tongue)
4) endotracheal intubation with C spine control
5) cricothyroidotomy: will require tracheostomy later
Breathing
assess
look at chest wall (symmetry, cyanosis, accessory muscle use)
listen
feel: trachea, subcutaneous emphysema, rib fractures (AP, lat compression)
O2 saturation, CXR
options
1) oxygen
2) chest tube: large 32, at the anterior axillary line in the 4th or 5th ICS
3) intubate if cannot oxygenate (pulmonary contusion on admission CXR)
4) MONITOR: O2 sat, ABG, CXR (following chest tube insertion)
tension pneumothorax: tracheal deviation, distended neck veins, absent breath sounds, hypotension
management: 10 to 12 gauge needle in the 2nd ICS at the MCL superior to the 3rd rib; then chest tube
Circulation
assess: Vitals, CNS status, Skin perfusion, Urine Output
type of shock: Septic, Neurogenic, Hypovolemic/hemorrhagic, Obstructive, Cardiogenic, AnaphylactiK
options for management of hemorrhagic shock:
1) insert 2 large bore ivs (16 gauge)
2) normal saline/ringers lactate: run 2L wide open
3) then consider packed RBCs:
O positive for men
O negative for children and women of childbearing age
type specific: takes 10-15 minutes
crossmatched: takes 45 minutes

PEP
given X-ray: tension pneumothorax: tracheal deviation, collapsed lung
give 3 clinical findings of tension pneumothorax: as above
43 yo woman with right ankle injury - history and physical
History
details of accident: when, how did it happen (inversion, eversion, rotation)
associated symptoms: open wounds, ability to weight bear, pain in knee
any other injuries
previous ankle injury
past medical history
medications, allergies
smoking, alcohol
last meal
Physical
neurovascular status of ankle
pulses, capillary refill
sensory: L4 medial foot, L5 dorsum of foot 1st webspace, S1 lateral foot
motor: L5 tibialis anterior (dorsiflexion), S1 gastroc/soleus (plantar flexion)
look for deformities, swelling, bruising
feel: pain in malleolar zone and tenderness distal 6 cm of medial or lateral malleolus
move: dorsi/plantar flexion, inversion/eversion, internal and external rotation
check for tenderness at knee (Maissoneuve fracture)
Remember:
ankle x-ray if 1) pain in malleolar zone and bony tenderness over distal 6 cm of medial or lateral malleolus or 2) inability to weight both
immediately and in ED
PEP - Ankle Fracture
discuss significance of Xray findings:
Ring principle of the ankle: lateral malleolus, medial malleolus, posterior medial malleolus, deltoid ligament (medial),
syndesmotic ligament (tib-fib), calcaneofibular ligament
AP, lateral and Mortise view (15 degrees internal rotation)
Mortise view: should be symmetric along joint line no > 4mm and no tilt
asymmetry means bony or ligamentous injury
discuss rehab and pain management one month later
undisplaced fractures: below knee cast
displaced fractures: reduction
ORIF: fracture-dislocations, type C fractures, trimalleolar fracture, talar shift/tilt
37 yo female wants a second opinion regarding breast examination (her sister has breast ca) - physical exam
Physical
inspect
breast: 1) size & symmetry; 2) contour; 3) superficial appearance (peau dorange)
areola and nipple: 1) size & shape; 2) ulcerations/eczema; 3) discharge,
4) supernumerary nipples
1) sitting with arms down; 2) arms raised; 3) hands pressed at hips; 4) lean forward
palpation
supraclavicular nodes
axillary nodes
breast
vertical strip method or radial vector method
boundaries: 2nd to 6th rib from sternum to midaxillary line
gentle, medium and firm pressure
medial breast: patient with arm behind head
lateral breast: oblique recumbent position (lie partly on other side to flatten out lateral breast)
describe size, shape, consistency, delineation, tenderness, mobility
if time left over: chest exam, abdominal exam
PEP
most common causes of a breast lump in order of occurrence
relative frequency

< 35 years

35-50 years

> 50 years

1
2
3
4
5

fibrocystic changes
fibroadenoma
mastitis
carcinoma
fat necrosis

fibrocystic changes
carcinoma
fibroadenoma
mastitis
fat necrosis

carcinoma
fibrocystic changes
fat necrosis
mastitis

mammogram is suspicious
fine-needle biopsy or core aspirate; however, if negative may still proceed to biopsy
excision biopsy and obtain pathology
Stage I
tumour < 2cm
no nodes

Stage II
tumour < 5cm
nodes not fixed

Stage III
tumour > 5cm or
tumour any size plus
- invading skin
- invading chest wall

Stage IV
metastases

if positive biopsy, then


staging: 1) CXR; 2) abdo US/CT + LFTs; 3) Bone scan
local therapy: lumpectomy/axillary node dissection/radiotherapy or mastectomy/axillary node dissection
adjuvant therapy:
estrogen positive:
node negative: tamoxifen +/- chemo (nothing if <1cm)
node positive: tamoxifen +/- chemo
estrogen negative:
node negative: chemo (nothing if < 1cm)
node positive: chemo
therapy for metastatic disease
chemotherapy
hormonotherapy
radiotherapy especially for bony mets
surgery
in situ breast carcinoma = stage I (follow above for local and adjuvant therapy)

21 yo female found unresponsive by mother, prior hx of increasing ICP - physical exam


Physical
address ABCDE
stat bloodwork
cocktail: thiamine, glucose, naloxone
general appearance: noggin (head injury), neck (meningismus), needles, eNt: otorrhea, rhinorrhea, odours (fetor, DKA, alcohol)
vitals: HR, RR, BP, temp
skin
respiration: progressions of
1) Cheyne-Stokes (bilateral cerebral hemisphere dysfunction / CHF)
2) central neurogenic hyperventilation (midbrain - pons)
3) apneustic breathing - prolonged pause at end of inspiration (pons)
4) ataxic breathing (medullary)
eyes
pupils
movements: intact brain stem
oculo-cephalic reflex: move head in one direction and eyes move to opposite
oculo-vestibular reflex: COWS
movements: conjugate tonic gaze (frontal eye fields push eyes to opposite side)
eyes look towards frontal deficit
eyes look away from pontine deficit
think about increased ICP affecting CN VI (because longest course)
think about transtentorial herniation affecting ipsilateral CN III (because compression)
motor
asymmetry
spontaneous motor response

response to pain
decorticate: damage to hemispheres
decerebrate: damage to midbrain or upper pons
flaccid: damage to medulla
reflexes
verbal response

PEP
identify CN palsy ( III: down and out; VI: inability to abduct eye)
Significance of palsy: as mentioned above = III uncal herniation; VI increased ICP
Assign GCS
Eye Opening
4 - spontaneously
3 - to speech
2 - to pain
1 - nil

Verbal Response
5 - oriented and converses
4 - disoriented, confused
3 - inappropriate words
2 - incomprehensible sounds
1 - nil

Motor Response
6 - obeys commands
5 - localizes pain
4 - withdraws to pain
3 - decorticate
2 - decerebrate
1 - nil

Woman with claudication


History
pain: location, OPQRST
reproducible claudication distance
relieved by rest (usually 5-10 minutes)
previous history of pain
rest pain, night pain, ulcers, gangrene
cardiac risk factors
past medical history
family history
medications
allergies
social: smoking, alcohol
Differential diagnosis: spinal stenosis, disc disease, arthritis, venous disease
Investigations: Ankle-brachial index <0.9 is abnormal, Angiogram = gold standard
PEP
management
conservative: stop smoking, control DM, hypertension, foot care, exercise
when to do surgery
critical ischemia < 0.5 (rest pain, night pain, ulcers, gangrene)
subjective disability severe (ie. walks less than 1block)
types of surgery
transluminal angioplasty
inflow procedures for aortoiliac disease
endarterectomy
profundoplasty
femoropopliteal bypass
Renal colic
History
pain: OPQRST
previous history of pain
associated GU symptoms: dysuria, hematuria, frequency, urgency
GI symptoms: nausea, vomiting, bowel symptoms
gyne symptoms: discharge, sexual history
constitutional symptoms
past medical history
family history

medications, allergies
social: smoking, alcohol
PEP
type of stone (90% of stones radioopaque KUB)
calcium stones 80%: calcium oxalate and calcium phosphate
increase water intake, avoid oxalate foods (caffeine, potatoes, rhubarb)
correct underlying cause
struvite stones 10%: triple phosphate, staghorn calculi, due to infection by Proteus, Klebsiella, Pseudomonas, Provididencia,
Staph aureus
complete stone clearance
acidify urine
antibiotics for 6 weeks with follow up urine cultures
uric acid stones 10%: radiolucent on Xray
increase fluid intake, avoid high protein/purine diet
alkalinize urine
allopurinol
cystine stones
increase fluid intake, avoid high protein/purine diet
alkalinize urine
management
medical: if < 5mm = do nothing (one mo. of medical tx before considered failed)
surgery
kidney
ESWL if stone is < 2.5 cm
+ stent if stone is 1.5 - 2.5 cm
percutaneous nephrolithotomy if >2.5 cm, staghorn, UPJ obstruction, cystine stone
open nephrolithotomy if extensively branched staghorn
ureter
ESWL
ureteroscopy: failed ESWL, ureteric stricture, distal 1/3 of ureter
IVP: find stone and describe findings
anatomy
obstruction
extravasation of dye
uric acid = filling defect
Man with hip pain
Physical
Inspect: resting position of hip (do not tend to see swelling/redness because joint is deep)
Palpate: tenderness
trochanter: widest point when lying, highest point when on side
trochanteric bursa: just posterior to greater trochanter
ROM: active and passive
flexion
internal rotation: keep femur center line, and move foot outwards
external rotation: keep femur center line, and move foot inwards
abduction: hand on opposite iliac crest and abduct until opposite crest moves
adduction: across other leg to about 20 degrees
extension with patient on side
SI joint: one inch lateral to dimples of Venus - palpate for tenderness and load laterally
If time: knee exam

Positive Trendelenberg: when standing on one foot on weak side, contralateral side of pelvis drops
weak abductors, trochanteric fracture
painful hip due to OA, acetabular instability
fractured pelvic side wall
Gait: Trendelenberg gait occurs when leans over the side that is weak

PEP
xray

OA describe joint space narrowing, sclerosis, subchondral cysts, osteophytes


management of OA
conservative (weight loss, PT, OT, rest)
acetaminophen, NSAIDs for inflammation, intraarticular steroids;
surgical options
replace: arthroplasty
realign: osteotomy
ablate: arthrodesis

Woman with hand pain, swelling in fingers


joint pain: location, onset/duration, swelling, function of joint, ADLs
characteristics: morning stiffness, worse after rest/activity
course: have you ever had it before (chronic/subacute/acute)
pattern of joint involvement: other joints
extra articular features: skin, eyes, bowel, kidney, lung involvement, urethritis, cervicitis, SI joints, enthesitis
constitutional symptoms
sexual history
past medical history
family history (arthritis, collagen vascular diseases)
medications, allergies
social history: smoking, alcohol
PEP
xray: periarticular osteopenia, erosions
management of RA
patient education, exercise, PT, OT
symptomatic treatment: NSAIDs, corticosteroids (intraarticular/systemic)
DMARDs
Hydroxychloroquine (antimalarial): dose 400 mg/day
six month trial
hyperpigmentation, myopathy, rash, diarrhea, retinal toxicity
baseline Ophthalmologic exam q 3 months
Methotrexate (folic acid antagonist)
starting dose 7.5mg/week p.o., injection, increased to 25mg/week
response within 6-8 weeks
GI, stomatitis, rash, alopecia, myelosuppression, hepatotoxicity, pulmonary toxicity
contraindications: sulfa containing Abx and HIV infection, alcohol abuse
CBC monthly, Liver q2months
Sulfasalazine
dose of 1g BID-TID
six month trial
rash, myelosuppression, GI, heartburn, dizziness, headache
CBC with differential and chemistry profile monthly
Long term IDDM hypoglycemic periods with new job, charts with glucose monitoring and insulin levels
cover manifestations of diabetes
macrovascular: accelerated atherosclerosis = CAD, stroke, PVD
microvascular: retinopathy, nephropathy, neuropathy (also autonomic)
meals, snacks, exercise
change insulin or lifestyle (activities)
past medical history
family history
medications, allergies
social: smoking, alcohol
While talking he passes out
hypoglycemia
symptoms: palpitations, sweating, anxiety, tremors

headache, fatigue, confusion, amnesia, seizures, coma


etiology
renal or hepatic failure
exogenous
drugs: insulin, oral hypoglycemics, alcohol
treat underlying cause
unconscious: give iv. D50W or if no iv. access, give glucagon IM
if conscious: give juice
PEP
management with chart: refer to previous scenario in terms of change in treatment
diabetic coma: distinguish between hypoglycemia as above and DKA
start with ABCs
rehydration: calculate fluid deficit and replace deficit over 1st 8 hours, then rest over next 16 hours
potassium: monitor carefully and add potassium to iv if K < 5.5 and patient is peeing
insulin: iv bolus 0.1-0.2 U/kg them maintain at 0.1 U/kg/hr until blood glucose <15; then add glucose to iv solution and
decrease insulin to 1-2 U/hr
bicarbonate: not recommended unless pH < 7.0
monitoring: glucose, lytes, anion gap, urea and creatinine q2hr
Woman with acute MI complains of chest pain
History
chest pain
associated symptoms: nausea, vomiting, SOB, diaphoresis
previous history of chest pain
cardiac risk factors
past medical history
family medical history
medications, allergies
social: smoking, alcohol
Physical Exam
general appearance
vitals
chest exam
cardiac exam
Investigations
CBC, lytes, BUN, Cr, AST, LDH, Bili, PT, PTT
CK/CK-MB or Troponin I, ECG
Indications for thrombolysis
ST elevation > 1mm in two contiguous leads
new bundle branch block
within 12 hours post onset of pain; the earlier the better < 6 hours
Contraindications for thrombolysis
recent major surgery (within 2-4 weeks)
active internal bleeding or bleeding diathesis (not including menses)
history of hemorrhagic strokes or any recent stroke
marked hypertension (BP>180) or diastolic >100
known intracranial neoplasm, AVM, or aneurysm
CPR >10 min of chest compressions within 3 weeks
pregnancy
Indications for Angioplasty
trend toward lower mortality, less reinfarction, fewer strokes when compared to thrombolysis
as effective but not superior to thrombolysis plus ASA
considered for those in which contraindications exist for thrombolysis
emergent in all patients with shock
Respiratory physical exam
Physical
General Appearance

Vitals: RR
Inspect: cyanosis, accessory muscle use, AP diameter
Palpate: tracheal deviation, respiratory excursion
Percussion: diaphragmatic excursion, lung areas
Auscultate
Special tests: tactile fremitus, whisper pectoriloquy, egophony
PEP
crackles at bases: atelectasis, pneumonia, upper border of effusion, pulmonary edema
pneumonia - community acquired
not severe, < 65 years: S. pneumoniae, Mycoplasma, Chlamydia
treatment: macrolides, or tetracycline/doxycycline
not severe, > 65 years +/- comorbid: Above + H. flu, Klebsiella, Staph, Legionella
treatment: extended gm negative coverage = cefuroxime, clarithromycin
treat iv. until afebrile 48-72 hours then switch to p.o.
treat Strep for 10 days
treat atypicals for 21 days
Breast lump - history
History
description of lump: size, location, mobility, tenderness, fluctuation with menstrual cycle
associated symptoms: skin changes/edema/erythema, nipple discharge/eczema/retraction
previous lumps: benign or malignant
constitutional symptoms
risk factors:
increasing age
family history: 1st degree relative
premalignant breast lesions
personal history of breast cancer
hormonal: age at first pregnancy, early menarche, late menopause, nulliparity
previous exposure to radiation
obesity, excess dietary fat, high alcohol consumption
past medical history
medications, allergies
social: smoking, alcohol
PEP
age groups: see previous chart with scenario on physical exam of breast lump
Woman with RLQ pain classic appendicitis
History
pain: OPQRS
previous similar pain
associated symptoms: nausea, vomiting, diarrhea, constipation, jaundice, weight changes
gyne symptoms (rule out ectopic and PID): LMP, sexual history, # of partners, vag discharge
GU symptoms: dysuria, frequency, urgency, nocturia, hematuria
constitutional symptoms
past medical history
medications, allergies
social: smoking, alcohol
Physical
General Appearance
Vitals
Abdomen: look for signs of peritonitis, RECTAL
Gyne: PELVIC exam
Investigations
CBC, lytes, BUN, Creatinine
beta HCG
pelvic US to rule out adnexal pathology

Man with hematemesis GI bleed


History
GI bleed: how much, circumstances (ie. retching, spontaneous)
previous history of bleed, PUD
associated recent symptoms: nausea, vomiting, hematemesis, heartburn, dysphagia, diarrhea, constipation, melena, hematochezia,
jaundice
resulting symptoms: anemic, fatigue, SOB, lightheaded, syncope
risk factors for PUD: NSAIDs, steroids, smoking, alcohol, anticoagulants
constitutional symptoms: fever, weight loss, night sweats
past medical history: abdominal OR, aortic grafts
medications, allergies
social: smoking, alcohol
Physical
General appearance
Vitals
CNS status: alert or decreasing LOC
Abdomen: + RECTAL for melena, OB positive blood

OSCE SCENARIOS
1) Headache history space occupying lesions
Pain
Onset: sudden
Precipitating: head injury
Palliating: meds
Aggravating: position, time, activity, stress
Quality different from previous headaches?
Quantity (?/10), wake from sleep
Radiation
Site
Temporal: ever had before, duration, worse in am, worsening
Associated symptoms
N, V, weakness, sensory disturbance
Constitutional fever, chills, weight loss, loss of appetite
Visual changes, jaw claudication, polymyalgia rheumatica
Aura, photophobia, phonophobia
Risk factors:
Age
Cancer, immunocomp
Family history aneurysms
HTN
PMH: migraines, aneurysms, meds ( analgesics, NTG, toxins-CO)
2) Oliguria history CHF

Pre-renal
Volume depletion intake?
Poor cardiac output (CHF, tamponade, MI, PE)
Sx:?DOE, orthopnea, PND, ankle edema
Precipitants: MI, HTN, valvular heart disease, congenital heart disease, pericardial disease, cardiomyopathy, PE, fluid overload,
sodium retention, salt overload, dysrhythmia, beta blocker, renal disease, anemia, fever and infection, pregnancy,
noncompliance with meds or diet.
Risk: CAD, HTN, cardiomyopathy
Shock/sepsis fever, chills, focus of infection

Renal
Glomerular nephritides
Tubular interstitial pyelo, hypercalcemia
ATN recent surgery
Nephrotoxic agents antibiotics, contrast dye, anaesthetics, NSAIDs, chemo
Vascular problems emboli, renal vessel thrombosis

Post-renal
Obstruction stones, tumour, BPH, strictures, clots, retroperitoneal mass
Bladder rupture trauma

3) Hip exam OA, femoral neck fracture


See Yulias answer
4) Knee pain physical exam

Inspection compare with other knee, examine pt. Standing as well


Swelling, erythema, muscle atrophy, deformity (e.g. varus/valgus), skin changes
Gait
Palpation
Temperature
Joint line tenderness (knee at 90deg, also palpate collateral)
Effusion bulge sign (milk fluid up medial knee, pass hand down lateral side), ballotment (balloon sign)
Patellofemoral compartment push patella against the femur and ask patient to flex quads pain +/- crepitus
Bakers cyst
Neurovascular pulses, sensation, reflexes, motor
Movement
Passive and active look for decreased ROM on flexion, extension, and hyperextension
Special Manoeuvers
Medial/Lateral collateral Ligaments: valgus/varus stress to open up, feel for ligament
Anterior/ Posterior Cruciate Ligaments: sitting on patients foot with knee flexed 90 degrees, pull leg towards/away from you
Menisci: McMurray test- knee flexed, leg externally rotated place valgus stress on knee then straighten knee postive with
palpable/audible click
Classically - pain along medial or lateral aspect of knee. Pain on full extension and pain along joint line + effusion = meniscal tear.
Mention you would examine joints above and below knee as well

5) Chest pain CAD, angina history

Pain description
Ever had any CP before?
Quality: heavy, burning, tightness, stabbing, pressure
Precipitating/ Aggravating: walking level/uphill - quantify, food, cold
Alleviating: rest, NTG (in 10 min)
Radiation: arms, jaw, epigastrium
Onset
Site

Associated Symptoms
N, V, diaphoresis, palpitations, dyspnea, orthopnea, PND

CAD Risk factors


Smoking, DM, HTN, hyperlipidemia, fam hx (first degree relative male<55 yo, female<60 yo)

PMH
Also, previous cardiac disease

Meds

6) Respiratory exam emphysema

Inspection
Face distress, nasal flaring, pursed lips
Cyanosis (frenulum/lips, finger/toes/nose)
Posture ( usually leaning fwd, elbows resting on knees)
Neck accessory muscles
Chest AP dia., deformities, indrawing, assymetry
Resp rate and pattern
Hands clubbing (not seen in emphysema), nicotine stains
Palpation
Trachea position and mobility
Chest excursion

Diaphragmatic excursion
Tactile fremitus
Percussion
Lungs hyperresonant? compare sides
Loss of cardiac dullness
Auscultation
Breath sounds
Vocal resonance
FET: normal individuals can empty their chest from full inspiration in 4 seconds or less. The end point of FET is detected by
auscultating over the trachea in the suprasternal notch. Prolongation of the FET to more than 6 seconds indicates airflow obstruction
Dont forget R middle lobe, anterior and posterior chest

7) History of foot pain in diabetic who stepped on nail

Examine legs and feet


Wound clean, infected
Neuropathic foot
Dry, warm and pink with palpable pulses
Impaired DTR
Reduced pinprick, light touch and vibration sensation
Ulcers usually painless and plantar

Ischemic foot
Skin is shiny and atrophic with sparse hair
Cold to touch
Peripheral pulses absent
Ulcers painful heels and toes

8) Facial trauma + diplopia examine

Inspection
Mandible: malocclusion, crepitus, dislocation, deformity, ?
Maxilla: swelling, malocclusion, crepitus, donkey face (II), CSF rhinorrhea (III)
Zygoma: periorbital ecchymoses, flat cheek
Nose: nasal bone fracture
Blowout: up/down gaze, vertical diplopia, enopthalmos, pseudoptosis, infraorbital nerve hypesthesia/anesthesia

9) Examine for lymphadenopathy above the diaphragm

Describe: size, consistency, tenderness, mobility


Areas
H&N
Occipital, p. auricular, p. cervical, superficial and deep cervical, tonsillar, submaxilllary, submental, ant. Auricular,
supraclavicular, infraclavicular also look inside mouth
Breast/Axilla
Epitrochlear
90 degree flexion of patients arm feel in fossa about 3 cm proximal to medial epicondyle of humerus, in groove between
biceps and triceps

10) Focused liver exam alcoholic

Hands
Clubbing, leuconychia (pale nail)
Dupuytrens contracture, palmar erythema, spider nevi, tattoos, hepatic flap, pallor, scratch marks, generalized pigmentation
Eyes and face
Icterus, cyanosis, parotid enlargement, fetor hepaticus
Chest

Spider nevi, loss of axillary hair, gynecomastia


Abdomen
Splenomegaly, ascites, hepatomegaly, caput medusae
GU
Testicular atrophy, loss of pubic hair
Leg edema

11) Hematuria history

ID/CC
HPI
Hematuria onset, amount of blood, timing (init=urethral, terminal= bladder neck, constant bladder of above), colour, clots
Associated Symptoms
Pain (perineal/ abdo/ flank)
Irritative dysuria, urgency, freq, nocturia
Obstructive hesitancy, straining, intermittent stream, incomplete voiding, decreased force/calibre of flow, post-void dribble
Provoking factors exercise, trauma
Previous kidney/urologic disease ( remember BPH, stones)
Recent UTI/STD/TB exposure/ pelvic irradiation/ bleeding diathesis
Recent URTI / sore throat (post-infectious GN)
Constitutional wt/fever/fatigue/ night sweats
Drugs NSAIDs/ anticoagulants / cyclophosphamide
Occupational dyes, leather, rubber, paint, benzene

12) Examine midline neck mass thyroid ca

Inspection of neck
JVP
Surgical scars
Enlarged cervical lymph nodes
Goitre

Palpation (from front or back) make sure water around


Seat the patient comfortably
Comment first on exopthalmos
Palpate isthmus and both lobes
Stand behind patient, whose neck is slightly extended. Put both hands around neck. Use left hand to push trachea to right
identify thyroid cartilage - ask for swallow feel for thyroid against right sternomastoid repeat on left
Size, mobility, texture, tenderness
Extra (cervical nodes, carotids, tracheal deviation)

Auscultate over gland for bruits


Test sternomastoid function ( may be infiltrated in thyroid malignancy)
? Thyroid function
Eye Signs
Lid lag, exopthalmos, lid retraction, EOM
Hands
Pulse (tachy/ a-fib), tremor, clubbing, palmar erythema, supinator jerks
Skin
Pretibial myxedema
Ankle jerks

13) Examine LLQ pain in male diverticulitis

History
pain: OPQRS
previous similar pain

associated symptoms: nausea, vomiting, diarrhea, constipation, jaundice, weight changes


GU symptoms: dysuria, frequency, urgency, nocturia, hematuria
GI symptoms: N,V, dysphagia, loss of appetite, hematemesis, diarrhea, constipation, melena, hematochezia
constitutional symptoms
past medical history
medications, allergies
social: smoking, alcohol
Physical
General Appearance
Vitals
Abdomen: look for signs of peritonitis, RECTAL

14) History of man with spells seizure

LOC? how long? (seizure s-min)


Before LOC: what were you doing (orthostatic hypotension, micturition syncope, vasovagal), warning (light headed, palpitations,
diaphoresis = likely syncope/ abdo pain, goose bumps and flush = likely autonomic)
Aura (sensory/motor seizure)
Triggers: TV, hypoglycemia, alcohol, sleep deprivation
Recurrent? Increasing freq?
Constitutional sx: fever, neck stiffness, weight loss, night sweats
PMH/Risk factors: Family history, alcohol use/WD, drugs TCA, MAOI, cocaine, amphetamines, history of head trauma, stroke, CNS
infections, cancer
Eye witness?: tonic-clonic, frothing at mouth, tongue biting, unconscious, incontinent, duration, weakness after attack (Todds
paralysis), drowsy

15) History and physical of joint RA, OA

History
Age
Gender ( males mostly seronegative)
Pain: worse with rest = inflammatory, other pain qs
Morning stiffness: >60 min = inflammatory, <30-60 min = non-inflammatory
Distribution of joint involvement
Symmetrical, asymmetrical
Large/small
Peripheral/central (spinal dist)
Upper/lower limbs
Temporal profile of disease activity (eg. OA slowly and steadily progressive, vs. gout, intermittent exacerbations and remissions)
Degree of disability: functional capacity and ADLs
Treatment
Family history: AS, SLE, RA
PMH: DM, IBD, psoriasis, GU/GI infections, renal disease
Meds: diuretics, cyclosporin, hydralazine, procainamide, anticonvulsants
Extra-articular features: (too many to list! Constitutional, skin, mucous membrane lesions, urethritis, Raynauds, conjunctivitis, GI,
pleuropericardial pain, etc.)

Physical Exam (compare affected joint to normal)


General appearance of patient, evidence of extra-articular features, adenopathy, organomegaly, etc.
Inspection: swelling, redness, deformity, muscle atrophy
Palpation: warmth, joint line tenderness, stress pain, effusion, crepitus
ROM: active, passive, against resistance ( to detect lesions in tendons and to measure power)
Supporting Structures: look for instability
Also: neuro, power, vascular, skin lesions

16) Assessment of volume status

Vitals
HR, RR, BP postural changes?
Mental status
HEENT
Mucous membranes moist/dry?
JVP
Resp
Crackles and decreased A/E
CV
S3
Abdomen
hepatomegaly
Skin
Decreased warmth/turgor/dry axilla?
Dependent edema sacral, calves

17) Melena history


See upper GI bleed
18) Respiratory exam consolidation
See #6 above for more complete exam
Salient features:
Purulent sputum
Tachypnea
Reduced movement of affected side
Trachea central
Impaired percussion note
Bronchial breath sounds
crackles
19) Post-exam probe: DDx LLL crackles

Congestive heart failure


Obstructive lung disease
Interstitial fibrosis
Pneumonia
Atelectasis
Pulmonary embolus
Bronchogenic carcinoma
Bronchitis
Bronchiectasis + Non-cardiogenic pulmonary edema

Man with Rectal Bleeding


History
- blood in stools - frank blood vs on surface vs mixed with stools
- melena, hematochezia, hematemesis (coffee ground stools), tenesmus
- abdo pain - OPQRST ; Nx,Vx, abdo distention, diarrhea, constipation (change in bowel habits)
- any other sites of bleeding - mucosal bleed, hemarthosis, epistaxis, easy bruising etc
- sx of anemia - fatigue, dizziness, SOB, C/P etc
- constitutional sx - fever, chills, weight loss!!

- diet - ?fiber
meds - NSAIDS, ASA
past medical history - hx PUD, diverticulosis, coag disorders, vasculitides, hemrrhoids, alcohol
surgical hx - endoscopy, aortic surgery
fam hx - IBD, cancer, blood dyscrasias
PEP - signs of acute blood loss; on physical examination, focus on:
vitals - tachycardia, hypotension with postural changes, tachypnea, urine output??
H/N - dry mucous membranes, flat JVP
CVS - poor peripheral pulses and poor cap refill
Skin - cool, clammy skin, pale
CNS - decreased LOC, confusion
Hemorrhagic Shock :
Class

Blood loss

BP

Pulse

Resp rate

U/O per hr

15%, <750

<100

14-20

>30

15-30%, 750-1500

>100

20-30

20-30

30-40%, 1500-2000

dec

>120

30-40

5-15

>40%, >2000

dec

>140

>40

Visual Problems - localization of findings


Most important test is visual fields to isolate lesion
1. right anopia (loss of vision) = right optic nerve lesion
2. bitemporal hemianopia (loss of lateral fields) = chiasmal lesion
3. left homonomous hemianopia (loss of left fields) = right optic tract lesion (behind chiasm)
4. left upper quadrantanopia (loss of upper left quadrant) = right temporal lesion
5. left lower quadrantanopia ( loss of left lower quadrant) = right parietal lesion
Medical history of diverticulosis - LLQ pain
History
- abdominal pain, OPQRST - relation to meals etc.
- associated symptoms - nausea, vomiting, flatus, abdo distension, diarrhea, constipation, melena, hematochezia, hematemesis
- R/O gyne causes - sexually active, LMP, menstrual hx, birth control etc.
- R/O renal causes - flank pain, urine output, hematuria, dysuria, obstructive sx's
- constitutional symptoms
- diet - ?fiber
- medication - ASA, heparin, coumadin, other blood thinners
- family hx of cancer, diverticulitis, polyposis
- past medical history - colonoscopy, GI ds, see below for ddx
PEP - Ddx LLQ pain
diverticulitis

rarely appendicitis
bowel obstruction
IBD
kidney stone
ruptured ovarian cyst
tubal torsion/salpingitis
ectopic preganancy
endometriosis
psoas abscess
Gross hematuria
Hematuria - OPQRST, had it before?
color, timing and pattern of hematuria - initial (anterior urethral), terminal (bladder neck,prostate), throughout (bladder or upper urinary tract)
Associated symptoms - flank pain, abdo pain, dysuria, urine output, frequency?
R/O coagulopathy - epistaxis, mucosal bleeding, melena, hemathrosis
Constitutional symptoms - fever, wt loss, chills, lethargy
ask re : ingestion of beets, dyes, menstruation, smoking,trauma
recent sore throat, strept skin infection (GN)
Past medical history - stones, foley cath, pyelo
Meds - cyclophosphamide, asa, coumadin, ibuprofen, allopurinol, phenobarb, naproxen
Family history - stones, bladder cancer, coagulopathy, renal disease, sickle cell, hypertension
PEP - Transitional Cell Carcinoma of Bladder - interpret IVP, initial investigations, Rx
Investigations - urinalysis, urine cytology, U/S, IVP
- staging - CXR, CT scan, liver tests
Rx - CIS --> TURBT + intravesical chemotx : BCG, thiotepa, mitomycin C
TIS,TaT1 --> TURBT and chemotx as above
invasive ds (T2-T3) --> radical cystectomy with urinary diversion + irradiation
metastatic ds (T4) --> irradiation and systemic chemotx
Head and Neck Exam / Examine Lymph Nodes above the diaphragm
Examination of LN's above diaphragm
- examine the following --> occipital, post auricular, pre-auricular, cervical chains (posterior, superficial, deep), tonsillar, submandibular,
submental, SUPRA-CLAVICULAR (inspire to feel properly)
--> EPITROCHLEAR (medial surface of arm, 3cm above the elbow), AXILLARY
PEP - DDx lymphadenopathy
Infection - Bacterial, viral (mono, CMV, cat scratch, HIV), parasitic (toxo), spirochetal (syphilis), mycobacterial (TB, MAC), fungal
(actinomycosis, cryptococcosis
Drug reaction - serum sickness, phenytoin

Malignancy - solid tumors with mets eg. H/N cancer (cervical), GI tumors (supraclavicular), breast cancer (axillary), anal ca (inguinal),
LYMPHOMA!!
Misc - sarcoidosis
PEP - staging for hodgkin's disease
I - single lymph node region (LNR)
II - two or more LNR's on the same side of the diaphragm
III - LNR on both sides of the diaphragm, spleen may be involved
IV - diffuse involvement of extra-lymphatic sites
To stage properly, order : CBC, diff, coags, U/A, CXR, CT chest, abdo and pelvis, BM aspirate and bx, consider lymphangiogram and
gallium scan
Comatose Patient - Scream out orders and do physical examination
Orders in comatose patient
- O2 sat - O2 standing by via mask
- 2 large bore IV's - consider giving bolus 2L NS
- Cross and type for 4U of blood
- Accucheck
- Draw blood for CBC, lytes, BUN, Cr, glucose, PT, PTT, INR, tox screen,

amylase

- foley and NG tube if needed


- xrays if needed, ct scan etc.
Physical Examination - ABC's as always
1. Airway - first secure/immobilize the spine
- chin lift / jaw thrust
- sweep and suction
- oral/nasopharyngeal airway/intubation if needed
--> look for resp distress, failure to speak, dysphonia, conduct (agitation, confusion, choking)
--> always reassess the airway because status can change
2. Breathing
-look - watch for chest movement, resp rate/effort, patient's color
- listen - auscultate for breath sounds and symmetry of air entry
- feel - feel for the flow of air, chest wall for crepitus, flail segments and sucking
- assess - tracheal position, neck veins, resp distress
3. Circulation
- check for pulse rate and rhythm, blood pressure
- LOC, skin color, temp, cap refill
- stop any major external bleeding, elevate bleeding extremity
4. Disability
- AVPU method (shout name --> sternal rub/pinch fingers)
- look at pupils
5. Exposure
- keep patient warm with a blanket

chest wounds

SECONDARY SURVEY
- AMPLE - allergies, medications, past med hx, last meal, events related to injury
- Head to toe survey - tubes and fingers in every orifice, splint fractures, look for toxidrome, ORDER X RAYS!!!!
- neurological, abdomen and perineal exam
PEP scenarios in past
1. Tension pneumothorax - needle decompression and chest tube
2. One fixed/dilated pupil - management of raised ICP, do GCS (nb!!)
- 100% oxygen, intubate and hyperventilate to a pCO2 of 30mmHg, mannitol 1g/kg, raise head of stretcher
Patient with a palmar gash - examination of the hand
1. Inspection - laceration, swelling, erythema, deformity, muscle atrophy, foreign bodies
2. Nerves
a. Radial - sensory - dorsum of 1st web space
- motor - extend wrist (extensor carpi radialis)
- "hitch hike" (extensor pollicus longus)
b. Ulnar - sensory - ulnar 1.5 digits, pad of pinky finger
- motor - flex DIP of little finger (flex dig profundus)
- froment sign - pull paper between index/thumb (adductor pollicus)
- spread fingers out - 1st dorsal interossei
c. Median - sensory - radial 3.5 fingers, index finger pad
- motor - thumb to ceiling (abductor pollicus)
- flex distal IP index (flexor digitorum profundus)
- pinky and thumb together (opponens)
--> check digital nerves with 2 point discrimination and light touch
3. Vascular
- feel for pulses
- capillary refill, skin color and temperature
- allen's test
4. Flexor tendons (palmar)
- flexor dig. profundus, isolate and flex DIP
- flexor dig. superficialis, isolate and flex PIP
- intrinsics, isolate and flex MCP
5. Extensors (dorsal)
- extensor dig communis - MCP extension
- intrinsics - PIP and DIP extension
--> never test tendons against resistance if suspect tendon laceration, let patient actively move the joints themselves
6. Palpation
- for tenderness, fractures
ASIDE - Flexor tendon sheath infection = Kanavel's 4 cardinal signs
1. symmetrical swelling of the digit

2. tenderness along flexor tendon sheath


3. flexed or semi-flexed attitude of finger
4. sever pain on passive extension DIP
Sciatica in a patient with L5-S1 nerve root involvement
Back pain history
- Eight most important questions:
1. Where is the pain worst? radiation?
2. Onset, duration?
3. Aggrevating/relieving factors
4. Previous attacks
5. Previous treatment
6. Surgery in past
7. Occupation
8. Limitations, ADL's
Ask about red flags :
- history of trauma, strenuous lifting
- tumor/infection : history of cancer, constitutional symptoms, IV drug abuse, recent bacterial infection, immunosuppresion (disease or meds)
- sx of cauda equina : fecal incont, urinary retention, frequency, neuro deficit in lower extremities, saddle anesthesia
R/O rheum causes so ask about Extraarticular features - uveitis, buttock pain, rash, heel pain, balonitis, abdo apin, etc....
PEP - physical examination
1. Observation - back shape, curvature, posture, trendelenburg
2. Back Flex/Extend - rotation, point tenderness, ROM, Gait
3. L4,L5,S1 conduction tests - sensation, relexes, heel walk, toe walk, squat
4. Irritative tests - SLR, femstretch test (L2-L4)
5. Babinski
6. Saddle senation S3-S5
L3-4 (L4)

L4-5 (L5)

L5-S1 (S1)

Pain

femoral

sciatic

sciatic

Motor

knee extension

dorsiflexion

plantar flexion

reflex

knee jerk

medial hamstring

ankle jerk

sensory

medial leg

dorsal foot

lateral foot

43 yo man requires a Liver Examination


Examination of the liver:
Upon inspection --> 5-5-5 rule :
5 signs in chest/head - spiders, jaundice/scleral icterus, parotid gland hypertrophy, gynecomastia, fetor hepaticus
5 signs in hands - palmar erythema, clubbing, terry's nails, dupretryn's, asterixis, leukonycia
5 in abdomen/pelvis - testicular atrophy, hepato/splenomegaly, ascites, caput medusae, hemrrhoids

- inspect patient's abdomen and look for distension


- percussion of liver - normal span 6-12cm (MCL)
- palpation of liver - feel along inferior costal margin, can extend across midline!!!
PEP - 3 auscultatory finding caused by the liver heard directly over liver
1. Hepatic bruit - high/turbulent flow over liver - hepatocellular ca, alc hepatitis 2. Hepatic friction rub - inflammation of liver
3. Venous Hum (epigastric) - soft humming noise with systolic and diastolic
circulation between portal and systemic

components, indicates increased collateral

venous systems, as in hepatic cirrhosis

PEP - Viruses that cause hepatitis and 2 with fecal oral spread
- Hep A,B,C,D,E, EBV, CMV
- Hep A and E and transmitted thru fecal-oral spread, both of which do NOT cause chronic hepatitis
32 yo male with a hx of spells
large differential diagnosis - try to separate seizures from syncope!!!
DDx seizures :
- epilepsy
- electrolyte disturbances - hypoCa, hypoMg, hypoNa, hypoGlc
- infectious - encephalitis, meningitis etc
- malignancy - brain tumor
- vascular - stroke, vasculitis, hypertensive encephalopathy
History for seizures:
- onset, duration, description of event (type of seizure), had it before??
- precipitating factors - noncompliance with meds, fatigue, sleep deprivation, infection, trauma, alcohol/drug withdrawl, cocaine, meningitis,
fever, uremia, hypoglycemia, stroke
- prodromal symptoms - aura (visual, auditory etc), lights, H/A, fever, chills, behavioural change
- postictal - incontinence, weakness, paralysis
Family history - epilepsy, stroke, etc
Personal history - past seizures and investigations
Medications, alcohol, drugs
DDx syncope :
Cardiovascular vs. Non cardiovascular
CVS - Reflex/Vasovagal (cough, micturition), orthostatic hypotension, drug induced, arrythmias, MI, PE, Aortic dissection, pulm HTN, Ao
stenosis, tamponade, HOCM
Metabolic - hyperventilation, hypoglycemia, hypoxia
Neurologic - CVA, NPH, seizure, increased ICP
Psychiatric - hysteria, depression
History for syncopeHistory of the event - time, description of episode, duration of unconsciousness, rate of onset, activity before and after event, body and arm
position

Precipitants - fear, hunger, pain, cough, micturition, swallowing, defecation, exertion, valsalva, hyperventilation, chest pain, palpitations
Prodromal symptoms - nausea, diaphoresis, pallor, lightheadedness, dimming vision
Postsyncopal symptoms - disorientation, confusion, vertigo, flushing, dyspnea, incontinence, weakness, time to return to normal
PMH - history of prior events, stroke, TIA, seizures, arrhythmias, DM, anxiety AND past testing such as holter, EEG's, CT scans etc
meds - especially antihypertensives, antidepressants, antiarrhythmics, insulin, and drugs of abuse
PEP treatment of seizures:
Primary generalized - carbemazepine, phenytoin, valproate
Partial seizure - carbemazepine, valproate, phenytoin, phenobarb
Absence seizure - ethosuximide, valproate, clonazepam
Status Epilepticus - ABC's, O2, IV acess
- glucose 1amp IV and thiamine 100mg IV
- initial control : lorazepam 4-8mg IV ,diazepam 5-20mg IV
- definitive control : phenytoin, phenobarb (if seizure persists)
- intubate if necessary
35 yo female with hx of oliguria - ie Renal Failure
Think pre-renal, renal and postrenal causes
History
- amount of urine output
- prerenal - symptoms of hemorrhage, heartfailure, sepsis, infection, vomiting, diarrhea, fever, chills, NSAIDS, ACE-I's
- ie blood loss, melena stools, hematemesis, chest pain, dyspnea, orthopnea, PND, palpitations, syncope, dizziness, abdo pain etc.
- renal - medications causing renal failure - radiocontrast, gentamicin, chemotx agents, aminoglycosides, rifampin, cimetidine, allopurinol,
thiazides, furosemide
- malignant HTN (H/A, blurry vision), pyelo (flank pain), HUS (skin lesions, abdo pain)
- prolonged hypoperfusion (as above)
- postrenal - BPH, stones (flank pain), urinary retention in past, Achol. meds, diabetes
Associated symptoms - dysuria, flank pain, obstructive urinary symptoms, abdo pain, hematuria, foamy urine(protein)
Past medical history - past episodes, recent UTI (post strept GN)
meds - as above
PEP - admitting investigations
- CBC, lytes, BUN, creatinine, glucose, urate, Ca, Mg, Phosphate
- Urine microscopy, C/S, culture, electrolytes, osmolality, creatinine
- 24 hour urine collection for protein, creatinine, pH, osmolality
- abdominal ultrasound
- Foley In/Out
- important initial causes of oliguria to consider include :
- CHF, hemorrhage, pulmonary edema, sepsis
57 yo man one month post MI

History
- chest pain - OPQRST, pleuritic, same type as before?
- exercise tolerance and CCS/NYHA class (based on <> 2 blocks on flat surface)
- palpitations, SOB, orthopnea, PND, diaphoresis, Nx/Vx
- cardiovascular risk factors - DM, HTN, smoking, prev MI, fam hx MI, incr. chol also ask re: obesity, sedentary lifestyle, depression
- medications - compliance with meds, difficulty in taking meds?
- if DM - glucose control, sx of hyper/hypoglycemia
- exercise routine, daily activities, ADL's
PEP
- Treatments that will affect likelihood of re-infarction
1. Risk factor modification
- glycemic control - HBA1C, daily accuchecks, optimize insulin OHA therapy
- weight loss, diet modification (Healthy heart diet)
- exercise
- blood pressure control - use beta blockers
- stop smoking
- lipid lowering agents - do fasting lipid profile, if total chol >5.5 or LDL > 2.6 - HMG CoA reductase inhibitors
2. Pharmocotherapy
- thrombolysis if meets indications and no contraindications
- anticoagulation - aspirin, heparin/coumadin (3 months)
- beta blockers - reduce mortality
- ACE-inhibitors - reduce mortality, especially good if hx CHF
- nitrates - symptomatic control only
- Define primary, secondary and tertiary prevention
Primary Prevention - preventing disease before it occurs, thereby reducing the incidence of disease (eg immunization, dietary
recommendations)
Secondary Prevention - early detection of disease in an asymptomatic period before it progresses and the treatment which may occur as a
result of screening
Tertiary prevention - attempts to reduce complications by treatment and rehabilitation, which are carried out primarily by the exsting health
care system.
37 yo man to be exmained for decreased lung volume in left hemithorax - focus posterior chest
Physical Examination :
Inspection - resp. distress (?paridoxical), cyanosis, indrawing, tracheal tug, chest movement (symmetry), use of accessory muscles
Palpation - tactile fremitus --> increased transmission of sound with atelectasis
--> decreased transmission of sound with pneumothorax
- chest excursion --> look for symmetry
- trachea midline vs shifted
Percussion - diaphragmatic excursion, normal being 3-5cm.

- percussion - areas of dullness, tympanic if pneumothorax


Auscultation - poor air entry if pneumo
- bronchial breath sounds and crackles above area of atelectasis
PEP :
- diaphragm sits at the 10th thoracic spinous process posteriorly
- PE - vitals include tachycardia, tachypnea, hypotension

pneumo

chest wall

trachea

percussion

movement

shifted

decr. over

contralat

hyper

affected side

shift

resonant

breath

advent.

sounds

sounds

decr. absent

none,

voice sounds

none

possible
pleural rub

atelectasis

decr. over

ipsilat shift

dull

affected side

decr. or

crackles,

may have

absent

bronchial

egophony

breath

above level

sounds

of atelectasis

above level
of atelect.

34 yo male drug addict assaulted while trying to rob a convenience store - examine face
Examination of the face
1. Inspection
- symmetry, bruising, swelling, deformity, bleeding from orifices, lacerations, proptosis, breathing, exophthalmos, enophthalmos,
septal hematoma, malocclusion
- basal skull fracture - lefort III - racoon eyes, battle sign, CSF otorrhea, hemotympanum
2. Palpation
a. deformities in skull --> go from top to bottom
- supraorbital rim/infraorbital rim
- maxilla, mandible/arches, nasal bones
- ?step deformity, crepitus, tender
b. intraoral exam
- grab front teeth and pull (lefort III)
- tear in mucosa/gingiva (mandibular)
3. Cranial Nerves, focusing on V
4. Feel for carotid
5. GCS, and look in ear
PEP - radiologic investigations
Structure

Best xray

Mandible

PA of mandible, Townes view (AP)


lateral obliques, Panorex

Nasal bones

no xray required

Zygomatic and orbital

CT scan is investigation of choice


Water's view, Caldwell's view
submento-vertex

Maxilla

CT scan axial and coronal

You might also like